Sei sulla pagina 1di 250

.

What gauge needle would be appropriate for a


subcutaneous injection?

25 gauge needle is preferred for subcutaneous


injections.

How should an artificial eye be cleaned?

with soap and water.

. What is the normal fetal heart rate?

120 and 160 beats per minute.

. On average, how much weight does a mother


gain during pregnancy?

25 and 30 pounds during pregnancy.

What is the best antidote for magnesium sulfate


toxicity

magnesium sulfate toxicity is calcium glutonate.

. When should a pregnant patient be tested for


alpha protein serum levels

18 to 20 weeks into gestation.

. Which of the following is NOT a factor that


causes the urine pH to be less than 4.5?

A diet high in protein, a fever, and metabolic


acidosis can all cause the urine pH to drop below
4.5.

t what age should solids be introduced into an


infant's diet?

4 and 6 months

What condition is characterized by involuntary


twitching movements of the muscles, particularly
rolling of the tongue?

The classic symptom of tardive dyskinesia is


involuntary spasms of the facial muscles,
especially rolling of the tongue.

Which foods should be avoided while on MAOI


antidepressants?

Foods containing tyramine should be avoided


while on MAOI antidepressants.

What kind of antacids are known to cause


diarrhea?

Magnesium-based antacids are notorious for


causing diarrhea

What is the best initial treatment for


hypercalcemia?

Saline and furomeside should be administered as


soon as possible during cases of hypercalcemia.

What is the most common transfusion reaction?

fever

What is another name for primal adrenal


insufficiency?

Addison's disease is also known as primal adrenal


insufficiency.

Pulse pressure (pp) is considered the

Difference between the systolic and diastolic


pressure.

maternal estrogen has been transfered to the


neonate what would the nurse see

enlarged breast tissue

molar pg

increased HCG levels

meconium in the amnotic fluid is nl for

breach position

restraints

should be removed q 2hrs and left off for 5 min for


ROM and skin checks

cool mist tent

decrease resp tract edema

a kid with celiac how can you monitor the


effectivness of tx

monitor stools

moro reflex

5 months

vacular lesions that ooze and form crust on face


and extrimites
impetigo
direct contact wash hands!

post op cleft lip

clean suture line after every feeding w/sterile


solution

cf diet

high cal high protein

pancrilipase

with meals and snacks

iron food

PB, green veggies and rasins

fontanels

should be closed by 18 months

PDA administer

indomethocin

car seat

4 or 40lbs

mo solids till

6 months

to prevent OM

place baby in upright position while feeding

preschool child

give dolls and medical stuff to play with

fetal heart tones

at 11 weeks with doppler

chylamydia

tx w/ zithromax

ectopic pg sx

abd pain vag pain and positve hcg

false labor contractions

felt in abd, irregular and relived by walking

epidural

be sure to hydrate before r/t the anestetic can cause


hypotension

apgar

hr, resp, reflex, irratiblity

going into 2nd stage of labor

uncontrolable urge push

common adverse effect of phototherapy

watery stools

rooting reflex

kid turns head to the side

cns depressant lathargy is s s/e.


mag sulfate

Tox = flushing reflex depression , decreased


urinary output, depresed rep.

folic acid

egg yolks

variable decels

change position

external monitor

lie any comfy position but try to stay off back

mild preeclampsia

htn, edema, proteinurea

placenta previa

soft relaxed non tender uterus

rh neg mom rh pos baby

give mom rhogam with in 72 hrs

to prepare for amnio

asl pt to void, assess fetal heart rate, monitor


maternal vs

premature labor

provide adequate hydration

hypogycemia in the neonate

lathargy is the inital sign

neonate wt loss

loose 10 % of birth wt in 3-4 days then nl

breast feed

q 1-3 hrs

placenta previa

painless vag bleed increase r/f after 35

ace inhibitors

inhibit k secretion

DI

give vasopressin sub q or intranassaly

mi

hypotension rapid pluse and cp

post op cataract

don't sleep on effected side

predinsone side effects

wt gain hypertension and insomnia

mefanide sulfate

topical abx for burns

neostigmine

for mg to give 45 60 min before meals to improve


chewing

type 1 hypogly event

admin 15-20 fast acting CHO

acute viral hepatitis

bedrest

bladder spasms and urge to void with foley after


turp

check for clot by ns irrigation at 40-60 gtts q min


amount instilled should be the amount with drawn
if patent

nsaids

w/food to prevent gi upset

autonomic dysreflexia

monitor foley patancy can precipitate condition

asending or transverse colonoscopy

liquid or simi liqued stools

myasthenia gravis

asses resp system and muscle weakness

long term predinsone

osteoporosis

ca

premeno 1000
post meno 1500 mg q d

rule of 9's

trunk post and ant and legs make up 18% . head


neck and arms 9%. perineum 1%

common allergies

berries, nuts and eggs

stoma irrigation

elevate bsg 18-24 inches above stoma,


500-1000ml lukewarm h20 if cramping stop
irrigation hold breath then resume

hip replacement sitting

limit flexion to 90 degrees

tens machine

blocks painful stimuli from traveling to small


nerve endings

p thyroidectomy

tetany may occur monitor for s/s of muscle


twitching tingling on numbness

gcs

3 indicates coma

chest tube is disconnected

place end in sterile water to prevent air from


entering chest tube

crutches

all weight on the hands

bed bath water temp

110-115

bacterium responsible for tetnus

colstridium tetani

tpn

monitor wt q d

positive ppd

induration and swelling 5-15mm

iron def anemia sx

dyspnea tacycardia pallor and fatigue

hemmacult rules

no red meat turnups or pultry or fish for 4 days

laminectomy

logroll pt

antacids w. aluminum

monitor for constipation

A nursing assessment on a male patient indicates


that he is showing evidence of increased
intracranial pressure. The first nursing action is to
Answer1.1= Place him in high-Fowler`s position.

= Place
him on his
side.with cystic
Question.2=
A 9-year-old
patient
fibrosis will take pancreatic enzymes 3 times a
day. The nurse will know the child`s mother needs
= Have him
cough
and deep
breathe.of these
more education
on the
purpose
and timing
enzymes if she says
= "The purpose of the enzymes is to help digest
the fat in foods."
= "The enzymes should be taken prior to meals."
= "They should be given following breakfast,
lunch and dinner."
= "They should be taken at meal times, 3 times a
day."

Attempt to have him deep breathe.


As the PaCO2 increases in the cerebral tissues,
blood rushes to the area and this further increases
the intracranial pressure. Decreasing the PaCO2,
accomplished by breathing deeper and more
slowly, will decrease the intracranial blood flow,
thus decreasing intracranial pressure.

= The purpose of the pancreatic enzymes is to


replace the enzymes unavailable in the child`s
system that assist with the digestion of fats.
Therefore, they should be taken prior to, not
following, the ingestion of food.

= Formulating a care plan for a patient with


chronic lymphocytic leukemia, the nurse will
expect that his laboratory results will reveal
increased

Lymphocytes.
= Monocytes.
= Platelets.
Question.4= As=part
the nursing
Redofblood
cells. care plan for a
patient with chronic lymphocytic leukemia, the
nurse will formulate nursing diagnoses. A priority
Nursing Diagnosis is

= Infection, potential for.


The nurse
is assignedina tissue
patientperfusion.
diagnosed with a
= Alteration
left-sided hemopneumothorax who has a chest
tube connected
to a water-seal
drainage system.
= Impairment
of skin integrity.
During the =
shift
change
assessment,
Fluid
volume
deficit. the nurse
observes that fluid in the suction control chamber
is bubbling. The intervention is to

Add water to the water-seal chamber.


= Make sure that there are dependent loops in the
= The nurse is assigned
to dry and wrap a newborn
system.
baby. The nurse will don gloves to complete this
task because
= Do nothing,
as this is normal.
It is important
to observe
universalsuction.
precautions
Sit the patient
up to increase
before the infant`s first bath.
= The infant requires protection from infection.
= It is the nurse`s choice whether or not to wear
On the first postpartum
day, a mother asks the
gloves.
nurse to bathe and change the baby. The best
response
is
= Gloves are standard
procedure
for care of
normal newborns.
Do you remember how you bathed your last
baby?"
= "It`s better if you care for the baby."
"How does it feel to have a new baby to care for?"

= "I`d be glad to."

= White blood cells (lymphocytes) increase in


number, although they are immature. With this
condition, there is a decreased number of red
blood cells and platelets.

Immature white blood cells predispose the patient


to infections, so this nursing diagnosis is a priority.
Fluid volume deficit may also be an important
nursing diagnosis, because the patient may be
prone to bleeding. It does not, however, have as
high a priority as (1)

= Bubbling in the suction control chamber is


expected. The tubing should always be free of
kinks without dependent loops. The patient should
be turned side-to-side to promote drainage. While
chamber levels must be maintained, bubbling in
the suction control chamber does not indicate a
need for adding water.
#2
With the high increase in numbers of HIV positive
newborns, the Centers for Disease Control (CDC)
guidelines include wearing gloves when handling
newborns before their first bath. They will be
covered with amniotic fluid and other secretions
from the mother, so it is important to be protected
from these body fluids which could harbor the
HIV virus.
#3
= It is important to find out how the mother feels
about baby care. She may just be tired, she may be
afraid, or she may have some other reason for her
request. The nurse needs to assess further, so this
question is the most relevant.

incontinent and soiled his clothes. This is the


second time this has happened on this shift. The
most appropriate nursing intervention is to

Put the patient in adult diapers to protect him from


embarrassment.
= Scold the patient and tell him not to wet his
pants again.
= Tell the patient to ask the nurse for assistance
A the
50-year-old
is to receive
next timepatient
he haswith
to goasthma
to the bathroom.
Aminophylline. The nurse will evaluate the
effectiveness of Aminophylline by observing for
decreased
= Tell the patient the
nurse will change his pants
and establish a 2-hour schedule of taking him to
Respiratory
distress.
the bathroom.
Question.10= A very= attractive
Rales. young man with
whom the nurse has had a nurse-patient
relationship=begins
make sexual advances
Mucustoproduction.
toward her, making her very uncomfortable. The
best=Pulse
approach
rate.is to
Ignore the advances, for the nurse knows that lack
of reinforcement usually extinguishes the
behavior.
= Tell his doctor, who should be informed of his
inappropriate behavior on the unit.
= The nurse caring for a patient with an ileostomy
= will
Be direct
in communicating
discomfort
with
carefully
assess the skin the
around
the stoma
his advances and set
limits
on
his
behavior.
because
= Continue the relationship, for the nurse knows
ending
it will
reinforce
his negative
It is very
difficult
to ensure
properself-image.
fit of the
appliance with skin breakdown.
= Digestive enzymes may cause skin breakdown.
Question.12=
Which
onethan
of the
following
= The
effluent is more
solid
watery
and may
treatment plans
most
in the
stickis to
the appropriate
skin.
nursing/medical management of a 10-year-old
child
with to
congestive
failure? stoma.
= It takes
longer
heal thanheart
a colostomy
High concentrations of oxygen, strict bedrest,
diuretics.
Answer2.12= Oral fluids, daily weights,
high-Fowler`s position.
Answer3.12= High-Fowler`s position, digoxin,
diuretics, daily weights.
Answer4.12= Digoxin, diuretics, prophylactic
antibiotics.

#4
= Even though the nurse may eventually have to
place diapers on the patient, this is not the first
intervention. An every 2-hour bathroom schedule
may solve the problem because the patient cannot
remember to tell the nurse when he needs to
urinate.

#1
= Aminophylline causes bronchodilatation and,
therefore, increased oxygenation. The patient is
more able to cough up secretions and the breath
sounds will become more clear. The CNS is
stimulated by this drug and the pulse rate may
increase.

=The patient needs direct feedback and clear


delineation of limits to the relationship for
example, "This is not acceptable behavior. If you
continue, I will not talk with you."

#2
Rational.11= The discharge from an ileostomy is
watery and contains many digestive enzymes that
have not been absorbed by the intestinal villi.
These enzymes may cause skin breakdown. None
of the other answers is accurate.
#3
Rational.12= Digoxin and diuretics are
administered in order to increase the force of
systolic contraction and to decrease venous
volume/congestion. High-Fowler`s position is
utilized to provide for maximum lung expansion.
Daily weights allow for constant monitoring of
fluid balance/status. Antibiotics are not
administered prophylactically.

Question.13= The nurse will know that a patient


undergoing radiation therapy understands the side
effects of the therapy when he says that he
schedules his meals
#2
Immediately before therapy.

Answer2.13=Two to 3 hours before therapy.


Question.14=
The most
Answer3.13=
One appropriate
hour before activity
therapy.or toy
for a 4 year old hospitalized for severe rheumatoid
arthritis is after therapy.
Answer4.13=Immediately
A VCR with a Disney movie or a radio.
Answer2.14=An erector set or Legos.
Answer3.14= A dartboard and darts or marbles.
Answer4.14=A wooden puzzle or crayons and
coloring
book intervention for a
Question.15= A priority
nursing
patient on total parenteral nutrition (TPN) is to
monitor the

Urine specific gravity.


Answer2.15=Sodium
level.patient was
Question.16=
A 42-year-old female
sugar
admitted Answer3.15=Blood
to the psychiatric unit
withlevels.
a diagnosis of
Answer4.15=Urine
pH.
acute depressive episode. She refuses to get out of
bed, go the dining room, or participate in any of
the unit`s programs. The most appropriate nursing
action is to

Rational.13= Food and fluids should be taken 2 to


3 hours before or after the treatments to decrease
the nausea experienced as a result of the
radiotherapy.
#4
Rational.14= A puzzle or crayons would be
appropriate for a 4 year old who is limited in
activities by his diagnosis. An erector set is too
advanced for his developmental level. A video
movie or radio is too passive and will not
encourage intellectual or creative development.
Darts are dangerous for a 4 year old the dart may
accidentally hit someone or something other than
the dartboard.
#3
Rational.15= Because parenteral nutrition
solutions are very high in dextrose, the pancreatic
insulin-secreting ability may not be sufficient to
maintain normal blood glucose levels. The patient
may require insulin during all or part of the TPN
course. There should be no change in the sodium
levels, as these solutions are not high in sodium
the patient should be in acid-base balance during
the therapy.
#3
Rational.16= Be positive, definite and specific
about expectations. Do not give depressed patients
a choice or try to convince them to get out of bed.
Physically assist the patient to get up and dressed.
This activity will help to mobilize her which, in
turn, will help the depression to lift.

Allow her to remain in bed until she feels ready to


join the other patients.
Answer2.16= Suggest she may be hungry later so
she had better get out of bed.
Answer3.16=
Tell
that is
thescheduled
nurse willfor
assist
Question.17=
A her
patient
laserher
out
of
bed
and
help
her
to
dress.
surgery to correct his retinal detachment. Before
Answer4.16=
Tellthe
hernurse
the rules
the unit
are that
the surgery,
will of
position
him
no patient can remain in bed.
Rational.17=The retinal detachment area must be
On total bedrest with the area of detachment
kept dependent (lowest part of the eye) in order to
keep the separation between the retina and its
dependent.
Answer2.17= In bed, in a prone position with his circulatory layer, the choroid, minimal. The patient
head supported by pillows.
should be on bedrest.
Answer3.17= On his right side with the area of
detachment in upward position.
Answer4.17= Up ad lib, but not turning his head.

Question.18= Postoperatively on the same day as


surgery for a right hip replacement with a
prosthesis, the patient may be positioned on

The patient`s back and left side.


Answer2.18=The patient`s back and right side.
Answer3.18= Either side.
Answer4.18=The patient`s back only
Question.19= A patient is several days
postoperative following a right hip replacement
with a prosthesis. Assisting the patient to sit in a
chair, the nurse will use

he lowest possible chair with the back reclined.


Answer2.19=A reclining chair with foot rest
elevated.
Answer3.19= A soft chair with arms.
Answer4.19=An upright elevated chair.
Question.20= To facilitate breathing in a child
with bronchiolitis, the nursing care plan will
include establishing an environment of

Humidified oxygen.
Question.21=
A patient has
a femoral
Answer2.20=Warm
mistsustained
with oxygen.
shaft fracture
and
is
being
treated
with
skeletal
Answer3.20= Cool, moist oxygen.
traction
using balanced suspension
withnoa mist.
Thomas
Answer4.20=Oxygen
therapy with
splint and Pearson attachment. The goal of
maintaining optimum positioning will be
accomplished by

Keeping the affected knee in a position of


extension.
Answer2.21= Maintaining the lower leg at a 90
degree angle to the upper leg.
Answer3.21= Maintaining the angle between the
affected
and athe
bed.catheter in a
Question.22=
While thigh
inserting
Foley
Answer4.21=
Notnurse
elevating
the head the
of the
bed
female
patient, the
will advance
catheter
above
ten
degrees.
through the urethra a distance of
1 to 1 1/2 inches.
Answer2.22=2 to 3 inches.
Answer3.22=Until urine flows.
Answer4.22=1 1/2 to 2 inches.

#4
Rational.18= A total hip replacement requires very
careful attention to position. The joint may
dislocate in the early postoperative period and the
nurse must maintain the affected limb in a position
of abduction and external rotation. Turning to
either side in the early postoperative stage would,
therefore, be disallowed.

#2
Rational.19= The ball in socket position is
maintained by minimizing hip flexion (60 degrees
or less). This is done with wheelchair and
commode extenders, high chairs, and proper bed
positions.

#3 Rational.20= Cool, moist oxygen is the


supportive therapy of choice for bronchiolitis. If
the child is at home, warm mist from the shower
may be administered for croup. Humidified
oxygen is administered for pneumonia.

#3
Rational.21= It is important that the established
angle between the affected thigh and the bed be
maintained. The patient can usually have the head
of the bed flat or elevated and the lower leg can be
exercised, then rest in the Pearson attachment. If
the patient migrates toward the head or foot of the
bed, then the angle between the thigh and bed
would be altered, so it is important that adequate
countertraction be maintained.

#2
Rational.22=The Foley catheter should be
advanced 2 to 3 inches into the female urethra.

Question.23= The nurse is caring for a patient


undergoing chemotherapy for cancer. One of the
goals of care is to discuss possible side effects of
this therapy, one of which is
Constipation.
Answer2.23=Alopecia.
Answer3.23=
Increased
appetite.
Question.24= The nurse
will know
a diabetic
Answer4.23=Weight
patient understands
exercise andgain.
its relation to
glucose when the patient tells the nurse that he eats
bread and milk before, or juice or fruit during
exercise activity because

Exercise stimulates pancreatic insulin production.


Answer2.24= A diabetic`s muscles require more
glucose during exercise.
Answer3.24= Exercise enhances the passage of
Question.25=
Which
of muscle
the following
glucose
into
cells. behaviors
would indicate
improvement
in the
coping
Answer4.24=
The pancreas
utilizes
more
glucose
behavior of a patient
who
has
the
diagnosis
of
during exercise.
depression?

Sleeping frequently during the day.


Answer2.25=Going to the dining room to eat.
Answer3.25= Turning off the television to have it
quiet.
Answer4.25=Initiating interaction with another
Question.26= Whenpatient.
a patient`s hallucinations
become louder, more demanding and difficult to
ignore, the nurse would judge that his mental
status is

#3
Rational.23= Alopecia, or hair loss, will probably
occur caused by damage to the rapidly dwindling
cells of hair follicles. Hair loss begins 2 to 3 weeks
after therapy begins and continues through the
course of therapy. The other side effects listed do
not occur, as nausea, anorexia and diarrhea are
common.

#3
Rational.24= Exercise promotes the passage of
glucose into muscle cells, so extra complex
carbohydrates are essential before or during
regular exercise. To prevent precipitous
hypoglycemia, the IDDM patient should inject
insulin into nonactive areas, such as the abdomen.

#4
Rational.25= If a patient initiates interaction with
another patient, it indicates he is not totally
absorbed in himself, too depressed to initiate a
conversation, or too preoccupied to focus on the
television. The other choices do not necessarily
indicate improvement in coping.

Rational.26=The more demanding and absorbing


hallucinations (hearing voices) becomes, the more
the patient`s condition may be deteriorating.
Secondarily,
this may indicate increased paranoia,
Question.27= A 3-day-old infant is admitted to the
mproving.
pediatric floor from the
nursery with a diagnosis of but not necessarily since paranoid schizophrenia is
=Showing
more
evidence
paranoia.
only one form of this condition.
esophageal atresia type III. His of
mother
comes to
= Remaining
visit her baby
and says tothe
thesame.
nurse, "I feel as
though I`ve done=Deteriorating.
something wrong to make my
child sick." The most appropriate response would
be to

can understand your feelings, but remember that


this is a congenital defect you did not cause."
"It does no good to feel that way. Your child is
sick and needs you. You should spend your time
caring for him."
= "Don`t be silly, your child was born with this.
You`ve done the best you can."
= "A lot of mothers feel guilty when their child is

#1
Rational.27= The nurse acknowledges the
mother`s feelings, but tries to show that they are
not based on fact.

Question.28= A 43-year-old female patient had a


subtotal gastrectomy several weeks ago. She has
been readmitted with a diagnosis of pernicious
anemia. Taking her nursing history, the nurse
expects to find that her diet has probably not
included

Eating enough foods high in iron.


Answer2.28=Taking
herisvitamin
B12. with
Question.29= A patient who
hospitalized
= Eating
enough
high
vitamin
chronic
congestive
heart
failure
hasB12
beenfoods.
placed on
telemetry. The nurse should know that the purpose
of this type of monitoring is to
=Taking her iron supplements.
dentify ventricular arrhythmias resulting from
hypoxia.
= Identify ventricular arrhythmias resulting from
hyperkalemia.
Question.30= A neighbor
asks the nurse to look at
= Assess
atrial
ectopic
beats
from
her
3-year-old
child`s
rash.
Herresulting
face, neck
and
hypoxia.
chest are covered with a maculopapular rash. She
= Assess
ectopic
resulting
fromare
appears
feverish
andbeats
her nose
and eyes
hyperkalemia.
"running." There
are small red spots with
bluish-white centers on the mucosa of her mouth.
The nurse recognizes that these manifestations are
most likely caused by

Measles (Rubeola).
= Impetigo (Staphylococcus).
= Scarlet fever (Beta hemolytic streptococcus,
Group A).
Question.31=
In
each
prenatal(Rubella).
visit, the assessment
= German measles
technique that evaluates appropriate fetal growth is
Measurement of fundal height.
=A nonstress test.
= Maternal weight gain.
=Blood pressure reading.

Question.32= A direct Coombs` test is done on the


cord blood of a newborn to detect the presence of

Antigens coating the mother`s red blood cells.


=Antibodies coating the baby`s red blood cells.
= Maternal red cells in the fetal circulation.
=Fetal red cells in the maternal serum.

#2
Rational.28= Patients who have pernicious anemia
following a subtotal gastrectomy do not have
enough intrinsic factor to utilize vitamin B12 in
foods and must be given injections or sublingual
doses of vitamin B12.

#1
Rational.29=When the lungs are filled with fluid,
oxygenation is not as efficient as it should be.
Hypoxia can cause ventricular arrhythmias, a
dangerous condition. These are identified easily by
the use of telemetry.

#1
Rational.30= Rubeola is a highly contagious virus.
It is more severe than rubella or roseola because of
the complications. The virus is transmitted by a
cough or sneeze.

#1
Rational.31= The fundus height changes
throughout pregnancy. The height of the fundus is
at the level of the umbilicus by 28 weeks and near
the xiphoid process by 38 weeks.

#2
Rational.32= The direct Coombs` test is done on
cord blood to detect the presence of maternal
antibodies attached to the neonate`s red blood
cells.

Question.33= Persons with personality disorders


tend to be manipulators. Which principle is it
important for the nurse to know in planning the
care of such a person?
#3
Rational.33=Because a person with this disorder
tends to manage his or her life through
The nurse should allow herself to be manipulated manipulation of others, when it doesn`t work, their
anxiety level goes up. The nurse should never
so as to not raise the patient`s anxiety.
= The establishment of a nurse-patient relationship allow the patient to manipulate her. Answers (2)
and (4) are not true.
will decrease the patient`s manipulations.
= When the A
patient`s
arebiopsy
not
Question.34=
patient manipulations
is aware that the
his anxiety
will
increase.
report successful,
has just returned.
As the
nurse
is changing
=
The
nurse
should
appeal
to
the
patient`s
sense
of
his dressing he says, "Do I have cancer?" The
most
loyalty appropriate
in adhering response
to the rules
of the
community.
at this
time
is
#1
Rational.34= This response allows the patient to
discuss fears about cancer and does not close off
"Would you like to discuss your concerns about
communication, as the other three responses do.
the results?"
= "Did your physician tell you the results?"
= "I`ll call your doctor and ask him."
=
"We
do not
know yet;
report hasarthritis
not
Question.35=
A patient
withthe
rheumatoid
has been on aspirin returned."
gr. xx TID and prednisone 10
mg BID for the last 2 years. The most important
#3
assessment question related to the patient`s drug
Rational.35= Aspirin impedes clotting by blocking
therapy is to ask her if she experiences
prostaglandin synthesis which can lead to
bleeding. A side effect of prednisone is gastric
irritation, also leading to bleeding. Tarry stools
Blurred vision.
indicate bleeding in the upper GI system.
=Decreased appetite.
= Tarry stools.
Question.36= A 40-year-old
male patient has the
=Headaches.
diagnosis of myasthenia gravis; the disease is
Rational.36= Myasthenia is a disease characterized
adequately controlled on Mestinon. The patient
by marked weakness and abnormal fatigue of
tells the nurse that sometimes he gets very tired
voluntary muscles. A deficiency of acetylcholine
when he eats. Nursing instructions should be to
results in a defect in transmission of nerve
impulses. Mestinon, an anticholinesterase
medication, increases levels of acetylcholine.
Eat several small meals each day.
Taking the medication before meals will assist in
=Eat foods
rich in vitamin
B6. finds a
Question.37=
One afternoon
the nurse
mastication.
= Take patient
Mestinon
30 minutes
before meals.
paraplegic
sitting
in her wheelchair
crying.
=Try
foods
until
thiscan
symptom
decreases.
Whenpureed
the nurse
asks
if she
help, the
patient
says, "Go away, no one can help." The best
nursing response is
#1
Rational.37= The best response is to acknowledge
she told the nurse to go away but to leave
communication open for a later time. Other
"I understand that you don`t want to talk right
responses put pressure on the patient or close off
now. I will come back and perhaps we can talk."
= "You must be having problems."
communication.
= "Many people feel sad in the hospital. You`ll be
home soon."
= "It`s better if you talk about it. Maybe I can
help."

Question.38= An adult male patient is being


discharged after his peptic ulcer has been
controlled on a medical regimen. One medication
he is taking is sucralfate (Carafate). The nurse asks
if he understands when he should take the
medication. The correct answer would be

Just before meals.=One hour before meals and at


bedtime.
Question.39=
A
new
is told that on
= In earlymother
morning.
admission to the=With
nursery
an injection
every
meal. of vitamin K
is given routinely to normal newborns. The nurse
explains that vitamin K is given because

#2
Rational.38= Carafate is a mucosal protective
agent with antipepsin activity. It is taken on an
empty stomach, 1 hour before eating, and at
bedtime.

#2 Rational.39=Vitamin K is used to promote a


normal clotting time until bacteria, necessary for
vitamin K synthesis, is present in the colon.
Vitamin K does not protect the newborn from
hyperbilirubinemia, nor does it play a role in blood
glucose regulation.

ewborns have an increased ability to clot blood.


= Vitamin K promotes a normal clotting time.
= Vitamin K prevents hypoglycemia.
Question.40=
A 60-year-old
patient
has been
= Newborns
need to bemale
protected
from
diagnosed with
heart failure and is started on
hyperbilirubinemia.
Digoxin 0.25 mgm PO QD and Lasix 40 mgm PO
BID. After 2 days, he develops an irregular pulse
rate of 64. The nurse would expect the physician to #4 Rational.40=Potassium loss secondary to Lasix
order
administration may cause arrhythmias and
potentiate the risk of digitalis toxicity. Therefore,
potassium is often ordered when Lasix and
digitalis are prescribed.
Calcium gluconate.
=Magnesium sulfate.
= Atropine sulfate.
=Potassium
chloride.
Question.41= When a patient
experiences a severe
anaphylactic reaction to a medication, the nurse`s
initial action is to
#3 Rational.41= Initially, the shock position is
necessary to maintain vital signs. The other
Prepare equipment for intubation.
interventions may be carried out, but will follow
=Start an IV following standard orders.
the initial action.
= Place the patient in a supine position.
=Assess the patient`s vital signs.
Question.42= The most appropriate needle used
for administering an intramuscular injection into
the deltoid muscle of an adult patient is a

25 gauge, 5/8" needle.


=22 gauge, 1" needle.
= 25 gauge, 1 1/2" needle.
=22 gauge, 1 1/2" needle.

#1
Rational.42=A 23 or 25 gauge 5/8" needle is the
most appropriate size when using the deltoid
muscle.

Question.43= Which of the following statements


best explains why premature infants are more
likely to develop an infection than full term
newborns?

Premature infants receive few antibodies from the


mother, because antibodies pass across the
placenta during the last month of pregnancy.
= Liver enzymes are immature in the premature
infant.
= Antibody formation does not mature until 6
weeks after birth of a full-term infant.
Question.44=
premature
infant has
the diagnosis
= The whiteAblood
cell defense
system
is only
of respiratory distress
syndrome
(RDS).
The
nurse
effective at full term.
understands that this disorder may be caused by
Low concentration of oxygen.
=The inability to produce surfactant.
= Cold stress.
=A genetic defect.
Question.45= A 70-year-old female Alzheimer`s
patient has been wandering in and out of other
patients` rooms. To deal with this behavior, the
most therapeutic nursing action will be to

Lock the other patients` rooms.


=Restrain the patient in her room.
= Confine the patient to the dayroom.
=Tell the patient she shouldn`t do tha
Question.46= A male patient with bronchospasms
has prednisone prescribed. The nurse`s teaching
instructions before he is discharged from the
hospital will include taking prednisone

With milk.
=With orange juice.
= Before meals.
Question.47= A=After
patientmeals.
comes to the walk-in
mental health clinic where the LVN is assigned to
do intake histories. To determine the presence of
depression, the primary sign or symptom the nurse
will assess for is

Mood with affect of sadness.


=Episodic euphoria.
= Loss of reality focus.
=Negative behavior.

#1
Rational.43= Antibody formation is immature in
the premature infant. Immaturity of the liver is
responsible for hyperbilirubinemia but is not
directly related to the infant acquiring an infection.
White blood cells may be ineffective, but that is
not necessarily related to full-term birth.

#2Rational.44=RDS often occurs in premature


infants due to decreased surfactant production in
the lungs. A high concentration of oxygen can also
result in this condition.

#3
Rational.45= The most therapeutic action is to
keep the patient with others in the dayroom, not
alone in her room. Telling her not to go into
others` rooms will not change her behavior
because Alzheimer`s patients have little short-term
memory. Locking rooms may be perceived as
punishment by the other patients.

#1 Rational.46= Prednisone may cause a stress


ulcer, so it should be taken with milk to protect the
lining of the stomach. Iron is given with orange
juice to increase assimilation.

#1 Rational.47= The distinguishing quality of


depression is mood, where the affect is sadness or
gloom. Negative behavior could be present, but
also accompanies other diagnoses. Loss of reality
is seen in schizophrenia, and euphoria is a
symptom of manic episode.

Question.48= The physician orders a nasogastric


tube for a female patient. As the nurse prepares to
insert the NG tube, the patient begins to cry
silently. The best way for the nurse to respond to
this behavior is to say

Don`t worry, I will give you pain medicine if you


need it."
= "It is all right to cry."
= "Can youAtalk
aboutscheduled
why you for
are surgery
crying?"has
Question.49=
patient
=
"The
insertion
of
the
tube
will
not
be
had preoperative teaching. An indicationpainful."
that the
teaching was successful would be that the patient

Does not question what the nurse said.


= Appears relaxed and can verbalize what the
nurse taught.
= Says he understands and wants to be alone now.
Question.50=
Aftertoa be
male
patient
returns from
= Appears
resting
quietly.
surgery following small bowel obstruction, he
complains of pain and asks for medication. The
first nursing action is to

Determine the location and severity of the pain.


=Administer the medication that is ordered.
= Give him half the ordered dosage.
Question.51=
A female
has been admitted
=Check
with patient
the physician.
with a diagnosis of compulsive disorder. In talking
with her one day, she tearfully asks the nurse, "Do
you think I`m crazy?" The nurse`s best response
would be

That depends on your definition of crazy."


="You`re upset. Let`s talk about something else."
= "I think you`re upset, but not crazy."
="Are
youThe
concerned
about
crazy?"
Question.52=
physician
hasbeing
ordered
IV fluids
at 100 mL/hr. Which one of the following
parameters is the most important indicator of fluid
balance?

Adequate intake measurement.


=Quality of skin turgor.
= Consistent vital signs.
=Hourly urine output.

#3Rational.48= Crying is not a normal response to


an NG tube insertion, so the nurse needs to find
out what has occurred in the patient`s past to
evoke such a response.

#2Rational.49= The person who understands


something can explain it to someone else. The
other responses are typical of stress responses
and/or denial.

#1Rational.50=Because the pain might be caused


from his position on the operating table or the
position of his arm with the IV, it may be relieved
without medication by actions such as
repositioning or giving a back rub. The patient
should receive medication if he needs it, but
medication should not be used in place of good
nursing care. The first dose of narcotic following
surgery may be only one half the ordered dose, but
an assessment of the pain should be done first.

#4 Rational.51=This is the only response that


opens communication. The nurse should
encourage the patient to express what she is
thinking and feeling in order to assist her to deal
with her fears.

#4 Rational.52= Hourly urine output is the best


indicator of fluid balance. Vital signs will change
however, they are slower to respond to fluid
changes than alterations in urine output.

Question.53= The nurse assigned to a woman in


labor has orders to begin an oxytocin (Pitocin)
drip. After 20 minutes of the Pitocin infusion, the
nurse observes a contraction that does not relax
after 90 seconds. The nurse`s first action is to

Start oxygen by mask.


=Stop the Pitocin drip.
= Notify
Question.54=
Whenthe
thephysician.
nurse is caring for a
=Turn
patient
side. signs
patient in
labor,the
which
oneonofher
theleft
following
indicates that the placenta has detached from the
uterine wall?

The uterus falls below the level of the symphysis


pubis. s
= There is a noticeable relaxation in the abdominal
wall.
=
The
mother
complains
of pain
lowwas
in her
back.
Question.55= A 48-year-old
patient
admitted
= The
cord
lengthens
outside
the interstitial
vagina.
2 days
ago
with
the diagnosis
of of
acute
pancreatitis. The nurse will evaluate for the most
common and fatal complication of severe acute
pancreatitis, which is

Severe hyperglycemia.
=Hypovolemia.
= Electrolyte imbalance.
=Infection.
Question.56= In counseling a patient, which
measure will promote the most comfort during a
Herpes Simplex Virus, type 2 (HSV-2) outbreak?
Keep the lesions clean and dry.
=Apply Acyclovir as ordered.
= Take sitz baths 3 to 4 times a day.
=Apply a local anesthetic or systemic analgesia.
Question.57= A male patient is admitted who is
semicomatose, dyspneic, and weak. His admitting
diagnosis is HIV immune depression. The
statement that best describes the source of this
disease is a/an

Mutated virus that kills helper T-cells.


= Autoimmune disease that is caused by the
release of an antigen.
= Virus that invades the immune system.
= Retrovirus that attaches to the helper T-cells.

#2Rational.53= If a contraction lasts longer than


90 seconds, the safe correct action is to turn off the
Pitocin. Prolonged contractions can result in a
ruptured uterus. The nurse may then administer
oxygen and call the physician.

#4Rational.54= As the placenta is expelled and


moves toward the cervix, the cord will precede it
and appear to get longer.

#2Rational.55= This condition causes large


amounts of serous fluid to leak from the blood
channels into the peritoneum, which decreases the
intravascular volume. Hypovolemia can occur.
Enzyme release may also damage the walls of the
blood vessels and precipitate hemorrhage.

#2 Rational.56=Acyclovir applied to the lesion


will result in earlier remission and also provide
more comfort. There is no known antibiotic for the
virus. Acyclovir applied to the lesions shortens the
episode but does not cure it.

HIV is caused by a retrovirus with a different life


cycle from that of a normal virus. It may be
dormant for years before affecting the helper
T-cells and causing immunosuppression.

Question.58= A female patient, age 58, has been


hospitalized for 2 days with chronic congestive
heart failure. While administering morning care,
the nurse observes that the patient frequently
removes her oxygen mask even though she is
dyspneic. The appropriate nursing intervention is
to
Increase the liter flow of O2 to maximum level.
=Change from O2 mask to O2 cannula.
Question.59=
female patienttohas
been admitted
= Change O2Aadministration
a Venturi
mask.
to the
coronarythe
care
unitonwith
a diagnosis
=Tighten
strap
the O2
mask. of
myocardial infarction. She is placed on a cardiac
monitor and has an IV of D5W at a "keep open"
rate. Initially, the nurse will assess for

Chest pain characteristics.


=Apical pulse.
= Blood pressure. D
Question.60= A =Heart
nurse colleague
sounds. working on the
pediatric unit is newly married and wishes to
become pregnant. Which of the following tests
would it be important for her to have as soon as
possible?
Lipoprotein test.
=Papanicolaou (PAP) smear.
=
Serologic
testfollowing
for Hepatitis
A.
Question.61=
Of the
interventions,
=Rubella
Serologic
test.if the cord
which action
shouldViral
the nurse
take first
has prolapsed in a woman in the final stage of
labor?

Pushing upward on the presenting part with a


sterile-gloved hand.
= Reporting the fetal heart rate to the physician
Question.62= A immediately.
mother in labor arrives at the
emergency
room.
An
initial
assessment
indicates
= Replacing the cord
very
slowly with
a
that crowning issterile-gloved
occurring. The
first nursing action
hand.
is toonto her left side.
= Turning the patient
sk the patient to push according to the nurse`s
instructions.
Apply gentle perineal pressure to prevent rapid
expulsion of the head.
= Notify the charge nurse or physician.
= Instruct the patient to take short shallow breaths
to improve fetal oxygenation.

#2Rational.58= Patients often feel that they cannot


breathe when experiencing pulmonary edema. A
mask may increase this feeling. A cannula is often
better tolerated and thus should be used in this
case.

#1 Rational.59= The presence of chest pain may


cause the pulse rate to increase and blood pressure
to drop. It can also increase patient anxiety. The
nurse`s priority is to assess the patient for pain.
The patient is on a cardiac monitor therefore, an
apical pulse is not the priority action. Once her
comfort has been established, apical pulse, blood
pressure, and heart sound determinations are
appropriate.

#4Rational.60= If the woman is planning to


become pregnant, exposure to the rubella virus is
important to detect. If a woman is exposed and she
is not immune, the fetus may have congenital
abnormalities at birth. A Pap smear detects cancer.

#1Rational.61= The first action is to move the


presenting part off the cord to allow oxygen to
flow via the cord. Other methods that can be used
to accomplish this are Trendelenburg or knee-chest
position. Turning to the left side (1) will not help
(2) and (3) are wasting valuable time.

#2Rational.62= The first action is to support the


perineum to prevent tears and rapid delivery. The
nurse would ask the patient to take short breaths
(following instructions) but to prevent pushing the
nurse will also send someone for a physician.

Question.63= A patient in labor is experiencing


severe back pain. The best source of relief for the
mother would be for the nurse to
Divert her attention to the TV.
=Apply counterpressure to her sacrum.
Question.64=
At theher
well-baby
clinic, a mother
= Have
walk around.
tells the nurse
sheher
is worried
about
her 1 month old
=Give
a narcotic
analgesic.
"catching something" before she gets her shots.
What would be the most appropriate response?

Protection from your antibodies will last until she


is at least 1 year old."
= "She has passive immunity to tetanus and
pertussis but not to diphtheria so she needs to be
protected from it by early shots."
= "She has active immunity from you so she won`t
be affected by any of the usual childhood
Question.65= Before
administering an
diseases."
immunization
to
an
infant,
is important
that the
= "The baby is protected forit the
first few months
nurse remember
a primary
contraindication
for
of life that
so you
need not
worry."
immunization is

An active infection.
=Cystic fibrosis.
= Congenital heart disease.
to thrive.
Question.66= A=Failure
41-year-old
male patient has had
recurrent sharp flank pain, nausea, and vomiting
for 24 hours. He is admitted to the hospital for a
genitourinary work-up. Which of the following
orders would be considered a priority?

Temperature every 4 hours.


=Strain
allvisit
urine.
Question.67= On
a home
to a male patient
= Administer
an antiemetic
hours.the
who
has had multiple
sclerosisevery
for 204years,
=Accurate
intake
and
output
records.
nurse reinforces the nursing care the wife provides
to her husband. Which of the following measures
would be most appropriate to include in the
teaching sessions?

Instruction in weight control.


= Exercises that promote muscle strengthening and
decrease tremors.
= Importance of regular bowel and bladder
evacuation.
= Side effects of routine medications.

#2Rational.63=Applying counterpressure will help


to release the pain. Moving the patient into
knee-chest position is also good for back pain.

#4Rational.64= The mother can be reassured by


knowing that passive immunity for diphtheria,
tetanus and pertussis are all transferred from
mother to infant during fetal life. There is very
little functioning maternal antibody in the infant
after age 6 months.

#1 Rational.65=Children with active infections


such as impetigo should not be immunized until
the infection has passed.

#2Rational.66= The patient has symptoms


indicative of a urinary calculus therefore, it is
important to strain all the urine in order to detect if
the stone has passed.

#3Rational.67= Bowel and bladder retention or


incontinence is a major problem of patients with
multiple sclerosis therefore, establishing a good
routine for evacuation is essential.

Question.68= The nurse observes a patient


walking down the hall carrying a knife in his hand,
and saying in an angry voice, "Get out of my way.
Don`t try to stop me." The nurse`s most
appropriate response would be
#1Rational.68= The nurse`s first priority is to give
the patient space and protect herself. Moving in on
Move away from him and find an area of safety, or the patient or showing resistance may agitate him
more. She should not move too close to the patient
put a piece of furniture between herself and the
until there is sufficient manpower to swiftly and
patient while yelling for assistance.
effectively handle the situation.
= Hold her ground and quietly ask him to give her
the knife while holding out her hand.
= "You appearCollecting
angry. Perhaps
it would
help
talk
Question.82=
physical
data on
an to
infant
about
it."
at the well-baby clinic, the nurse will observe
= which
"Knives
allowedsigns
on the
I want
you
ofare
thenot
following
in unit.
an infant
with
to congenital
give it to me
#4 Rational.82= Abduction is limited in the
hipimmediately!"
dysplasia?
affected leg. Assessment will also reveal
asymmetrical gluteal folds, an absent femoral
Limited adduction of the affected leg.
pulse when the affected leg is abducted, and a hip
"click" on abduction.
= Symmetrical gluteal folds.
= Femoral pulse when the hip is flexed and the leg
Question.83= A male
patient is unable to sleep. He
is abducted.
is pacing
the floor,
head of
down,
and wringing
Limited
abduction
the affected
leg. his
hands. The nurse recognizes that he is anxious.
What is the most appropriate intervention?
#1 Rational.83= This is a more comprehensive
answer and includes the others. Sleeping
medication should be avoided if at all possible or
unless absolutely necessary, because it only
Let him know you are interested and willing to
temporarily helps suppress the patient`s feelings.
listen.
= Explore with him the alternatives to pacing the
floor.
= Give him his PRN sleeping medication.
Question.84=
A him
pregnant
patient
advised
= Encourage
to talk
abouthas
his been
behavior.
to increase her iron intake. The nurse will know
#1 Rational.84= It is most important to take iron
she understands the dietary teaching if she says
with vitamin C, such as orange juice, because this
that she should take the iron supplement
vitamin helps the body assimilate the iron. Iron
should be taken on an empty stomach to increase
With vitamin C or juice.
assimilation. Eggs or milk products interfere with
=With milk.
assimilation.
= With her other prenatal supplements.
=At mealtime.
Question.85= In a 2 year old with increased
intracranial pressure, which one of the following
signs would be cause for alarm?
Diminishing sunset sign.=
Absence of nystagmus.
= Equal pupils that react to light.
=Increasing lethargy and drowsiness.

Rational.85=Increasing lethargy and drowsiness,


although not specific in themselves, are signs that
would lead the nurse to investigate further since
level of consciousness is an excellent indicator of
increased intracranial pressure. The other
answers-diminishing sunset signs, equal pupils,
and absence of nystagmus-are all normal signs.

Question.86= A patient`s chart states he has a


stage III gluteal pressure ulcer. When assessing the
patient, the nurse would expect to find a(n)
Reddened, nonblanching area of intact skin.
=Area of abraded skin.
= Open wound with loss of subcutaneous tissue.
=Deep crater with visible muscle tissue.
Question.87= When assessing the following
patient care situations, which one would require
the nurse to wear goggles?
Changing a dressing.
Administering an IM injection.
= Catheterizing the urinary bladder.
=Emptying a Foley drainage collection bag.
Question.88= The nurse is taking the history of a
patient who is scheduled for diagnostic tests,
including an MRI. Which reported condition is of
greatest importance to know about when the nurse
is preparing the patient for an MRI?

#3Rational.86= Standardized pressure ulcer


staging systems progress from stage I (red
nonblanchable, intact skin) to stage IV (deep crater
with visible underlying structures such as bone,
muscle, tendons).

#4Rational.87=Goggles should be worn when


there is a risk of contact with body fluids through
spattering or splashing.

#3
Angina.
=Iodine allergy.
= Claustrophobia.
=Hypertension.
Question.91= Which intervention is most useful
when communicating with an aphasic patient?

#4Rational.91= Aphasia leads to feelings of


frustration and discouragement. Patients require
frequent praise for their efforts and encouragement
to continue to try to communicate. Aphasic
Use correct medical terminology when teaching or
patients can best provide information by
explaining.
responding to closed questions that require
= Ask open questions to obtain information.
answers such as Yes or No. If the patient does not
= Repeat the same word until the patient
understand a word, use a different word rather than
understands.
repeating. Use clear, simple, consistent language
= Provide frequent praise and encouragement.
to teach or explain avoid complex medical
terminology.
Question.92= Which medication would the nurse
#2 Rational.92= After a head injury, patients
expect to be ordered for an alert patient who is
should not receive medication that might cause
experiencing severe headache after a head injury
drowsiness or depress the central nervous system
(such as morphine or ibuprofen) or that may
Morphine.
interfere with blood clotting (such as aspirin or
=Acetaminophen (Tylenol).
ibuprofen). Acetaminophen does not have these
= Ibuprofen (Motrin).
undesirable effects.
=Aspirin.

Question.94= A patient is NPO and is receiving


continuous nasogastric tube feedings. Select the
correct nursing action related to administering
medication through the patient`s nasogastric tube.

Flush the tube with water after each medication.


= Stop the tube feeding for 30 minutes before and
after medication instillation.
= Add the medications to the tube feeding
solution.
Question.95=
The
nurse
is assigned
patientinstill
who
= Mix the medications together
and aslowly
is potentiallyas
suicidal.
following nursing
a bolusOf
bythe
gravity.
objectives, which one is the most important?

Involve the patient in activities with others to


mobilize him.
= Recognize a continued desire to commit suicide.
= Observe the patient closely at all times.
= Provide a safe environment to protect the
patient.
Question.96= The major advantage of monitoring
the fetal heart rate electronically is that this
method
akes less nursing time to use.
=Records the actual heart beat.
= Determines beat-to-beat variabilities.
=Causes less discomfort to the mother.
Question.97= A male patient has just had a
cataract operation without a lens implant. In doing
discharge teaching, the nurse will instruct the
patient`s wife to

Keep the eye dressing on for 1 week.


= Allow him to walk upstairs only with assistance.
= Feed
him soft foods
for several
days
to prevent
Question.98=
The nurse
has just
inserted
a
movement.
nasogastric tubefacial
for drainage
into a young male
=patient
Have her
remain
in bed for
days.
and husband
must check
placement.
The3 most
accurate method is to

Insert air into the tubing and with a stethoscope,


listen for a "whish" sound.
= Aspirate the stomach contents and test with
litmus paper.
= Place the tip of the nasogastric tube in a glass of
water and observe for bubbling.
= Send the patient to x-ray for an abdominal film
as ordered.

#1 Rational.94= Each medication should be


instilled separately and followed by 10 mL of
water to prevent obstruction of the tube.
Medications are not added to the feeding solution
because of possible incompatibilities, difficulty
maintaining the medication schedule, and delay in
medication absorption because of the slow flow of
continuous tube feedings. Continuous tube
feedings are interrupted only while the medication
is being administered.

#4Rational.95= Because it is unrealistic to observe


a patient every minute, the environment must be
kept safe for patient protection. Answer (2) is
important, but not the most important.

#3Rational.96= A continuous record of the fetal


heart rate and its relationship to uterine
contractions is done through fetal monitoring to
show beat-to-beat variability. Fetal monitoring can
be done by both internal and external devices.

#2Rational.97= Without a lens, the eye cannot


accommodate. It is difficult to judge distance and
climb stairs when the eyes cannot accommodate.

#2Rational.98= Checking the aspirate with litmus


paper indicates tube placement. An acidic response
means the tube is in the stomach. Many instructors
continue to teach the air insertion technique
however, this is NOT the currently accepted
method. An x-ray check will be used if tube
feedings are to be administered.

Question.99= A 23-year-old female patient was in


an automobile accident and is now a paraplegic.
The orders are for an intermittent urinary
catheterization program and diet as tolerated. A
priority assessment should be to observe for
#4
Urinary retention.
Question.101=
A female patient
has been in the
Answer2.99=Bowel
evacuation.
second stage
of renal failure
for several
Answer3.99=
Weight
gain. months.
Her condition
has
deteriorated
to
uremia, and she
Answer4.99=Bladder distention.
is now being admitted to the hospital. Checking
her lab values, the nurse will expect to see

BUN 180, Creatinine 18, Urine specific gravity


1.010.
Answer2.101= BUN 35, Creatinine 16, Urine
Question.102=
A patient
a radioactive
specific
gravitywith
1.035.
moves
in bed 12,
and Urine
the implant
becomes
=implant
BUN 10,
Creatinine
specific
gravity
dislodged. The priority
nursing
intervention
is to
1.020.
= BUN 100, , Urine specific gravity 1.005.
Put on a lead apron and gloves and place the
implant in a covered container.
= Wait until a radiation team can be called to
recover the implant.
=Question.103=
Leave the roomAas
quickly asfemale
possible
and report
54-year-old
patient
is 3
to
the
head
nurse.
days postoperative following abdominal surgery.
=The
Uselab
long-handled
tongsthat
andher
place
theblood
implant
results indicate
white
cellin
a
lead
box
kept
in
the
patient`s
room.
count is 8,000/cu mm. After analyzing this lab
report, the nursing action is to

equest that the lab repeat the test, as results don`t


make sense.
= Contact her physician so he will be aware of the
high abnormal count.
= Do nothing because this is a normal count.
When physician
evaluatingso
a patient`s
=Question.108=
Contact the patient`s
he will be
understanding
of
a
low
potassium
diet,
the nurse
aware of the abnormal low count
knows that he understands if he says that he will
avoid

Pasta.
=Dry cereal.
=French bread.
=Raw apples.

#1Rational.101= While many factors affect the


BUN, creatinine is a very specific indicator of
renal function. Both the BUN and creatinine are
elevated in renal failure, frequently with a 10:1 to
20:1 ratio. Specific gravity that stabilizes at 1.010
indicates the kidney has lost ability to concentrate
or dilute urine.

#4

Rational.103= This is a normal cell count (the


range is 4500 to 11,000/cu mm) so the nurse
would do nothing except note that it is normal.

Rational.108=Raw apples are high in potassium,


while white-enriched and French bread, dry cereal,
and pasta are foods low in potassium.

Question.109= Irrigating a nasogastric tube should


be carried out using which one of the following
protocols?

Instill 30 mL sterile saline, forcefully if necessary,


and allow fluid to flow into basin for return.
= Gently instill 20 mL normal saline and then
allow fluid to flow into basin for return.
Gently instill 20 mL normal saline and then
withdraw solution.
= Instill 30 mL sterile water and then withdraw
Question.110= A patient
is scheduled for a voiding
solution.
cystogram. Several hours before the test,
physician`s orders will instruct the nurse to

Medicate with urinary antiseptics.


=Force fluids.
= Maintain NPO =Administer bowel preparation.

#2Rational.109= Gentle pressure is necessary


when irrigating a nasogastric tube to prevent
damage to the stomach wall. Normal saline is
recommended to prevent electrolyte loss through
gastric drainage.

#2Rational.110= Forcing fluids ensures a


continuous flow of urine to provide adequate urine
output for specimen collection and, also, to
prevent multiplication of bacteria that may be
introduced during the procedure.

Question.111= Following a cystoscopy, the nurse


will evaluate the patient`s condition for the
complication of
Burning on urination.
=Difficulty voiding.
Development of cold chills.
=Pink-tinged urine.
Question.114= Collecting growth and
development data on the social development of a
1-month-old male infant, the nurse knows that it
should include

Responding to "No, no."


= Turning his head to a familiar noise.
= Actively following movements of familiar
persons with his eyes.
= Discriminating between family and strangers.
Question.118= Following dialysis of a patient, the
nurse expects to find an improvement in the blood
values of
Hypokalemia.
=Low hemoglobin.
= High serum creatinine levels.
=Hypocalcemia.

3 Rational.111= Cold chills could indicate the


spread of infection throughout the urinary tract. `

#3Rational.114= Actively following movements


would occur at 1 month. Responding to "No" and
turning the head in response to noise begins at 4
months, and discrimination between family and
strangers appears at 5 months of age. This is
considered intellectual or cognitive development.

#3

Question.119= A 3-month-old infant has


unrepaired Tetralogy of Fallot. Which of the
following signs and symptoms would the infant be
expected to exhibit?
achycardia, hypertension, decreased femoral
pulses.
= Hypotension,
bradycardia,
dyspnea.
Question.120=
An adult
patient with
a tentative
=diagnosis
Circumoral
spells,
feeding
of cyanosis,
antisocial hypoxic
personality
refuses
to
fatigue.
participate in unit activities. When he is on the
= Cyanosis,
tachypnea,
in upper
unit, he makes
fun ofhypertension
the other patients.
extremities.
Considering the patient`s diagnosis and behavior,
which one of the following nursing plans would be
most effective for the nursing staff to follow?

Question.121= A 56-year-old patient admitted to


et the patient
know the
rules on
the unit. to
the hospital
with angina
pectoris
is scheduled
Answer2.120=
the patientWhile
to isolate
himself
have a cardiac Allow
catheterization.
filling
out a
so for
thatthe
he following
does not upset
other patients.
menu
day, the patient
says to the
Answer3.120=
Confer
with
the
patient,
his In
nurse, "I always get a rash when I eat shellfish."
psychiatrist
the staff
about
patient`s
of
reading the and
patient`s
chart,
the the
nurse
sees nolack
such
participation
on
the
unit.
notation. Following safety protocol, the initial
= Require
the patient
to participate
in activities.
nursing
intervention
should be
to

#3Rational.119= Cyanosis, hypoxia, and feeding


difficulties are classic manifestations of cyanotic
heart disease.

3 Rational.120=In dealing with manipulative


behavior, it is important that all members of the
team as well as the patient are clear about
expectations. (1) pits the nurse against the patient
(2) is incorrect because it is not therapeutic to
isolate a patient and (4) is nontherapeutic because
the patient is not involved in the treatment.

#1
sk the nurse in charge to communicate the
patient`s
reaction to
the physician.
Question.122=
A shellfish
few hourstoafter
a heart
Answer2.121=
Ask
the
patient
if
there
are any
catheterization, the nurse notes that the patient`s
other
foods that
cause suchfrom
a reaction.
blood
pressure
has decreased
136/72 to
Answer3.121=
the dietitian
the patient`s
110/64
mmHg. Notify
The patient
has beenofvoiding
large
reaction
and
request
a
"no
shellfish"
diet.
amounts of urine. The nurse should recognize that
Answer4.121=
Place achange
note onisthe
chart regarding
this blood pressure
probably
due to
the patient`s reaction to shellfish.
The patient going into cardiac failure.
Answer2.122= A reaction to the dye used during
procedure.
Question.123= Athe
patient
is admitted to the labor
Answer3.122=
Insufficient that
fluidare
intake
for4 the
room with contractions
about
to 5past
12
hours.
minutes apart and 30 seconds long. The couple
Answer4.122=
Lost fluids
becauseclasses.
of the diuretic
have attended Lamaze
childbirth
During
effectshould
of the support
dye.
transition, the nurse
a breathing
pattern that is

Slow, deep abdominal pattern.


Answer2.123= Shallow chest breathing at 20 to 30
times a minute.
Answer3.123= Shallow chest breathing at 40 to 60
times a minute.
Answer4.123= Slow, deep chest breathing.

#4Rational.122= The dye used during the cardiac


catheterization is hyperosmolar and, therefore, acts
as a diuretic. As fluid volume drops, the arterial
blood pressure drops.

Rational.123=Shallow chest breathing, slightly


faster than a normal rate, is the pattern used most
in transition. Slow breathing, while effective for
some women during transition, is usually most
effective in earlier labor. Supporting a more rapid
rate of 40 to 60 times a minute is not
physiologically sound and could lead to
hyperventilation.

Answer1.125= Inject air and withdraw NPH, then


withdraw
regular
insulin
the same
syringe.
A 21-year-old
patient
is into
described
as bizarre,
withdrawn, talking and laughing to herself. Orders
include chlorpromazine (Thorazine) 200 mg, PO
Rational.125=Regular insulin is withdrawn into
Answer2.125=
Inject
bottle, then q
BID.
If refused, 100
mgair
IMinto
mayNPH
be administered
the syringe after injecting the prescribed amount of
injectPRN
air and
insulin,
then
4 hours
forwithdraw
agitation. regular
Benztropine
mesylate,
air into the NPH bottle. The last step is to
withdraw
appropriate
amount
of
NPH
into
the
MSD (Cogentin) 5 mg BID is also prescribed.
withdraw the NPH insulin into the same syringe.
syringe.
Assessing thesame
patient,
the nurse finds her
screaming and banging her hands on the wall. She
refuses oral medication. The best nursing action
Answer3.125= Withdraw
would NPH
be to and regular insulin
into separate syringes and give two injections.
Answer1.126= Stay with the patient and get
assistance to give her the IM injection of
Answer4.125= Inject
air and withdraw regular
Thorazine.
insulin, then inject air and withdraw appropriate
amount of NPH into the same syringe
Question.127= The nurse finds a patient on the
psychiatric unitPut
sitting
quietly,instaring
out until
the
Answer2.126=
the patient
restraints
window,
and down,
laughing
The nurse
she calms
theninappropriately.
administer Thorazine.
should recognize that the patient is probably

Getting
ready
to "act
out."to administer
Answer3.126=
Assign
another
nurse
both Thorazine and Cogentin.
Question.128= In the last few months, a
Answer2.127=Laughing
insteadthat
of crying
56-year-old patient has noticed
he hasabout
been
the
"slowing
up" in allTell
of situation.
his
andtothat
Answer4.126=
the movements
patient firmly
stophis
speech
hasher
become
hitting
hands indistinct.
or she willThe
hurtphysician
hersel
Answer3.127=
Hallucinating.
makes a diagnosis
of Parkinson`s
disease. When
teaching the patient about Parkinson`s disease, the
a funnythe
scene.
nurseAnswer4.127=Seeing
should include encouraging
patient to

#1
Rational.126=The patient`s behavior indicates that
she needs Thorazine. To decrease the amount of
external stimuli, the nurse must provide limits
because the patient cannot staying with her will be
reassuring.

3Rational.127= Inappropriate laughter in psychotic


behavior is generally caused by visual or auditory
hallucinations.

1Rational.128=It is very important to keep patients


with Parkinson`s disease actively involved in work
and social situations. They experience altered body
images and tend to withdraw from society. When
aintain employment as long as possible.
this happens, the complications of immobility
increase. As complications increase, they tend to
Answer2.128=Retire and take part in limited social
decrease patients` life spans.
Question.129= Aactivities.
28-year-old primipara is
admitted to the hospital in early labor. She is
comfortable
andImmediately
visiting withdecrease
her husband
when
Answer3.128=
his work
her water breaks. Which
of
the
following
nursing
schedule.
assessments should be completed first?
3Rational.129=When the water breaks, there is a
Answer4.128=Work only when he feels up to it.
possibility of prolapsed cord, which must be ruled
out immediately. This is the first nursing
Monitor the fetal heart rate.
assessment. Following this intervention, the nurse
would check the fetal heart rate.
Answer2.129=Attach the external fetal monitor.
Answer3.129= Check for prolapsed cord.
Answer4.129=Note color and quantity of fluid.

Question.135= A postsurgical patient develops


bacterial pneumonia. When assessing her
condition, which of the following signs or
symptoms would the nurse expect to be present

Irregular pulse.
Question.136= Before a patient who has gout is
Answer2.135=Hypertension.
discharged
from the hospital, it is important to
evaluate his knowledge of dietary management.
Which one of the following diet choices would
Answer3.135=
Temperature
above 37.7 his
degrees
C
indicate
to the nurse
that he understands
dietary
(100
degrees F).
restrictions?

Rational.135= Bacterial pneumonia causes an


increase in temperature, so the nurse would know
that it may be present when the patient`s
temperature is above 37.7 degrees C (100 degrees
F). This type of pneumonia would not result in
hypertension, irregular pulse or even shallow,
irregular respirations. The primary symptoms, in
addition to a high temperature, are rapid onset,
constant cough, pleuritic pain, anxiety and
dyspnea.

Answer4.135=Shallow, irregular respirations.


Crab cakes, rice and peas.
Question.137= A 3-year-old child is brought to an
emergency department by his mother because she
Answer2.136=Antipasto
salad,
rice and asparagus.
could not get him to wake
up completely
after his
nap. He is semiconscious and has a low-grade
fever. The physician suspects lead poisoning. The
Answer3.136=
Liver,
potato
nurse
should expect
that athe
childand
willspinach.
be treated
with

Rational.136= Steak is the best choice because


foods highest in purine include shellfish, liver,
chicken, beans, and various vegetables. The
appropriate diet will include high carbohydrates
with calorie control.

Answer4.136=Steak, baked potato and green


salad.
Calcium disodium edetate (EDTA).

Rational.137=Calcium disodium edetate (EDTA)


is a chelating agent which promotes the excretion
of lead from the body. It is given IV or deep IM
for children who cannot tolerate IV.

Question.138=
The nurse should
know that if a
Answer2.137=Syrup
of ipecac.
patient with heart failure develops a ventricular
arrhythmia,
physician`sActivated
orders will be to
Answer3.137=
administer
charcoal.

Answer4.137=Erythromycin.
Morphine
sulfate,primigravida
USP.
Question.142=
A 26-year-old
who is
hydrochloride,
USP.
27Answer2.138=Lidocaine
weeks pregnant is admitted
to the hospital
with
pre-eclampsia. The physician prescribes
Answer3.138=
Digoxin
magnesium
sulfate therapy.
The(Lanoxin).
nurse will place
the patient in a private room because
Answer4.138=Propranolol
hydrochloride
A quiet, darkened
room is(Inderal).
important to reduce
external stimuli.

Answer2.142= A rigid regimen is an important


aspect of eclamptic care.
Answer3.142= It will allow her husband to be with
her.
Answer4.142= She would be disturbed if she were

Rational.138= Lidocaine hydrochloride, USP, is


the medication of choice because it depresses
ventricular irritability. Inderal is contraindicated,
as it is a beta-adrenergic blocking agent--it also
depresses cardiac function. For a patient who
already has a compromised cardiac status, this
could be fatal. Morphine sulfate, USP, reduces
anxiety, but will not prevent arrhythmias. Digoxin
is used to strengthen ventricular contraction.

#1Rational.142= An important aspect of the


treatment for pre-eclampsia is absolute quiet, and
only a private room will accomplish this objective.

work-up for peptic ulcer disease. He is scheduled


for an upper GI series at 8:00 AM. The most
important nursing intervention related to his
intermediate-acting insulin dose will be to

Administer his usual dose of intermediate-acting


insulin before the test.

Answer2.143= Explain that he will not receive his


insulin until after the test is completed.
Question.144= When completing an assessment on
Answer3.143=
Administer
extrathat
insulin
to
a 9-year-old
child, the
nurse knows
a primary
withstand
the
stress
of
the
test.
developmental task of a 9 year old, according to
Erikson, is
Answer4.143= Administer regular insulin before
the test and explain that his breakfast will be
served
after
Trust.
Question.145= The
nurse
has the
a full work load and
Answer2.144=Initiative.
must reassign
some of her patients to the nursing
assistant. The most appropriate patient to reassign
Answer3.144=
is a(n) Industry.

Rational.143=The patient is taking


intermediate-acting insulin, as he is a moderate
diabetic. He will most likely receive insulin and
then be fed breakfast when he returns after the test.
Therefore, (1) is the most correct. Extra insulin
would not be administered.

Rational.144=From age 6 to 12, Erikson has


theorized that the main developmental task is
industry versus inferiority. Trust (1) is resolved in
the infant stage of development initiative (2)
versus guilt is the task of preschoolers, age 3 to 6,
and identity (4) is the task to be resolved at
puberty and adolescence.

Rational.145=The most appropriate patient would


be the one with the CVA diagnosis. This patient
would have been hospitalized for 2 days, so the
initial assessment is completed. This condition
does not demand immediate assessment or
intervention, as does the colostomy patient
Answer2.145= CVA patient who has been
(assessing for hemorrhage, vital signs, etc.), the
hospitalized for 2 days.
oncology patient (determining the effectiveness of
the pain protocol), and the new patient with
suspected
pancreatitis (who needs a complete
Question.146= When assessing a 2-month-old
Answer3.145=
Newly
patient withthat
assessment).
infant`s
reflexes, the
nurseadmitted
should understand
4 Rational.146=The palmar grasp, elicited by
suspected
pancreatitis.
at this age the infant should exhibit
placing a finger in the infant`s palm, disappears at
Question.147=
57-year-old
patient
Answer4.145=AOncology
patient
whoisisadmitted
in severeto about 4 months of age, so it would still be present
while assessing the 2 month old. A positive
the nursing
unit forBabinski`s
an
work-up. The
pain controlled
byendocrine
epiduralreflex.
analgesia.
negative
Babinski`s
reflex is present until 12 to 18 months
tentative diagnosis is Cushing`s syndrome,
of age. In adults, a positive Babinski`s reflex is
etiology
unknown. Theabsent
patienttonic
saysneck
to the
nurse, "I
Answer2.146=An
reflex.
look so awful! I just can`t stand having my friends indicative of disruption of the pyramidal tract. The
tonic neck reflex disappears at 4 months, as does
see
me like this!"
the following
Answer3.146=
AnWhich
absentofrooting
reflex.
the rooting reflex while the infant is awake.
responses would be best?
Answer4.144=Identity.
Patient just returning from the recovery room
following colostomy surgery.

Answer4.146=A palmar grasp.


I know how you feel, but they will get used to it
very soon."

Answer2.147= "The way you look is a result of


your disease. When it is controlled, you will return
to your normal appearance."

Answer3.147= "It is best not to worry because the


stress will make your symptoms worse."

Rational.147=The signs and symptoms of


Cushing`s syndrome are all caused by an excessive
steroid production. When this condition is
normalized, these signs will disappear. This is a
factual, therefore reassuring, statement.

understands that serum laboratory changes are


associated with radiation therapy. The nurse will
focus the patient assessment on

Answer1.148= Checking the need for


subcutaneous injections of vitamin K.
Question.149=
AfterChecking
1 month lab
of hospitalization,
Answer2.148=
tests for low a
schizophrenic patient
states
that
voices
are telling
hemoglobin.
him he will die tonight, and he asks the nurse if
this is true. This question should indicate to the
that thechanges
patient in electrolytes.
Answer3.148= nurse
Monitoring

Rational.148= Patients undergoing radiotherapy


may have a decreased white blood cell count and
should be observed closely for infections.

3
Rational.149=The patient exhibits the awareness
not improving.
Answer4.148= Is
Observing
for signs of infection.
to question his symptom, thus he is improving.
The nurse can reinforce this strength, then stay
Question.150= An adult patient with a gunshot
with him and help him discuss his fears. (4) is
Answer2.149=
to enter
the emergency
manic phase
wound to the Will
chestbegin
is brought
to the
incorrect as his prognosis cannot be calculated. (1)
of his illness.
department. A left-sided
pneumothorax occurred,
and (2) are not substantiated by the data.
and the physician inserted two chest tubes, one
into the right anterior lobe and one into the right
Answer3.149=
Is improving
needs system
reassurance
lower lobe. The
water-sealbut
drainage
is
from thesuction.
nurse. To assist in chest
connected to a walled-in
drainage, the nurse should place the patient in
Rational.150=Positioning a patient on the left side
Answer4.149= Has a poor prognosis.
will assist in drainage and semi-Fowler`s will
assist in breathing. The patient can usually be
igh Fowler`s position.
turned to both sides and the back.
Question.151=
A patient turns position,
abruptly in
and
Answer2.150=Semi-Fowler`s
onbed
his left
dislodges his chestside.
tube. The nurse`s first
intervention is to
Answer3.150= Supine position.
mmediately get a sterile dressing and apply over
the opening.
Answer4.150=Low-Fowler`s
position, on his right
side.
Answer2.151= Obtain a sterile towel and place it
over the opening.
Question.152=Place
An 11-year-old
patientgauze
with the
Answer3.151=
a sterile Vaseline
over
diagnosis of acute rheumatic
fever
will
have
a
care
the opening.
plan that includes the most important nursing
measure
ofopening with the
Answer4.151= Seal
off the
nurse`s hand.
Sufficient vitamins for tissue repair.
Answer2.152=Adherence to bedrest regimen.
Answer3.152= Breathing exercises to increase
oxygen exchange.
Answer4.152=Isolation for prevention of
infection.
C

Rational.151=The nurse must seal off the opening


immediately therefore, a hand is the closest thing
available and should be placed over the opening. If
air is sucked back into the chest cavity, the lung
could collapse again.

Rational.152= The most important nursing care


would be to maintain bedrest so that the heart is
not overtaxed and the body has sufficient rest time
to repair the damage caused by the systemic
inflammatory disease. Warm soaks are given to
reduce joint pain, and a good diet is necessary for
repair of any organ, but in this instance they are
not the most important components of nursing
care. Normally, isolation is not required.

Question.171= A newborn passes meconium after


the first day and the mother is concerned. The
nurse can explain to the new mother that infant
meconium is normally

Rational.171= Meconium is normally passed


during the first 24 hours for all babies. It is not a
transitional stool (3) which is partly meconium and
partly milk, usually passed from the second to the
fifth day.

Question.172= If there is a physician`s order to


irrigate the bladder, which one of the following
nursing measures will ensure patency?

Answer1.172= Irrigate with 20 mL of normal


saline to establish patency.

Question.174= The most important nursing


intervention for patients on IPPB therapy is to

Rational.174= The detection of alterations in vital


signs could be an indication of nebulizer
medication side-effects so this intervention is the
most important.

Question.182= A 29 year old is in the labor room


Rational.182= At this stage, the most important
and has been there for 6 hours. It is the change of
action is to support the baby`s head. The nurse
shift and, when the nurse is receiving report, a
would not try to hold back or stop the delivery, but
nurse`s aide rushes in and says that the patient is
allow it to occur naturally. If possible, send
Question.183=
Following
emergency
delivery,
delivering. As
the nursean
enters
the room,
the
someone else to notify the physician. Instruct the
the
nurse
is alone
with theThe
mother
theaction
baby
baby`s
head
is crowning.
first when
nursing
mother to pant, not push, to avoid rapid delivery.
is finally born. After suctioning
the baby`s mouth
is to
with a bulb syringe, the nurse realizes the newborn
has not yet begun to breathe. The first nursing
action is to

nitiate infant CPR.

Answer2.183= Hold the baby upside down by his


heels so that mucus will drain out.
Answer3.183= Gently rub the baby`s back or soles
of his feet.
Answer4.183= Sharply slap the baby`s back to

#2 Rational.183= The first action is to hold the


baby so that the head is in a dependent position.
This will allow mucus and fluid to drain out. Then,
if the baby does not breathe on his own, action (3)
should be done. The nurse would NOT slap the
baby on its back.

the psychiatric unit. She isolates herself in a part of


the day room. The staff nurse wishes to establish
communication with her. The best plan would be
to approach the patient

Introduce yourself and ask if you can stay with


her.

Answer2.184= Nonverbally because she is


nonverbal.
Answer3.184= Introduce yourself and tell her that
you are going to sit with her a few moments.
Question.185= A 25-year-old female with AB
negative blood
para 0,
1. She hasand
Answer4.184=
Useistouch
to gravida
get her attention,
aborted
after
10
weeks
and
is
going
be you.
kept in
ask her if she would like to work to
with
the hospital 1 night for observation. Considering
her Rh blood type, what action would the nurse
expect to be performed

3Rational.184= The most therapeutic response


would be to introduce yourself and sit quietly a
few moments making NO demands on her by
asking questions. A nonverbal approach would be
threatening. The nurse would not use touch with a
schizophrenic (potentially paranoid) patient.

85= The patient would be given an injection of


RhoGAM within 72 hours
Rational.185=RhoGAM must be administered
within 72 hours to an Rh-negative female who has
not been sensitized. Because this is her first
pregnancy, this patient should have the RhoGAM
within 72 hours.

Answer3.186= High susceptibility to infection.


Question.186= A 50-year-old patient has received
the diagnosis of agranulocytosis. The nurse
understands that one of the most serious
consequences of agranulocytosis is

Question.187= A psychiatric patient rapidly


improves and is scheduled to be discharged
tomorrow. Which of the following responses
demonstrates that the nurse has a good
understanding of termination of a relationship?

Question.193= A patient is receiving lithium


carbonate for manic behavior. Administration of
this medication should be guided by

Rational.186= This blood disorder is characterized


by profound neutropenia and can lead to severe
infection which could be fatal. The symptoms
would be chills, fever, sore throat, and lethargy.

ou`ve done some good work here. I hope you`re


able to follow through on it."
Rational.187= Termination is an on-going process,
so encouraging the patient to follow through with
what was learned is a good response. The nurse
would NOT encourage the patient to "stop by"
because the relationship is terminated.
Answer3.193= Telling the patient that a lag of 7 to
10 days can be expected between the initiation of
lithium therapy and the control of manic
symptoms.
Rational.193= There will be 7 to 10 days before
the patient will experience a decrease in the manic
symptoms. A therapeutic dose is 300 mg TID
regular blood studies must be continued
throughout drug therapy muscle weakness is an
expected side effect and does not indicate toxicity.

Question.194= A 24-year-old male patient with no


feeling or sensation in his lower extremities is in
spinal shock. The nurse will be able to recognize
that his condition is improving when

Answer3.194= Hyperreflexia occurs.


Rational.194= Reflex activity begins to return
below the level of injury because of automatic
activity inherent in nervous tissue.

Answer1.195= High calorie, high protein, low fat.

Question.195= The type of diet that would be


indicated for a child with cystic fibrosis is

Rational.195=High calorie and protein intake will


support growth and weight gain. A low fat diet is
encouraged because the fat is poorly digested.
Pancreatic enzymes are replaced and given with
meals to assist with the digestion and absorption of
fat.

1cm above the level of the ischial spines.


Question.198= After a vaginal examination, the
patient`s obstetrician tells her that she is at -1
station. When she asks the nurse what this means,
the nurse tells her that station -1 means the
presenting part of the fetus is

Rational.198= Station is the degree to which the


presenting part has descended into the pelvis-the
relationship between the presenting part and the
ischial spine. The fetus moves from above to
below the level of the ischial spines.

Left or right occiput anterior.


Question.199= In assessing the baby`s position for
delivery, the nurse knows that the most favorable
position for delivery of an infant is a

Rational.199= Left or right occiput anterior (LOA


and ROA) are the most favorable positions for
delivery. Breech, where the buttocks or lower
extremities are the presenting part, is a difficult
delivery position and a transverse lie necessitates
delivery by C-section.
Answer4.204= Ankle and calf circumference and
leg length after the patient has been lying down.

Question.204= When measuring a patient for


elastic hose, the nurse will measure the

Rational.204= The leg length and ankle and calf


circumferences should be measured after the
patient has been lying down. This causes the
peripheral edema to be minimal and ensures that
the hose fit snugly to offer maximum support.

Fluid retention, mood changes, anxiety.


Question.207= A patient has physician`s orders to
take chemotherapeutic hormonal agents. The nurse
would prepare the patient to anticipate which of
the following side effects

Question.209= The physician has just completed a


liver biopsy. Immediately following the procedure,
the nurse will position the patient

Question.210= The nurse collects the following


information when taking a nursing history from a
postmenopausal patient. Analyzing the data
reveals a risk factor for the development of
osteoporosis when the patient

Rational.207= Fluid retention, mood changes, and


anxiety are common side effects. They are a direct
result of using androgens or estrogens as
chemotherapeutic agents.

Rational.209=Placing the patient on his right side


will allow pressure to be placed on the puncture
site, thus promoting hemostasis and preventing
hemorrhage. The other positions will not be
effective in achieving these goals.

as been on prednisone (Deltasone) for 3 months.


Rational.210= Glucocorticoids, such as
prednisone, promote protein catabolism and are a
known risk factor for the development of
osteoporosis.

Question.212= The nurse is at the bedside when a


9 year old has a seizure shortly after admission.
The first action during the seizure is to

Answer3.212= Protect the child from injury by


removing objects from the bed.

Question.214= While explaining the side effects of


oral contraceptives to a teenage patient, the nurse
will tell her that

Answer4.214= Side effects may appear but these


usually disappear within the first to third cycle.

Question.216= A 65-year-old male patient with


Parkinson`s disease is being treated with L-Dopa.
The nurse will know he understands the teaching
principles when he says that he avoids foods rich
in

Rational.216= Foods rich in B6 block the desired


effects of L-Dopa therefore, they need to be
omitted from the diet. Examples of foods to be
avoided include meat, especially organ meats,
whole-grain cereals, peanuts, and wheat germ.

Question.220= A 20-year-old patient is admitted to


a hospital with a diagnosis of acute schizophrenia.
He is becoming more withdrawn and suspicious of
other patients, and he constantly tries to argue with
the nursing staff that several of the patients are
"out to get him." The best nursing approach to this
behavior is to

Answer3.220= Avoid disagreeing with him and


get him involved with an activity.

Question.222= A 50-year-old female patient is


scheduled for a cholecystectomy. Following
surgery, she has a T-tube in place and is returned
to the surgical unit. Which position will ensure
optimal functioning

Rational.222=Initially a low-Fowler`s position,


then a semi-Fowler`s position is encouraged, but
not high-Fowler`s. The objective is to facilitate
drainage, as well as allow a position of comfort for
the patient.

Question.223= A patient is admitted with a


tentative diagnosis of metabolic acidosis.
Assessing for the clinical manifestations most
representative of this condition would include

headache, diarrhea, =Metabolic acidosis results


from conditions such as diabetes mellitus and
diarrhea

Question.230= A female patient has orders to take


lithium regularly after being discharged from the
hospital. Her serum level has been regulated at 1.4
mEq/L and she seems to be doing well. Which of
the following is the most important discharge
information to impart to the patient and her family

Answer2.230= Drowsiness, tremors and slurred


speech are early indications of lithium toxicity.

Rational.230= Most important is that the patient


and family be aware of lithium toxicity these
symptoms are indicators of this condition. Answer

Answer1.236= Administering pilocarpine eye


drops.
Question.236= The preoperative nursing care plan
for a patient scheduled for an iridectomy will
include

Rational.236=Pilocarpine acts directly on the


myoneural junction it constricts the pupils and
forces the iris away from the trabecular, allowing
fluid to escape. Cycloplegic drops are given
preoperatively with a cataract to paralyze the
ciliary muscle, and postoperatively to relax the
ciliary musclethe
with
an iridectomy.
Answer2.237=Turn
patient
onto her left side.

Question.237= A patient is lying comfortably on


her back 30 minutes after receiving an epidural
injection. The nurse checks fetal heart tones and
the rate is 100 per minute. The first nursing action
is to

Rational.237= This form of regional anesthesia


may cause transitory fetal bradycardia. The first
nursing action is to turn the patient onto her left
side, shifting the weight of the fetus off the inferior
vena cava. If the condition does not change after
repositioning, then administer oxygen, as ordered.
The next two actions would be to check the fetal
heart rate and the mother`s blood pressure.
Rational.239=The appropriate technique is to pull
the ear auricle down and back so that the ear drops
Question.239= A 2-month-old female infant
fall into the ear canal. The infant may be on her
develops an ear infection. While administering ear
side or back with her head turned for this
drops, the appropriate technique is to place the
procedure. This question asks for technique, so
baby
answer (1) is more appropriate. If the nurse is
administering ear drops to an adult, the procedure
is to pull the ear auricle up and back.

Question.241= One of the primary goals of


treatment for a patient with acute
glomerulonephritis would be to encourage

Rational.241= Bedrest during the acute phase will


protect the kidneys from added stress. Activity
may increase urinary abnormalities as well as
facilitate diuresis. When the kidneys recover, fluid
balance and mild hypertension will be alleviated.
The diet should be low, not high, protein to protect
the kidneys from processing protein waste
products.

Question.247= During a typical emergency


delivery, which of the following principles best
explains why the nurse will not cut the cord

Rational.247= The rationale for not cutting the


cord following an emergency delivery is that lack
of sterility could lead to infection. Hemorrhage
might also occur, but not from the placenta.
Answers (1) and (4) are not relevant principles to
explain why the cord should not be cut.

Question.250= A nursing goal to be included in


the care plan for a patient with chest tubes in place
will be to

Question.251= A patient has recently been


diagnosed with tuberculosis. The wife of the
patient is concerned that she will contract the
disease. The nurse can teach the patient`s wife that
the most important preventive method would be

Question.252= A 2-year-old patient has a fracture


of his right femur. Observing whether Bryant`s
traction is properly assembled, the nurse will
expect to see the

Answer2.250= Keep the bottles below bed level to


prevent backflow.

Rational.251= Those people infected with the


tubercle bacillus without the disease and those at
high risk for developing the disease should receive
a drug regimen of isoniazid and vitamin B6 as
prophylaxis. Measures to prevent the spread of the
disease should also be implemented but,
depending on the relationship, not living with her
husband or keeping him in isolation are not the
usual methods.
Rational.252= Bryant`s traction is a form of skin
traction and, therefore, does not require a pin
insertion. Moleskin is frequently used as the
stabilizing material for traction application. The
weights must hang freely from the crib to maintain
alignment and decrease the fracture.

Answer4.252= Weights attached to skin traction


and hung freely from the crib.
Question.254= A patient with the diagnosis of
schizophrenia tells the nurse that there is nothing
wrong with her-it`s the fault of her family that she
is in the hospital and that they are out to get her.
This defense mechanism is called

Question.255= The nurse observes a schizophrenic


female patient sitting, staring into space,
occasionally saying something like "Is the world
coming to an end?" The nurse`s understanding of a
comment like this is that the patient is

Rational.254= The patient is using projection


when she blames her family, saying they are out to
get her. Projection is placing the blame for one`s
difficulties on others it is an indicator of paranoia.

Answer2.255= Having difficulty telling the


difference between her own wishes and fears and
what is real.

Rational.255= A central problem in schizophrenia


is difficulty in reality-testing, where the patient
often cannot tell the difference between what is
real and what is not. This statement is not
necessarily paranoid ideation, so answer (

Question.259= For a diagnosis of viral pneumonia


in a 6 month old, the nursing diagnosis is fluid
deficit. The best method of ensuring a proper fluid
balance during the acute stage of the infant`s
illness is to administer

Rational.259= One of the dangers of high


temperatures in young children is dehydration. It is
critical to restore fluid balance, so IV infusion is
the preferred treatment. Dextrose and water is a
hypotonic solution that causes cells to expand or
increase in size. It is the fluid replacement of
choice for diarrhea and dehydration.

Question.261= The nursing staff of a psychiatric


unit planned an all-day outing. Many of the
patients were on large doses of phenothiazines. A
nursing action important to implement prior to the
outing is to

Rational.261=Patients taking phenothiazines are


sensitive to the sun (photophobia). The correct
answer is to have these patients put on sunscreen
and wear hats when they are in the sun. Answer

Question.263= A 23-year old female patient,


following an automobile accident, was placed in
skeletal traction for a fractured femur. In the
assessment of this patient, one of the nurse`s first
concerns will be to assess for

Rational.263= Proper alignment is critical to


maintain skeletal traction in counterbalance or
correct pull. It is also important to assess for
pressure points or skin excoriation, but these
observations would be continual as long as the
traction is in place.

Question.268= Dilantin is ordered to control a 10


year old`s seizures. While teaching her mother
about the medication, one of the side effects to
emphasize is

Rational.268= The primary side effects of the


drug, Dilantin, are bleeding gums and gum
hypertrophy, rash and GI symptoms. Answer (3) is
the preferable choice because it will involve
teaching the patient to use a soft toothbrush, brush
frequently, and eat a diet high in vitamins and
minerals to protect the gums.

Question.274= During the postoperative period


following abdominal surgery, the patient will
require airway suctioning. The primary rationale
for keeping the suction catheter in the airway for
only 10 to 15 seconds is to prevent

Rational.274= Suctioning longer than this length


of time will remove excess oxygen and this may
lead to hypoxemia. Another technique of
preventing hypoxemia is to administer oxygen 1 to
2 minutes before suctioning.

Question.276= There is an order to perform the


crede procedure on a male patient every 6 hours.
"Crede" is a term used for a

Rational.276= Crede is a French term that refers to


the manual expression of urine, made necessary
because of a hypotonic bladder. This method will
help initiate bladder retraining.

Question.277= A patient has a chest tube


connected to two-bottle water-seal suction. The
purpose of water in the second bottle is to

Rational.277= The end of the drainage tube is kept


under water this water seals the tube so air cannot
enter and be drawn back into the pleural space.

Question.279= Immediate postoperative nursing


interventions for an above-the-knee amputation
will include

Rational.279= The possibility exists that the


patient could hemorrhage from the stump.
Therefore, it is safe nursing care to have a
tourniquet at the bedside.

Question.280= The nurse will help a child with


acute glomerulonephritis follow a diet regimen of

Rational.280= A diet with restricted potassium and


protein is necessary for all children who
demonstrate some degree of renal failure.

Question.282= A child is admitted to the hospital


with marked symptoms of nephrosis. The
following information should be included in the
admission nurse`s notes. Which information is
most pertinent in terms of the child`s condition

Rational.282= It is important to note the degree


and extent of generalized edema that occurs with
nephrosis. The condition is characterized by severe
proteinuria which results in hypoalbuminuria
leading to the shift of fluid from the intravascular
to the extracellular compartment.

Question.288= The instructions to a patient just


learning to use a four-point crutch-walking gait
would be to move the

Answer3.288= Right crutch, then the left foot.

Question.291= A 1-month-old female infant has


been readmitted to the hospital for a cleft lip
repair. In planning for her preoperative care, the
nurse will feed her in a/an

Rational.291= An upright or sitting position is


important when using a soft, large-holed, cross-cut
or special nipple. The nipple should be placed on
the opposite side from the cleft.

Question.294= An adult male patient is admitted


to the hospital for a cardiac work-up. During the
initial physical assessment, the nurse listens for
bronchovesicular breath sounds. These sounds can
normally be heard over the

Rational.294= These hollow, muffled breath


sounds can be heard over the bronchial area below
the clavicle.

Question.296= A male patient has a tracheostomy


and requires suctioning. The nurse knows that
signs of hypoxia may occur during this procedure.
A nursing action to prevent hypoxia is to

Answer3.296= Hyperinflate the lungs with 100%


oxygen prior to and following suctioning.
Rational.296= Hyperinflation of lungs with
oxygen before and after suctioning prevents
potential cardiac complications due to a sudden
drop in blood oxygen levels.

Question.299= A 24-year-old married female


suspects that she is pregnant. She states that her
last menstrual period started on August 9, 2005.
She asks, "If I am pregnant, when will my baby be
due?" According to Nagele`s rule, the due date
would be

Rational.299=Nagele`s rule is to count back 3


months from the first day of the last menstrual
period and add 7 days.
Answer1.299= May 16,2006.

Question.301= A 1 1/2-year-old is brought to the


hospital with a diagnosis of pneumonia. His
temperature is 38.8C (102F), respiratory rate is 40
and he appears lethargic. A tentative diagnosis of
pneumonia is made. Evaluating his condition, an
indication that he is improving is a respiratory rate
of

Rational.301=A normal respiratory rate for a 1 1/2


year old is 30 breaths/min. This rate would
indicate that the child`s respiratory condition is
improving. The usual respiratory rate for a 14 year
old is 18 breaths/min. A respiratory rate of 20 is
usual for a 10 year old. A respiratory rate of 50 is
abnormally high.

Rational.302= All drugs may be expected to cross


the placental barrier and are especially damaging
during the first 8 weeks, when fetal organogenesis
Question.302= A new mother-to-be asks why her
is taking place. Drugs taken later in pregnancy
doctor told her to discontinue taking all
may also affect the fetus, but their effects may not
unnecessary drugs during pregnancy. Based on the
be known for years. Even drugs taken during labor
understanding of the relationship between drugs
may have a depressive effect upon the CNS of the
and the fetus, the nurse`s reply would be
fetus and may take several days to wear off.
Nicotine from smoking may cause low birth
weight infants as well as congenital defects.
Rational.303= The diet must include at least one
fruit or vegetable high in vitamin C, and should
include a total of four fruits and vegetables.
Question.303= Adequate nutrition is essential
Pregnancy requires the addition of 300 calories a
during early pregnancy for optimum fetal
day over regular caloric intake, and 1500 calories a
development. The nurse recommends a daily diet
day would be inadequate. The recommended
that would include
calories for someone aged 28 are 2300 a day. New
research indicates that sodium is essential, so a
low sodium diet is not recommended.

Question.306= A cerebral arteriogram is


performed on the patient. When he returns from
the operating room, the nurse observes that he may
be having a reaction to the dye. The sign or
symptom that suggests this complication is

Rational.306= Numbness of the extremities is a


symptom of delayed reaction to the dye.
Respiratory distress is a frequent early sign of
anaphylactic shock. The release of histamine
causes major vascular and bronchial symptoms
resulting in anaphylaxis.

Question.307= The patient goal for fluid intake


following abdominal surgery for an inguinal hernia
would be

Rational.307= 2000 to 3000 mL/day would be


fluid maintenance postsurgery. The body will
require additional fluids over the minimum due to
fluid loss and the recovery process after surgery.

Question.309= A 26-year-old primigravida is


Rational.309=Edema, proteinuria and hypertension
mildly pre-eclamptic and will be followed on an
are the three cardinal signs of pre-eclampsia.
outpatient basis. Which of the following signs or
Normal urine output or oliguria occurs rather than
symptoms should the nurse expect to observe if the
polyuria.
eclampsia is becoming more severe

Question.311= The RN has discussed the baby`s


newborn status with the mother and asks the
practical nurse to reinforce it. She explains to the
mother that the insulin level in newborns of
diabetic mothers

Rational.311=Insulin levels are increased in these


infants because the mother`s glucose readily
crosses the placenta and stimulates the fetal
pancreas to secrete increased levels of insulin. The
fetal insulin does not cross the placenta.

Question.315= A patient is being prepared for an


oral cholecystogram. Before the dinner meal the
practical nurse instructs the nursing assistant to

Rational.315= Diarrhea is a very common


response to the dye tablets. A dinner of tea and
toast is usually given to the patient. Each dye
tablet is given 5 minutes apart, usually with 1 glass
of water following each tablet. The number of
tablets prescribed will vary, because it is based on
the weight of the patient.

Question.316= A 7-year-old male complains of


pain and limited movement in his left hip. The
physician suspects Legg-Perthes disease. At this
stage of the disease, the major goal of treatment is
aimed at

Rational.316= Legg-Perthes disease affects the


femoral epiphysis in which aseptic necrosis
occurs. Pressure on the necrotic femur can cause
permanent damage.

Answer4.316=Preventing pressure on the head of


Answer3.318= "I understand
the femur. you are concerned.
The staff will see that you are safe."
Question.318= A 34-year-old male patient with a
diagnosis of schizophrenia, paranoid type, barges
into the dayroom yelling, "The President is on my
side. If you bother me, I`ll send him after you!"
The nurse could most effectively respond by
saying

Rational.318= The patient`s grandiose attacking


statements probably reflect his feelings of fear and
his anger at being afraid. Reassurance that he will
be safe is important. His fear should be respected
but not necessarily confronted since this might
increase his anxiety. Confrontation would
probably escalate his aggressiveness and add to his
defensiveness. In his present state, a probing
question would be threatening and inappropriate,
although it might be useful later.

Rational.319= An indication of a comatose state is


Question.319= A male patient was admitted with a
increased blood pressure and slowing respirations.
diagnosis of subdural hematoma and transferred to
Aphasia is also a result of increased pressure
the ICU in a semicomatose state. If the patient
however, due to a decreased level of consciousness
goes into a coma, the nurse would be likely to
which is sometimes present, this is not always an
observe
accurate indicator.

Question.320= A male patient is taken to surgery


for evacuation of a subdural hematoma.
Immediately after the evacuation, the priority
nursing assessment is to observe

Rational.320= All of the nursing interventions


listed would be carried out for the patient however,
the most important one is to prevent cerebral
hypoxia (which contributes to cerebral edema) by
maintaining a patent airway. The acid-base
imbalance and hypoxia are often mistaken for
signs of increased intracranial pressure, leading to
unnecessary surgical intervention. A patent airway
will establish adequate oxygenation and prevent
carbon dioxide build-up.

Question.323= A female patient has sustained


burns of her right arm, right chest, face, and neck.
She has just been admitted to the burn unit. Her
weight on admission is 50 kg. Using the rule of
nines, the estimate of the extent of her burns is

Rational.323=The extent of the burns is 27


percent, calculated by adding the head = 9 percent
(face and neck each equal 4 1/2 percent), arm = 9
percent, and chest = 9 percent.

Question.324= To promote adequate nutrition, a


burn patient`s diet after the first week of
hospitalization should include

Rational.324= A diet high in carbohydrates is


essential to allow the protein to be spared for
tissue regeneration. High protein is also needed for
tissue repair.

Question.325= Completing a general assessment


on a 6 month year old, the nurse knows that this
age infant is able to

Answer1.325= Sit for a short time, look toward


sounds, and begin babbling.

Question.327= Each newborn receives an Apgar


score shortly after birth. The nurse understands
that the purpose of Apgar scoring is to

Rational.327=The purpose of Apgar scoring is to


determine the viability of the infant. Apgar scoring
is the evaluation of five vital signs: heart rate,
respiratory rate, muscle tone, reflex irritability, and
color. Scores of 0, 1 or 2 are given to each vital
sign for a total of 10. A score of 7 to 10 is
considered vigorous.

Question.333= The nurse has completed a


postoperative assessment for hypoparathyroidism
following a thyroidectomy. If the symptoms of this
condition are present, the nurse would check with
the RN and expect to administer

Rational.333=Signs of hypoparathyroidism
following a thyroidectomy are evident in an acute
attack of tetany. The drug of choice is calcium
gluconate to counter the low calcium level.

Question.334= A patient is admitted to the hospital


with an obstruction just proximal to the old
ileostomy stoma. The nurse will monitor for a
major complication which is most likely to be

Rational.334= Due to the extreme loss of fluids


from the high colon interruption, fluid and
electrolyte imbalance is the most common
complication. The lower colon reabsorbs a major
portion of the fluid, whereas the upper colon does
not have this function. A great potassium loss also
occurs, because it is found in large amounts in the
upper colon.

Question.335= The nurse will know that a patient


with an ileostomy understands dietary restrictions
when she indicates that she does not include which
one of the following foods in her diet

Rational.335= Corn may cause obstruction of the


ileostomy and thus should be avoided.

Question.338= A 54-year-old male patient with a


history of cirrhosis from alcohol abuse has been
admitted for bleeding esophageal varices. While
the patient is on bedrest, he is in a semi-Fowler`s
position. The major objective for using this
position is to

Rational.338= Any position that impedes


respirations by the pressure of abdominal contents
on the diaphragm should be avoided therefore, the
best position for this patient is semi-Fowler`s
position, which increases effectiveness of
breathing.

Question.339= The nurse is assisting the physician


to insert a Sengstaken-Blakemore tube. Prior to
insertion the nursing action is to

Rational.339= In order to prevent the trauma of


reinsertion, the balloons must be checked for leaks
before insertion. This is done by inflating them
and placing them in water to observe for bubbles.

Question.340= A child with the diagnosis of


phenylketonuria (PKU) may not eat which of the
following foods

Rational.340= The accumulation of phenylalanine,


an amino acid breakdown of protein, is toxic to
brain tissue. Therefore, any foods high in protein
must be restricted. These include meat, fish,
legumes, lima beans, milk products, etc.

Question.342= A female patient is being treated


with intracavitary irradiation using a Cesium
implant. The nursing care plan should include

Rational.342= A low-residue diet helps to prevent


frequent bowel movements, a major side effect of
radiation therapy. Absolute bedrest with logrolling
from side-to-side is allowed. The head of the bed
can be raised no more than a 45 degree angle.

Question.343= When a patient with advanced


cirrhosis selects a snack, the choice that indicates
understanding of the dietary requirements is

Rational.343= Carbohydrates are one of the


mainstays of the cirrhotic patient`s diet. The liver
can metabolize only very small amounts of
protein, so usually only 40 to 50 grams of protein
is allowed per day (normal diet is 60 to 80 grams
per day). The banana is the only nonprotein
choice.

Question.346= Which of the following


assessments of the patient with an abdominal
aortic aneurysm requires immediate reporting to
the RN or physician

Rational.346=The increase in blood pressure could


cause a rupture of the aneurysm, so this finding
should be reported immediately. Palpation of a
pulsating mass and a bruit are normal findings.
Moderate anxiety over the surgery is also normal.

Question.347= A patient is having a prolonged


labor and there is no progress past a dilation of 8
cm. Her physician decides to do a cesarean
delivery. The new mother and her partner express
their disappointment that they will not have a
natural childbirth. The best response is to say

Most couples who have an unplanned cesarean


birth feel cheated and disappointed.

Rational.354= To prevent trauma to the mucous


membranes lining the airway, suction should be
Question.354= When using nasotracheal suction to applied intermittently and only while withdrawing
clear a patient`s airway of excessive secretions, the the catheter. The catheter should be lubricated with
nurse would
a water-soluble substance to prevent lipoidal
pneumonia. Suctioning attempts should be limited
to 10 seconds or less to prevent hypoxia.

Question.356= When a patient has lung cancer that


has been classified as Stage 1 , NO, MO, the nurse
knows that this indicates that the patient has a
tumor

Rational.356= "" means the tumor is less than 3


cm in diameter without invasion proximal to a
lobar bronchus. "NO" means there is no
demonstrable metastasis to regional lymph nodes
and "MO" indicates no distant metastasis.

Question.357= The parents of a 4 month old


noticed that many bruises were forming on their
son`s knees, buttocks and thighs. The blood tests
reveal that he has classic hemophilia. The nurse
understands that hemophilia is

Rational.357= Hemophilia is a sex-linked


recessive disorder. The asymptomatic mother
transmits the disorder to the son on the X
chromosome.

Question.362= A drug commonly administered to


reduce the extrapyramidal side effects of
phenothiazines used in the treatment of
schizophrenia is

Rational.362=Cogentin or Artane are the


antiparkinson drugs usually prescribed for
reducing extrapyramidal effects caused by
phenothiazines. Benadryl is also commonly used
and has fewer side effects. The other answers are
incorrect-Niamid and Ritalin are antidepressant
drugs and Valium is classified as an antianxiety
drug.

Question.363= A 2 year old has eaten half a bottle


of his grandmother`s ferrous sulfate tablets. When
the mother calls the clinic, the nurse will tell the
mother to

Question.364= A 58-year-old patient with a


diagnosis of schizophrenia, chronic
undifferentiated type is taking 400 mg of
chlorpromazine (Thorazine) TID. The nurse
notices on the morning rounds that he is drooling
and flapping when he walks. The best nursing
action would be to

Rational.363= Contact either the poison control


center or the emergency department first and
follow their instructions. In this case they will
probably advise giving the child water to dilute the
ferrous sulfate tablets and syrup of ipecac to
induce vomiting. Because some poisons will be
damaging if vomited, the center would not always
advise the mother to give syrup of ipecac. Then the
nurse would instruct the mother to bring the child
to the hospital.
Rational.364= The patient is experiencing side
effects to phenothiazines, but these are not
life-threatening and can be brought to the attention
of the RN and physician within an appropriate
amount of time. (4) is incorrect. Thorazine is
effective at certain blood levels, and holding the
drug would lower the blood level. If possible, it is
preferable to check with the physician before
holding a drug.

Question.365= A 65-year-old female patient has


suffered a cerebral vascular accident (CVA)-left
hemisphere lesion. The most appropriate method
of communicating with her is to

Rational.365= For a left hemisphere lesion, the


best method of communication is to pantomime
what you are communicating while speaking in a
normal tone of voice. Pantomime will confuse a
person who has suffered a right hemisphere lesion.
Before communicating, however, the nurse will
assess the patient`s ability to understand speech. It
is also important to give feedback as you
communicate.

Question.367= A young patient, following a


motorcycle accident, sustained spinal cord injury
with respiratory function impairment. The cord
segments involved with maintaining respiratory
function are

Rational.367= Nervous control for the diaphragm


(phrenic nerve) exists at the level of or of the
spinal cord.

Rational.368= GI bleeding is a very common


Question.368= The nurse is assigned a patient with
complication associated with portal hypertension.
the diagnosis of portal hypertension. The most
Obstruction of portal circulation leads to increased
important assessment with this condition is for the
collateral circulation which can result in bleeding
complication of
tendencies.

Question.369= An elderly patient with organic


Rational.369= In organic brain disorder,
brain syndrome suffers from insomnia and asks the
barbiturates commonly cause delirium, confusion,
nurse for something to help him sleep. The
and paradoxical excitement, thus they should not
physician will not order barbiturates for this
be ordered for patients with organic brain disorder.
patient because they cause

Question.370= A diabetic patient takes 22 units of


Rational.370= Intermediate insulin peaks from 8 to
NPH insulin at 7:30 AM each day. Evaluating the
12 hours after injection. 3:30 PM is the most
effects of insulin, at which time during the day will
appropriate time to assess for signs of insulin
the nurse assess her for signs of restlessness,
reaction.
memory lapses and headache

Question.374= A patient is to receive 2500 mL of


IV fluid over 24 hours. The IV tubing delivers 15
gtts per 1 mL. How many drops per minute should
the patient receive

Rational.374= First, determine the number of mL


to give the patient each hour by dividing the total
volume by the total number of hours: 2500 mL
divided by 24 hours yields a desired hourly
amount of 104 mL. One method for determining
the minute rate uses the formula of: Volume
divided by Minutes multiplied by gtts/mL equals
gtts/minute. 104 divided by 60 multiplied by 15
gtts/mL equals 26 gtts/ minute.

Question.378= The highest priority goal in the


care of a newborn with tracheo-esophageal fistula
(TEF) and esophageal atresia is to

Rational.378= The anatomical malformation in


this anomaly threatens the newborn`s airway.
Maintaining a patent airway is the highest priority
in any situation in which the airway is threatened.

Question.385= A patient, age 60, is admitted to the


hospital for a possible low intestinal obstruction.
His preoperative work-up indicates vital signs of
BP 100/70, P 88, R 18, and temperature of 96.4(F.
Listening to bowel sounds, the nurse would expect
to find

Answer3.385= Hyperactive, high-pitched sounds.

Question.386= A patient, age 68, has an external


shunt placed in preparation for hemodialysis. The
nursing care plan will include

Rational.386= Shunts should be inspected several


times each day for presence of possible clotting.
Dark spots will quickly be followed by separation
of the sera and cells if clotting becomes complete.
When dark spots appear, patients should be
instructed to immediately seek treatment for
declotting.

Question.387= A patient sustained a fracture 3


days ago. When the following blood studies are
returned, they are all elevated. Which elevation
would be considered a normal finding following a
fracture

Rational.387=Bone is rich in alkaline phosphatase


and blood levels normally increase following a
fracture and during fracture healing. Elevation of
the other blood studies should alert the nurse to the
need for further assessment of the patient, because
of the probable presence of an illness.

Question.388= A patient with a fractured right leg


Rational.388= Dorsiflexion of the foot requires an
has been in Buck`s extension traction for a week.
intact peroneal nerve. Compression from any part
The nurse checking the patient finds that he is now
of the traction apparatus along the lateral surface
unable to dorsiflex his right foot. The nurse will
of the leg just below the knee can exert pressure on
notify the charge nurse because there is a
the peroneal nerve and impede its function.
possibility that the

Question.389= During visiting hours, a patient the


nurse is caring for becomes very agitated and
angry with his visitor. The most effective nursing
approach to this patient is to

Rational.389= This approach would help decrease


the patient`s anxiety and assist him in gaining
insights.

Question.391= Using Leopold`s maneuvers, the


nurse palpates the presence of a firm round
prominence over the pubic symphysis, a smooth
convex structure on the patient`s right side,
irregular structures on the left side, and a soft
roundness in the fundus. The nurse would
conclude that the fetal position is

Rational.391= The head is down, the back is on


the right side, legs on left, and fetus` buttocks in
the fundus indicate the position of ROA.

Question.392= A patient who developed cerebral


edema following a head injury is given mannitol
(Osmitrol) intravenously. The outcome that most
clearly indicates the drug has achieved its desired
therapeutic effect is when the

Question.393= When a patient with a diagnosis of


manic episode returns to the clinic to have lithium
blood levels checked, her lithium level is only
slightly higher than the previous week but she
complains of blurred vision and ataxia. The first
intervention is to

Rational.392= Mannitol is given to reduce cerebral


edema by promoting the movement of water from
the tissues into the plasma followed by its
excretion through the kidneys. The patient`s level
of awareness is the most sensitive indicator of the
effects of increased intracranial pressure.
Improvement in the level of awareness, therefore,
indicates a therapeutic response to the mannitol.
The increased urinary output is simply a means
through which the desired therapeutic effect is
achieved. The absence of seizures does not
indicate a therapeutic response to mannitol.
Slowing of respirations may indicate increased
cerebral edema.
Rational.393=These are symptoms of toxicity and
the nurse must withhold the next dose. The nurse
would then notify the physician. The patient needs
to maintain a normal fluid level to prevent toxicity,
but this may not be the cause of her symptoms.

Question.395= A quadriplegic patient tells the


nurse that he believes he is experiencing an
episode of autonomic hyperreflexia (dysreflexia).
The first nursing intervention is to

Rational.395= Blood pressure can become


dangerously elevated during an episode of
dysreflexia and can cause cerebral and retinal
hemorrhages. Elevating the head will help prevent
these complications and should be the nurse`s first
action. Identifying the precipitant is useful in
terminating the episode by removing the noxious
stimulus which provoked the exaggerated
autonomic response. A full bladder may
precipitate dysreflexia and emptying the bladder
would be appropriate if it was the precipitant. The
blood pressure and pulse should be monitored
throughout the episode of dysreflexia.

Question.396= A 3 month old with a diagnosis of


chalasia is admitted to the hospital. He has had
severe weight loss because of frequent vomiting.
To minimize vomiting, the nurse would place the
infant

Rational.396= The greater curvature of the


stomach is toward the left side so the right side
position affords less pressure. Elevation of the
head would lessen the tendency to vomit.

interaction with othes


Question.398= Which of the following behaviors
would indicate improvement in coping for a
patient

Rational.398= If a patient can initiate interaction


with another patient, it indicates he is not totally
absorbed in himself, too depressed to initiate, or
too absorbed in delusions or hallucinations to
interact.

Question.400= An elderly patient with the


diagnosis of COPD has been admitted to the
hospital. In teaching the patient about his diet,
which diet choice would indicate that a patient
with COPD understands nutritional needs

Rational.400= Carbohydrate metabolism produces


carbon dioxide which increases the blood levels of
carbon dioxide. High protein prevents muscle
wasting and helps preserve the strength of
muscles, including the muscles of respiration.
Calorie and energy needs are met by increasing the
fat intake.

Question.410= A patient has an arteriovenous


fistula as an access site for hemodialysis. Which
assessment finding indicates that the fistula is
patent

Rational.410= The flow of blood through a patent


arteriovenous fistula produces turbulence
manifested by a bruit audible when the fistula is
auscultated.

Question.422= A male patient is becoming


increasingly angry and verbally abusive. The first
nursing intervention is to

Rational.422= The first intervention would be to


calmly set limits to defuse the situation. The nurse
would not place the patient in restraints until he is
totally out of control nor would the nurse summon
a male staff member initially -a sudden
involvement of others could escalate the situation.
After the initial intervention, the patient could be
sent to his room.

Question.423= Teaching a new mother the


principles of breast feeding, the nurse will know
the mother understands how to care for her breasts
when she says

Rational.423= The breasts need to be dried after


feeding. No soap should be used because it
removes natural oils. The baby should not be
pulled off the breast because it will make the
breast sore. The baby should not be nursed more
than every 2 hours.

Question.425= The nurse in a well-baby clinic


reminds a mother that at 4 months of age the infant
should receive immunizations that include

Rational.425= At 4 months, most children receive


DTaP, (diphtheria, tetanus and pertussis toxoid)
inactivated oral polio vaccine hemophilus
influenzae type b Pneumococcal vaccines and
hepatitis B, if not given earlier. MMR (measles,
mumps and rubella) is given at 15 months and 11
to 12 years. This is recommended by the Schedule
Committee on Infectious Disease from the
American Academy of Pediatrics.

Question.431= The nurse is assigned to a patient


who is to have a Miller-Abbott tube inserted.
Assisting the physician with this procedure, the
nurse will position the patient in

Rational.431= High-Fowler`s with neck flexed is


the position necessary for tube insertion, as it is
inserted through the nose into the intestine.

Question.454= When performing naso-oral


suctioning, the correct nursing action is to

Rational.454=Clean, rather than sterile, gloves can


be used for naso-oral suctioning and the nurse
would insert the catheter through the nares without
applying suction. Suctioning is limited to 10
seconds to prevent excessive removal of oxygen.
Suction on the catheter is released intermittently
during the procedure and when being withdrawn.

Question.458= A patient makes a suicide attempt


Rational.458= Even though all of the reasons are
on the evening shift. The staff intervenes in time to
important and should not be ignored, the most
prevent harm. In assessing the situation, the most important task for the staff is to assess the patient`s
important rationale for the staff to discuss
behavior and to identify cues that might indicate
regarding the incident is that
an impending suicide attempt.

Question.460= A manic patient is on lithium. The


nurse will instruct her to report signs of lithium
toxicity, which will not include

Rational.460= Diarrhea is associated with lithium


toxicity. Some common symptoms of lithium
toxicity are diarrhea, vomiting, nausea,
drowsiness, muscle weakness, ataxia, blurred
vision, confusion, dizziness, and restlessness.

Question.462= A baby is born with a


myelomeningocele. The best position for this baby
before surgery is

Rational.462= The prone Trendelenburg position


is preferred to minimize tension on the fragile
meningeal sac. In addition, feces and urine are less
likely to come into contact with the defect.

Question.463= Assessing for the classic signs of


basilar skull fracture, the nurse will observe for
blood behind the tympanic membrane and

Rational.463= Cerebrospinal fluid leakage needs


to be monitored closely. If leakage persists beyond
a week, surgical repair may be necessary. Pain in
the back of the head and retinal hemorrhage are
not associated with a basilar skull fracture.

Question.465= An infant is hospitalized with a


diagnosis of meningitis. A late sign of meningitis
that the nurse should immediately report is

Rational.465= Because the infant`s fontanels are


open, opisthotonos, a sign of increased intracranial
pressure, appears late in the course of the illness.
Hypothermia is a decompensator adaptation to
many illnesses. "Sunset" sign of the eyes is found
in uncorrected hydrocephalus. The fontanels
continue to bulge as the meningitis remains
unresolved.

Question.467= When an infant is one day old he


has surgery for reduction of a myelomeningocele.
Which nursing intervention is critical during the
postoperative period?

Rational.467= Infants with myelomeningocele are


prone to develop hydrocephalus, which is
sometimes noted before surgery. When surgery is
performed soon after birth, it is a critical nursing
responsibility to observe for signs of increased
intracranial pressure. Passive ROM, manual
expression of urine, and observation of stool are
important nursing activities, but the priority is
assessment for increasing head circumference.

Question.468= If untreated, a child who has


contracted diptheria may die because of

Rational.468= In severe cases persons with


diptheria may develop a membranous lesion that
covers the tonsils and can spread to cover the soft
and hard palates and the posterior portion of the
pharynx. The individual is unable to exchange air
through the membrane and death results.

Question.472= A patient who is 6 weeks pregnant


complains of a thick, white vaginal discharge. She
denies having any itching of the vulva,
blood-tinged mucus, or foul order with the
discharge. The nursing intervention is to

Rational.472= Vaginal secretions during


pregnancy are thick, white, and acidic (pH 3.5 to
6.0). The increased acidity is due to the presence
of Lactobacillus acidophilus. An increase in
leukocytes in the vaginal discharge results in
leukorrhea. No culture or testing is needed.

Question.485= Elements of cause of action in


negligence/malpractice include

Question.493= A female patient experiences


swelling from her left shoulder to her fingertips
within 24 hours after her left modified radical
mastectomy with node dissection. The nurse
explains to the patient that this results most
directly from

Question.494= During preparation for a cardiac


catheterization, instructions include information
about contrast medium injections. The nurse will
tell the patient to expect to experience

Question.507= The nurse observes 210 mL of


bloody drainage in the chest tube collection
chamber of a patient who underwent cardiac
surgery two days earlier. Which of the following
actions should the nurse take?

Question.508= The nurse is caring for a newly


hospitalized patient identified with locked-in
syndrome secondary to amyotrophic lateral
sclerosis (ALS). Which of the following
interventions would be appropriate to establish a
form of communication?

Rational.485= Breach of duty, foreseeability,


causation, injury, and damages are all constituents
of malpractice and negligence. The level of
damages is not related to negligence/malpractice
(1). Negligence/malpractice injuries can be
physical or psychological (2). The permanence of
Rational.493=
When
axillary
nodes are removed
an injury doesn`t
dictate
negligence/malpractice
and biopsied, the lymphatic
(4). system loses important
sites of waste and fluid returns to the venous
system. The resultant swelling can resolve without
incident, or become severe and warrant
intervention. Elevating her left (operative) arm on
pillows decreases the swelling gravity assists the
flow. Mastectomy is not usually offered to a
woman who has widespread breast cancer. Also
the nurse would be stating a medical diagnosis that
could severely alarm the patient. With incisional
drains in place postoperatively, it is unlikely that
drainage from the arm would be impeded enough
Rational.494=
warmth,
facial
flushing,
to allow bloodNausea,
and fluid
collection
under
the
and
a
salty
taste
in
the
mouth
are
all
expected
incision. However, the drains should be routinely
sensations when
an iodine-based
contrast medium
checked
for patency.
is injected. Severe dyspnea could indicate a
beginning anaphylactic reaction or perforation of
the vessels, heart, and/or lungs by catheter.
Generalized itching often heralds an allergic
reaction to the dye. The patient who is cool and
clammy may be experiencing hypovolemic shock
or neurogenic
shock
anxiety.
Rational.507=
Bloody
chestortube
drainage is
expected in the postoperative phase of cardiac
surgery and no immediate intervention is
indicated. Drainage in excess of 500 mL in the
first 24 hours postoperative could indicate
hemorrhage. Suction should never be increased
without a physicians order (1). The chest drainage
system should not be elevated above the level of
the patient`s heart or the insertion site (2).
Drainage in excess of 500 mL for the first 24 hours
postoperative should be reported (3).
Rational.508= Locked-in syndrome occurs in
persons with a paralyzing illness and prevents
them from communicating through writing,
gestures, or speech. Because voluntary eye closure
is spared, using eye blinks in order to
communicate may prove useful. patients with
locked-in-syndrome are unable to write. patients
experience paralysis with locked-in-syndrome.

Question.511= When the nurse is caring for a


patient with a new endotracheal tube, the most
important action is to

Rational.511= Auscultate the patient`s lungs


immediately after the endotracheal tube has been
inserted and every hour thereafter. Oral hygiene,
repositioning, and arterial blood gas results are not
the highest priorities.

Rational.512= A tension pneumothorax occurs


when air is trapped within the pleural space.
Mediastinal shift can develop as pressure
Question.512= Which of these assessment findings increases, causing the layrnx and trachea to deviate
would indicate a tension pneumothorax?
away from the injury. Wheezing on the affected
side is not a symptom of pneumothorax, since the
lung is collapsed therefore, lung sounds would be
distant or absent

Question.516= Which of the following ECG


monitor rhythm-strips would indicate the body is
compensating for heart failure?

Rational.516= Sinus tachycardia is a hallmark sign


of heart failure and results from sympathetic
stimulation. Atrial fibrillation may cause heart
failure (1). Supraventricular tachycardia is an atrial
dysrhythmia, not a common cause of heart failure

Question.517= Nursing care for a 6 month old who


had a lumbar puncture performed to rule out
bacterial meningitis should include

Rational.517= Rest, quiet environment, and fluids


should be promoted postprocedure, and the child
should be positioned flat or slightly elevated.
Restraints can make the child more agitated and
restless.

Question.518= A patient admitted for shortness of


breath demonstrates arterial blood gas reults of pH
7.47; PCO2 33; and HCO3 26. A nurse should
recognize these results as indicative of

Answer3.518= Uncompensated respiratory


alkalosis.

Question.519= Interventions in the plan of care for


a neonate who is admitted for sepsis should
include

Rational.519= Treatment for neonatal sepsis


includes: 1) obtaining blood, urine, and
cerebral-spinal fluids for culture 2) initiating
antibiotic therapy after cultures are obtained 3)
providing respiratory support after blood gas
determinations are drawn and 4) instituting
measures for fever reduction which do not
influence lab data.

Question.521= When evaluating the reliability of


PKU blood test findings, the nurse should be
confident with the validity of the results if the
blood was acquired

Rational.521= The Guthrie test should not be done


until 24 hours after feeding has been started with
formula or breast milk. It takes time for the
metabolites from feeding to accumulate, hence the
need to wait 24 hours.

Question.523= When sending a specimen to the


laboratory for ABG analysis, information that
should be on the laboratory slip includes

Answer1.523= O2 running at 3 L/minute when the


blood was drawn.

Question.528= A toddler is admitted to the


pediatric unit with a diagnosis of near drowning.
The nurse`s initial plan of care on admission
should primarily focus on

Rational.528= With all near drownings, airway


and breathing are the first priorities and can lead to
subsequent cardiovascular, neurologic, and renal
injuries if oxygenation is not maintained.

Question.529= A toddler is scheduled to receive


the measles-mumps-rubella (MMR) and
inactivated polio (IPV) vaccines. The nurse should
not administer the MMR vaccine if the child

Rational.529= MMR is a live vaccine which


should not be given if immunosuppressive therapy
is currently being administered. A mild illness or
HIV positivity is not a contraindication for MMR
immunization.

Question.531= A patient with a bipolar disorder


Rational.531= When lithium is used to treat mania,
who is slightly manic is to start on lithium therapy.
the patient may be aware of a slowing down of the
The patient and the nurse have discussed the
thought processes. Diarrhea is a sign of lithium
drug`s actions. Which of these statements by the
toxicity.
patient indicates he understood the discussion?

Rational.532= Dehydration caused by excessive


heat or fever can cause a buildup of lithium due to
Question.532= When a patient taking lithium has a
sodium loss, and toxicity may then occur. The
fever, it is extremely important for the nurse to be priority of care is to prevent lithium toxicity due to
sure that the patient
sodium and fluid loss (1). Sunlight is not an issue
in the person taking lithium (2). A nonstimulating
environment does not address lithium toxicity (4).

Question.534= If a patient who is addicted to a


benzodiazepine, such as diazepam (Valium), were
to withdraw abruptly from the drug, the nurse
should expect the patient to have

Rational.534= When a patient becomes addicted


and develops a tolerance to Valium, the seizure
threshold of the brain is raised. Abrupt cessation
lowers the threshold and causes seizures.

Question.535= A patient who has been on


long-term antipsychotic drug therapy shows early
signs of tardive dyskinesia. The nurse`s action is
based on the understanding that

Answer2.535= The drug may be discontinued.


Rational.535= At this time, the only known
treatment for tardive dyskinesia is to discontinue
the antipsychotic drug causing the symptoms.
Antiparkinsonian medications are not effective in
treating tardive dyskinesia (1). Increasing the
dosage may mask symptoms initially, but they will
reemerge

Question.536= A patient who is taking an


antipsychotic drug complains of beginning
symptoms of dystonia. The nurse`s action is to

Answer2.536= Obtain an order for benztropine


mesylate (Cogentin).
Rational.536= Dystonia requires immediate
treatment. It is easily treated with an
antiparkinsonian drug, such as Cogentin.

Question.537= A female patient is started on


chlorpromazine (Thorazine) therapy. The patient
has received instructions regarding the drug`s side
effects. Which of these comments by the patient
indicates that she understood the instructions?

Rational.537= Photosensitivity may occur with


Thorazine therapy staying out of the sun and
wearing protective clothing are the best ways to
deal with this problem. Thorazine does not
decrease the desire for sex. It may cause
impotence in men, or irregular menses (1). Weight
gain is a common side effect of chlorpromazine
(2). Sedation is a common side effect of
chlorpromazine (4).

Question.547= A therapeutic nursing strategy to


assist parents whose infant has just died is to

Rational.547= Alerting all personnel to a patient`s


condition can help minimize pain related to
thoughtless remarks. A symbol or a flower placed
on the patient`s door alerts other staff to the
special needs of the grieving. Parents often feel
isolated and health care personnel tend to avoid
these parents due to feelings of awkwardness (1).
Health care personnel do not need to express
sadness, only sensitivity to the parents` loss

Question.548= A patient is admitted to the labor


and delivery unit in active labor. She gives birth to
a 37-week stillborn baby. Immediately following
the birth, the patient accuses the physician of
"making a mistake" and "making a sick joke." She
requests the nurse to bring her baby to her because
she can hear him crying. The nurse`s best action is
to

Rational.548= Actually seeing and holding the


dead infant may help with the grieving process.

Ask her if she wants to hold the baby

Question.552= Which of the following statements


would the nurse use when trying to get the
demented patient, who is angrily arguing about
where he is, to come to dinner?

Answer4.552= "You may not want to eat now, but


it is dinnertime."

Question.554= When working with a depressed


patient, the nurse should focus the initial
nurse-patient contacts on

Rational.554= Establishing trust is essential to


assist a person with depression.

Question.560= The rationale for using


antipsychotic drugs with schizophrenia is to
relieve

Rational.560= Antipsychotic medications work


directly to stop hallucinations in a schizophrenic
patient and are also helpful in eliminating or
minimizing paranoid ideation.

Question.561= A 20-year-old patient on a


psychiatric unit tells the nurse, "You are the only
one who listens to me. The other nurses on this
unit hate me." This person is attempting to create

Answer3.561= Splitting of the staff.


Rational.561= The patient who expresses this is
usually trying to manipulate the nurse in a manner
that will put one staff member up against another.
The patient is not attempting to create a fantasy
world (1). The patient may be trying to create
disorder and confusion, but it is accomplished
through splittinging

When talking with a suicidal patient, it is


important to establish

does he have a plan

Question.566= A 45-year-old woman is


demanding of everyone and shows no concern for
anyone but herself. Erickson would note that her
stage of development is

Answer3.566= Generativity versus stagnation.

Question.567= Which of the following


interventions is most appropriate when one is
working with the aggressive, agitated person?

Rational.567= The highest priority of nursing


diagnoses for the patient who is agitated and acting
aggressively is high risk for injury to self and
others. By creating a calm environment, the patient
is provided with the opportunity to regain control.
The patient is not violent at this time, so calling in
extra staff isn`t appropriate and may exacerbate
the situation (1). Setting limits is appropriate as it
helps the patient regain control, but a more
important priority is to remove persons at risk for
injury

Question.568= When teaching a patient who is


taking lithium to treat a bipolar disorder, the nurse
would note that a way to prevent lithium toxicity is
to

Rational.568= Lithium is excreted by the kidneys,


and increased intake will ensure more efficient
excretion. Exercise and hygiene aren`t methods to
prevent lithium toxicity

Question.571= An African-American mother asks


how to assess for jaundice in her new baby. Which
of the following is the MOST appropriate answer
for the nurse to offer?

Rational.571= Jaundice usually begins in the head


and moves down the body and can be detected by
blanching the neonate`s nose, palms of the hands,
or soles of the feet. The sclera of the eyes will also
be yellow tinted. Neonates of dark skin or olive
complexion may appear to be jaundiced when in
actuality they are not.

Question.577= Which of the following assessment


findings on a 34 week gestation infant who is 4
days old would it be a priority to preport to the
physician?

Answer2.577= Distended abdomen.


Rational.577= Preterm infants are at risk for
necrotizing enterocolitis (NEC), a condition with
50% mortality. Assessment of increasing
abdominal distention or absent bowel sounds,
vomiting, blood in the stools, and signs of
infection are vital to report. Treatment includes
antibiotics, discontinuation of oral feeding,
administration of parenteral nutrition, and possibly
surgery.

Question.579= Which of the following comments


by a patient with pregnancy-induced hypertension
should alert the nurse to potential problems?

Rational.579= Preeclampsia may progress to


eclampsia, the severe phase of PIH. In severe
cases, patients may experience frontal or occipital
headache that is not relieved by analgesics.

Question.580= As part of the assessment of a


patient suspected to have pregnancy-induced
hypertension (PIH), the nurse will

Rational.580= Brisk deep tendon reflexes may


suggest cerebral irritability secondary to decreased
circulation and edema associated with
preeclampsia.

Question.581= A patient with preeclampsia


complains of epigastric pain. The nursing
intervention is to

Rational.581= A sign of impending eclampsia is


right upper quadrant or epigastric pain. Early
detection and management of preeclampsia are
associated with reducing risks and progression of
this condition.

Question.584= During an infant tube feeding the


best position for the infant is to

Rational.584= Parents should be encouraged to


hold the infant during tube feedings. If
regurgitation has been a problem, the infant may
be positioned on his right side.

Question.586= A postpartal patient is complaining


of shortness of breath and chest pain, and
exhibiting tachypnea and tachycardia. The priority
nursing intervention is to

Rational.586= Deep vein thrombasis (DVT) and


pulmonary embolism are possible complications
occurring in the postpartal period. Signs and
symptoms of pulmonary embolism include
sudden, sharp chest pain, syncope, tachycardia,
rales, and tachypnea. Treatment depends upon
rapid assessment and notification of the physician
of the patient status.

Question.590= A newly married couple have


Rational.590= The Billings method, also called the
decided to practice natural family planning. They
cervical mucus method or the ovulation method,
ask the nurse about the Billings method. The nurse
depends on the characteristic changes in the
explains that the Billings method is performed by
cervical mucus at the time of ovulation.

Question.591= When teaching patients about


improving their chances for fertilization, which of
the following time periods should be highlighted
postovulation?

Rational.591= The total critical time for


fertilization is 24 to 48 hours, 12 to 24 hours
before ovulation and 12 to 24 hours after
ovulation.

Question.592= The nurse is teaching the expectant


mother how the production of progesterone and
estrogen prepare the uterus for implantation of the
fertilized ovum. Which pituitary hormone would
the nurse emphasize?

Rational.592= LH is released by the anterior


pituitary, stimulating ovulation and the
development of the corpus luteum, preparing the
endometrium for implantation.

Question.594= Betamethasone is ordered and


given to a patient diagnosed with preterm labor at
34 weeks. In evaluating the effects of this
medication, the nurse would note that it is given to

Rational.594= Betamethasone may be given to the


mother to hasten fetal maturity by stimulating
development of lecithin when membranes are
ruptured and premature labor cannot be arrested.
The incidence and severity of respiratory distress
syndrome has been found to be reduced if
glucocorticoids (e.g., betamethasone) are
administered to the mother at least 24 to 48 hours
before birth."

Question.601= When performing a physical exam


on a newborn infant, the nurse notes a circular
swelling with bluish discoloration around the top
of the baby`s head. This is recorded as

Rational.601= Caput succedaneum is a


generalized, easily identifiable edematous area of
the scalp.

Question.605= When instructing a pregnant


patient about average weight gain, the nurse would
reinforce that the ideal weight gain during
pregnancy is

Answer4.605= 25 to 35 pounds.

Question.607= Assessing a newly admitted patient


to the ER, an early sign of shock would be a

Rational.607= A rise in the pulse rate is one of the


earliest signs of shock. A blood pressure of 110/70
and a respiratory rate of 24 are normal and do not
reflect shock (2) (3). The temperature of
101degree F is not associated directly with a sign
of shock but may indicate many other problems
(4).

Question.611= When teaching a patient the proper


use of a walker, the nurse would direct the patient
to

Rational.611= The walker should be advanced 6


inches (15 cm) in front of the patient before the
patient moves toward it. Picking the walker up and
only using it periodically would negate the
purpose of the walker (2). The patient should be
taught to advance the weaker leg first (3). The
arms should be slightly flexed at a 30-degree angle
at the elbow. A 90-degree flexion is unsafe and
makes the walker too high for the patient (4).

Question.613= Assuming a normal meal schedule,


the patient who takes isophane insulin suspension
(NPH) SC at 6 AM is most likely to demonstrate
symptoms of hypoglycemia at

Rational.613= This intermediate-acting insulin has


a peak action of 6 to 12 hours. If the patient is
given this product at 6 AM, the window for a
reaction would be highest between 12 PM and 6
PM. Seven AM is too short a time period for
intermediate insulin (1). Nine AM would be more
consistent with the peak effects of regular insulin (

Question.614= An order reads "Infuse 1000 mL


D5W over 10 hours." The nurse knows that the
drop factor is 15gtt/mL. You would infuse
________________________drops per minute.

Rational.614=The answer is 25 drops per minute.


To determine the drip rate, first note that 1000 mL
administered over 10 hours equals 100 mL per
hour. 15 gtt/mL x 100 mL = 1500 gtt per hour.
1500 gtt/hour divided by 60 min = 25 gtt/min.

Rational.616= Sudden dyspnea and poor oxygen


saturation may be signs that the esophageal
balloon has slipped. If this is the case, the
Question.616= A male patient is being treated for esophageal balloon should be cut immediately, as
ruptured esophageal varices with a
it is occluding the patient`s airway. Releasing the
Sengstaken-Blakemore tube. His vital signs have gastric balloon will not improve airway function in
been stable and the suction port is draining scant
the person who is experiencing airway obstruction
amounts of drainage. He suddenly becomes
as a result of movement of the inflated esphageal
acutely dyspneic and oximetry reveals an O2 sat of
balloon (2). Increasing suction or irrigating the
74%. The nurse`s immediate action is to
gastric balloon will not improve the airway
function in the person who is experiencing airway
Rational.621=
or intermittent
obstruction
as aContinuous
result of movement
of the traction
inflated
is maintainedesophageal
as prescribed.
The
patient
may
balloon
realign herself, but the weights must remain in
place. Never lift or remove the weights to care for
Question.621= Following the application of
the patient or to assist the patient in moving up in
skeletal traction, a patient complains of "the
bed (2). Caution the patient`s family not to lift or
traction pulling too much." The nurse`s response is
remove the weights. Administering analgesics is
to
appropriate, but the nurse should assist the patient
to realign her body first (3). Do not remove any
weights when caring for a patient in skeletal
traction

Question.623= A patient with a fractured tibia is


being evaluated in the orthopedic clinic for
possible compartmental syndrome. If the patient
were experiencing this complication, she would be
likely to complain of

Rational.623= The signs and symptoms of


compartment syndrome include pain, pallor,
pulselessness, paresthesia, and paralysis.

Question.624= A postoperative patient returns


from the PACU with two chest tubes inserted in
his right chest. The upper chest tube serves the
purpose of

Rational.624= An upper chest tube is used to


remove air. Two chest tubes may be inserted on
one side: an anterior-superior tube for the removal
of air, and a posterior-inferior tube for removal of
fluid. The lower chest tube is inserted to remove
fluid which may possibly have some clots (1). No
chest tube will prevent clots (2). Milking a chest
tube (this procedure is not allowed in many
facilities) helps to facilitate patency of the tube,
but tube placement doesn`t facilitate "milking" (4).

Question.626= A nurse is caring for a patient who


has a tracheostomy tube and requires frequent
suctioning. When performing tracheal suctioning,
the nurse should limit the procedure to periods of

Answer4.626= 5 to 10 seconds.

Question.627= The nurse is reviewing with the


staff a care plan for a patient with myasthenia
gravis. She explains that the main objective of
therapy for a patient with myasthenia gravis is to

Rational.627= Medical management of the patient


with myasthenia gravis is primarily accomplished
by giving anticholinesterase medication to
maintain an effective balance of acetylcholine and
cholinesterase at the neuromuscular junction.
Stimulating synaptic terminals to produce ACTH
or giving ACTH are not effective treatments (1)
(4). Supportive care would be inadequate, because
medication is available to treat this condition (2).

Question.631= After consultation with the


physician, a 71-year-old female is weaned from
mechanical ventilation. After the weaning trial has
been in progress for an hour, the patient`s ABGs
and vital signs are blood pressure 140/78, pulse 94
(sinus tachycardia), respiratory rate 22, Sp0 with
subscript((2)) .98, pH 7.23, Pa0 with subscript((2))
91, PaCo with subscript((2)) 59, HCO with
subscript((3)) 24. Based on the above information,
the nurse interprets the ABGs as indicating

Rational.631= Respiratory acidosis is present


when the pH is decreased and the PaCO with
subscript((2)) is increased. No compensation is
present because the kidney hasn`t started to retain
HCO with subscript((3)). The normal HCO with
subscript((3)) does not indicate a compensatory
mechanism (1). Acidosis is present, but not
compensatory alkalosis or uncompensated
alkalosis (2) (4).

Question.632= A patient receiving radiation


therapy complains about the dark black marking
placed on her skin. What is the most appropriate
comment by the nurse regarding these markings?

Rational.632= Skin markings are used as a marker


for treatment areas and should not be washed
away. They are not permanent or caused by the
radiation

Question.635= A patient with a pituitary tumor is


transferred to a surgical floor after having a
transphenoidal hypophsectomy. The nurse caring
for this patient is aware that which of the
following procedures would be contraindicated in
the immediate postoperative period?

Rational.635= Frequent mouth care should be


given, but the teeth are not to be brushed, because
this may damage the gingival suture line.
Advancing the diet as tolerated, observing for
signs of meningitis, and maintaining the bed at a
30-degree angle are appropriate interventions.

Question.637= The nurse is caring for a patient


who is admitted for Addisonian crisis. Which of
the following symptoms would the nurse expect
the patient to exhibit?

Rational.637= The patient experiencing an


Addisonian crisis experiences the extremes of
hypotension, hyponatremia, dehydration, and
hyperkalemia. The other options are not clinical
manifestations of a sudden life-threatening
Addisonian crisis.

Question.638= A common complication resulting


from bacterial meningitis is

Rational.638= Most complications from


meningitis involve damage to the cranial nerves.
The most common complication is hearing loss.

Rational.641= Absence seizures result in brief loss


of consciousness. They are usually very brief (5 to
Question.641= In children, staring, inattentiveness,
10 seconds) and do not include the dramatic
or daydreaming may be signs of
symptoms (e.g., falling, rigidity, pallor)
characteristic of tonic-clonic seizures.

Question.647= To decrease neurologic sequelae,


children with spinal cord injuries that result in
motor deficits are given

Rational.647= Methylprednisolone is administered


in high doses. The drug must be started within 8
hours of the injury to achieve maximum
anti-inflammatory effect.

Question.648= Digoxin overdose is more common


when the child`s

Rational.648= Digoxin overdose is more common


when serum potassium levels are low. Many
children with heart disease receive diuretics that
cause potassium to be excreted in higher amounts.

Question.649= Children with Kawasaki may


disease often receive aspirin primarily for

Rational.649= Children with Kawasaki disease


develop thrombocytosis, which places them at risk
of thromboembolism. Aspirin is given until the
platelet count returns to normal.

Question.651= Recombinant human


deoxyribonulease (rhDNase) is an aerosolized
medication administered to children with cystic
fibrosis for the purpose of

Rational.651= This medication helps to thin and


liquify tenacious mucus in the bronchioles and
maintain airway patency.

Rational.654= To maintain the prescribed balance


Question.654= The nurse checks the young patient
of pull and countertraction in Bryant`s traction, the
in Bryant`s traction. Proper body alignment is
child`s buttocks must not be allowed to rest on the
being maintained if the legs are perpendicular to
mattress. The angle of the hips should be 90
the trunk and the buttocks are
degrees.

Question.658= A characteristic lesion that occurs


in children with measles (rubeola) is

Rational.658= Koplik spots appear in the mouth


about 2 days before and after onset of the rash.
They are small, irregular, bluish-white spots on a
red background.

Question.661= Which of the following


abnormalities is a sex chromosome defect?

Rational.661= Klinefelter syndrome is the most


common of all sex chromosome abnormalities
occurring in about 1 in 800 - 1000 live male births.
It is caused by the presence of one or more extra X
chromosomes.

Rational.664= A significant amount of the


medication may still remain in the distal tubing.
Question.664= When administering IV
For example, if the tubing held 15 mL and the
medications to children, it is recommended that IV
medication were mixed in 30 mL of IV fluid, 50%
tubing be flushed after an infusion because
of the medication would remain in the tubing if not
flushed through.

Question.667= Giving an infant formula that is


mixed with too little water can cause

Rational.667= Improper mixing of the formula


introduces a high solute intake without adequate
water, which is a cause of hypernatremia.

Question.670= Neonates are at higher risk than are


older children for loss of fluid in the urine because

Rational.670= Neonates have a limited ability to


concentrate or dilute urine thus they are unable to
conserve or excrete fluid in response to fluid
shifts.

Question.678= A 37-year-old G VII, P VI gave


birth in the LDRP room one hour ago. She was
straight catheterized for 950 mL of clear yellow
urine prior to birth. Her labor lasted 8 hours. Since
birth, her vital signs have been stable, fundus firm
and 1 cm above the umbilicus with moderate,
bright, red vaginal bleeding. Which of the
following factors would the nurse assess for
because it would predispose the patient to
pospartum hemorrhage?

Rational.678= Postpartum hemorrage is defined as


blood loss of 500 mL or more after birth. Risk
factors for early postpartum hemorrhage include
uterine atony, trauma, and lacerations. Grand
multiparity (5 or more births) is a predisposing
factor for uterine atony as well as previous uterine
atony, overdistention of the uterus, and precipitous
labor. Postpartum hemorrhage may also occur
later, or 24 hours after birth.

Question.682= A patient requests information


about the cause of megaloblastic anemia during
pregnancy. The nurse should respond

Rational.682= During pregnancy, increased


maternal red blood cell production and fetal
demands can result in folic acid or folacin
deficiency. Folic acid is a co-enzyme necessary for
the synthesis of nucleic acids. Nucleic acid and
nucleoprotein synthesis are required for the
production of red blood cells.

Question.683= The nurse obtains the following


assessment data for a female, 12 weeks pregnant
with her third child: P = 112 (resting), R = 20, T =
99degreeF, profuse sweating, pronounced palmar
erythema. Based on this data, the nurse could
anticipate the physician ordering a

Rational.683= Symptoms of hyperthyroidism


indicate the need for further diagnostic assessment.
These symptoms include tachycardia, weakness,
increased appetite, heat intolerance, sweating,
enlarged thyroid, exophthalmos, nervousness,
weight loss, and the tremors. She may also exhibit
pronounced palmar erythema and resting pulse rate
greater than 100 beats per minute.

Question.687= A patient receiving thyroid


medication for hypothyroidism asks the nurse
whether she can breast-feed. The nurse should
respond

Rational.687= A woman receiving thyroid


replacement therapy can still breast-feed, as only
minimal amounts of thyroid hormone are passed to
the infant through the breast milk.

Question.688= A patient is scheduled for a


culdocentesis. The nurse explains that the test

Rational.688= Culdocentesis may be used to


confirm ectopic pregnancy, especially when
ultrasound is not available. The physician inserts a
needle through the posterior wall into the
cul-de-sac and aspirates fluid. Aspiration of
nonclotting blood may indicate a ruptured ectopic
pregnancy.

Question.691= A new mother asks if


amniocentesis can help detect a fetus with spina
bifida (myelomeningocele). The nurse should
respond that yes, during the fourth month,
assessment of the amniotic fluid would indicate a
nervous system defect if

Rational.691= Between the 14th and 16th weeks


of pregnancy, abnormally high levels of
a-fetoprotein in the amniotic fluid indicate a neural
tube defect. Although the extent of the defect
cannot be determined, an amniocentesis should be
performed before the 18th week because, after this
time, a-fetoprotein levels decrease. In addition,
there would still be time for an induced abortion.

Question.692= The nurse assessing a newborn,


initiates the Moro reflex by

Rational.692= The Moro, or startle, reflex


indicates the newborn`s sense of balance. A
sudden stimulus such as jarring the infant or
making a loud noise should elicit the reflex. The
baby should be lying quietly in the supine position
to provide the best response. Gentle handling of
the infant and the crib will not elicit the reflex.

Rational.693= Immediately after birth the baby`s


temperature is the same or slightly higher than the
mother`s. Because he is wet, suddenly exposed to
Question.693= After his arrival from the delivery
a dry, cool environment, and has a large body
room, the infant`s initial temperature is 36degreeC surface for weight, he loses heat via evaporation,
(96.8degreeF). Nursing interventions should
conduction, convection, and radiation. His
include recording the temperature, dressing and
immature hypothalamus cannot initiate
wrapping him, and then placing him in a (an)
mechanisms such as shivering to raise his body
temperature. Placing him in a heated crib under a
Rational.694=
normal
tonic
should
radiant
warmerAshould
raise
hisneck
bodyreflex
temperature
reveal
that
when
the
head
is
turned
to
one
side,
to 36.6degreeC (97.8degreeF) in 2 - 4 hours.the
arm on the same side is adducted and extended in a
fencing position. The infant with brachial plexus
palsy has flaccid paralysis of the affected arm,
Question.694= A baby is suspected to have
which does not extend when the tonic neck reflex
brachial plexus paralysis of the left arm. Which
is elicted. Brachial plexus is not a congenital
observation would the nurse make during the
defect, so the affected arm is not longer, but may
newborn assessment to confirm the diagnosis?
appear that way because of the limpness in that
arm (1). There is no resistance of the arm when
adducted (3). When the infant has limited use of
the affected arm or asymmetric Moro reflex, a
Rational.695=
normal isMoro
reflex (4).
should be
fracturedAclavicle
suspected
symmetrical and complete. An incomplete or
Question.695= When the nurse elicits a baby Moro
absent Moro reflex indicates brain trauma. An
reflex, his arms and legs adduct and extend
asymmetrical reflex reveals a muscle or peripheral
simultaneously. His arms then adduct in an
nerve injury, as in Erb-Duchenne paralysis. The
embracing position, and his legs flex with his soles
response was symmetrical and complete (1). The
turned inward as he starts to cry. The nursing
response was normal, not hyperactive (3). Since
intervention is to
the response was symmetrical, no weakness or
paralysis is suggested (4).

Question.698= A 6-year-old child is admitted with


a diagnosis of asthma. An early sign of respiratory
distress would be

Rational.698= An increased pulse rate indicates


respiratory distress and a decrease in the oxygen
content of the blood.

Question.700= A young child has a diagnosis of


nephrosis. Which of the following medications
would the nurse plan to discuss with his parents?

Rational.700= The use of glucocorticoids has been


found to have a therapeutic value in treating
nephrosis.

Question.702= The clinic nurse is teaching the


parents of a newborn about the immunization
schedule they will follow. The immunizations that
should be started after the first birthday or at 15
months are

Rational.702= The MMR is recommended to be


given at 15 months. The presence of the maternal
antibody to measles in the infant`s blood is
sufficiently reduced by 12 months, so the vaccine
is effective if given after the first birthday.

Question.706= When monitoring a 10-year-old


child who has undergone a cardiac catheterization,
which of the following signs would have the
highest priority for continued monitoring?

Rational.706= Cardiac catheterization is a


diagnostic procedure but not without risks, one of
which is transient arrhythmias.

Question.709= A young patient with cyanotic


heart disease would present with

Rational.709= The child with a cyanotic heart


defect is more likely to have frequent respiratory
infections because of lowered resistance and poor
cardiac function.

Question.710= The nursing priority when caring


for a child in the early phase of Guillain-Barre
syndrome is

Rational.710= Guillain-Barre syndrome results in


muscle weakness that begins in the legs and
spreads to the trunk, chest, neck, face, and head.
The child may experience weakness of the
respiratory muscles, resulting in inadequate
ventilation that may necessitate intubation and
mechanical ventilation.

Question.713= Follow-up evaluation of children


who have recovered from meningitis is important
because they

Rational.713= The child is at risk of developing


complications from meningitis. The more common
sequelae include hearing loss, attention deficits,
seizures, developmental delay, and septic arthritis.

Question.725= Because drug therapy is often not


successful, the treatment of choice for children
with aplastic anemia is

Rational.725= Aplastic anemia results from the


failure of the bone marrow to produce adequate
numbers of blood cells. A bone marrow transplant
from a compatible sibling or family member donor
is the treatment of choice.

Question.729= Discharge instructions for a child


who has had rheumatic fever need to emphasize
compliance with

Rational.729= On discharge, a daily low-dose


antibiotic is prescribed or a monthly long-acting
antibiotic injection is given. This prophylactic
treatment may be indefinite, and the family must
understand the importance of preventing future
strep infections, and that heart damage can result
from recurrent rheumatic fever.

Question.731= A child with thalassemia minor or


thalassemia trait would exhibit which of the
following symptoms?

Rational.731= Thalassemia minor or thalassemia


trait produces a mild microcytic anemia.

Question.732= Which of the following locations


would place a child with sickle cell anemia at
greatest risk for a sickle cell crisis?

Rational.732= Hypoxia resulting from general


anesthesia is a major surgical risk. Sickle cell
crisis could begin during surgery, placing this
patient at high risk. Special precautions must be
taken before any surgery. Therefore, it is necessary
that children with sickle cell anemia be
appropriately identified prior to surgery.

Question.742= An infant is 2 months old and had


Rational.742= Crying increases tension on the
repair to a cleft lip 10 hours ago. He is now crying. suture line. The nurse should anticipate the infant`s
The most important nursing intervention would be
needs and provide immediate comfort by having
to
his mother hold him.

Question.747= A safety precaution when caring


for a child with a tracheostomy is to

Rational.747= A clean tube of the same size


should be immediately ready for insertion in case
the tracheostomy tube should fall out. A second
tube of the next smaller size should also be
available in case there is difficulty inserting the
same-size tube.

Question.748= A child with cystic fibrosis is at


risk for a deficiency in which vitamins?

Rational.748= Vitamins A and D are fat soluble. A


child with cystic fibrosis may have difficulty
digesting fats - so these vitamins will not be
assimilated - because of the absence of pancreatic
enzymes. Children are given water soluble forms
of vitamins A, D, E, and K.

Question.750= To determine unresponsiveness in


an infant, the nursing action is to

Rational.750= Unresponsiveness is determined by


gently tapping the infant and speaking loudly
enough to get a response.

Question.751= Infants develop bronchopulmonary


dysplasia as a result of

high pressure vent


Rational.751= Bronchopulmonary dysplasia is a
condition that results from the treatment given to
infants with severe respiratory problems.

Question.755= The young patient has just been


admitted for evaluation. The nurse reports that the
physical assessment reveals a finding that may
indicate coarctation of the aorta, which is

Rational.755= Coarctation is a narrowing that


occurs in the descending aorta. Blood flow to the
legs is decreased, and flow to the arms and head is
increased. The femoral pulses are weak, and the
radial pulses are full. The blood pressure is higher
in the arms, lower in the legs.

Question.756= The pediatric nurse is teaching


inservice to a new RN staff member. She knows
the RN understands accepted skill procedures
when she says the dorsogluteal muscle can be used
for injections for children at the age of

Rational.756= The dorsogluteal muscle develops


with walking and should not be used for injections
until the child has been walking at least 1 year.
However, because these muscles are poorly
developed, they are not the ideal choice for a child
younger than 5 years old.

Question.757= A child with cerebral palsy needs a


diet

Rational.757= The child with cerebral palsy


requires a high-calorie diet because of increased
muscle activity and feeding problems. Many
children with cerebral palsy have difficulty
chewing and swallowing. Offering high-calorie
soft foods in small frequent feedings is often
helpful.

Question.758= A child has been diagnosed with


pulmonary stenosis. The nurse is teaching the
parents about their child`s diagnosis and tells them
that Pulmonic stenosis causes

Rational.758= Stenosis (narrowing) of the


pulmonary artery valve makes it more difficult for
blood to enter the pulmonary artery, causing
increased pressure within the right ventricle. This
leads to right ventricular hypertrophy, because the
ventricle must pump harder to move the blood into
the artery.
r ventricle affected

Question.760= An infant has been diagnosed with


patent ductus arteriosus. The nurse teaches the
parents that symptoms resulting from this
condition are directly related to the

Rational.760= After birth, blood in the aorta is


under higher pressure, causing blood to flow
through the ductus into the pulmonary artery. This
extra blood entering the pulmonary system can
lead to increased pulmonary vascular congestion.

Question.768= The needle size that the nurse


would use to give an intramuscular (IM) injection
to a newborn is a

Rational.768= You would use a thin (25-gauge),


short (0.5-inch) needle for giving an IM injection
to a newborn.

Question.786= Which of the following data place


a patient at risk for developing pregnancy-induced
hypertension (PIH)?

Rational.786= Primigravid women or women


pregnant for the first time after an abortion are
more susceptible to PIH than are multiparous
women.

Question.789= A patient with hyperemesis


gravidarum is at risk for having a (an)

Answer1.789= Infant who is growth retarded.

Question.792= The nurse should assess the


small-for-gestational-age (SGA) neonate for

Rational.792= Small-for-gestational age neonates


frequently develop hypoglycemia because of low
glycogen stores.

Question.798= A male patient is being treated for


exacerbation of his chronic obstructive pulmonary
disease (COPD). Oxygen would be administered at
2 liters via nasal cannula because

Rational.798= Precise amounts of oxygen


delivered to COPD patients is critical and depends
on the hypoxic drive of the patient. Too much
oxygen can depress respirations and cause
hypoventilation. The size of the lungs is not
directly related to the hypoxic drive (1). The
opposite would occur in this patient if given high
concentrations of oxygen (2). The statement about
his lungs and chest diamter is true, but does not
explain why he responds to hypoxia (3).

Question.799= Which of the following


interventions would the nurse plan to do for the
patient demonstrating symptoms of Addisonian
crisis?

Rational.799= Addisonian crisis is a


life-threatening complication of Addison`s disease
(hypocorticism). Cortisone replacement can be
life-saving. The patient is likely to be severly
dehydrated and hypovolemic. IV saline will be run
rapidly in bolus amounts (

Question.804= Several days following surgery to


remove a thyroid tumor, a female patient begins to
complain of muscle twitching in her legs. The
most likely cause of this is

Rational.804= Accidental parathyroidectomy after


removal of the thyroid gland results in a
hypocalcemic condition manifested by GI
complaints (vomiting and diarrhea), convulsions,
and tetany.

Question.807= While walking with a 79-year-old


dementia patient, the nurse shows him a car key.
She asks the patient to name the object, but he is
unable to. This inability to describe the key is
known as

Rational.807= The failure to recognize previously


known objects is called agnosia. This condition
can be seen in several types of nervous system
disorders such as Alzheimer`s dementia and
cerebral vascular accidents. Apraxia is the inability
to carry out a series of actions (1). Aphasia is a
disorder in communication (2). Agraphia is the
inability to write (3).

Question.811= Which of the following tests would


the nurse employ when evaluating the hearing of a
patient?

Rational.811= The Weber test is conducted by


using a tuning fork to measure whether the patient
is able to hear sound conducted to both ears. If
there are conductive or sensorineural losses, it will
lateralize to one of the ears. The Trendelenburg
test is used to check vascular competency in the
lower leg (1). The Snellen test determines vision
(3). The Allen test assesses for radial and ulnar
patency at the level of the wrist (4).

Question.816= A patient with carcinoma of the


thyroid suddenly develops symptoms of
pheochromocytoma. Which of the following
symptoms would the nurse consider a top priority
when planning nursing interventions?

Rational.816= Severe, life-threatening


hypertension can result from excess catecholamine
secretion.

Question.817= The nurse is caring for a patient


with aldosteronism. The nurse should anticipate
which of the following symptoms?

Rational.817= Hypertension and hypokalemia are


common signs of aldosteronism.

Question.831= The nurse is preparing to


administer a blood transfusion in the emergency
room. After ordering the blood, the nurse`s first
action would be to

Question.836= The patient who has had an MI


hopes to resume sexual intercourse with his
partner. The nurse determines that the patient
understands the instructions given for resuming
sexual intercourse on discharge from the hospital
when he says he will

Question.837= While developing a care plan for a


patient receiving intermittent pelvic traction (to
relieve lower back pain due to sciatica), the nurse
would include

Rational.831= The patient and the family should


be supported prior to and during the procedure,
and anxieties should be allayed by the nurse.
Rechecking the order and reviewing the typing and
crossmatching are done by two nurses as part of
the pretransfusion checklist (1) (3). It is seldom
necessary to contact the laboratory, because
information on the blood bag label is sufficient to
Rational.836=
on the
the proper
recovering
determine thatThe
thestrain
lab sent
unit heart
(4).
should be avoided. After a heavy meal, blood flow
is diverted to the GI tract. Increased workload
increases the risk of underperfusion of the
coronary arteries. The patient should lie on his or
her back while the partner kneels to take some of
the weight from the patient (1). Answer (2) is not
an appropriate recommendation for this patient.
The patient recovering from an MI without
complications may usually resume sexual activity
in about 5 to 8 weeks. One index of readiness is
the ability to walk up two flights of stairs without
Rational.837=
Pelvic traction
is skin
becoming
dyspneic
(4). traction that
can be manipulated by the patient. It can be
intermittent and the patient may not require it,
except during specific times. The patient`s pelvis
should partially rest on the bed for support (2). The
patient`s knees should be slightly bent to relieve
pressure on the lower back (3). Pelvic traction
does not involve pins inserted into any extremity,
as is the case in skeletal traction (4).

Question.838= A 45-year-old male patient comes


to the clinic for evaluation. He is receiving
continuous ambulatory peritoneal dialysis. For the
past three cycles, he has retained between 250 and
400 mL with each cycle. He has no urine output
and has not moved his bowels in 3 days. The
effluent is clear. The nurse should

Rational.838= The physician should be notified


When the CAPD patient has a retention of more
than 250 mL of fluid, as glucose absorption may
become a problem. The physician should also be
notified if the outflow is 500 mL greater than the
amount that was instilled. Dialysate solution
concentrations should not be changed without a
physician`s order (1). The patient may be
constipated, but the overall priority at this time
should be to notify the physician (3). The next
cycle should not be started until after discussing
this problem with the patient`s physician (4).

Question.842= An elderly patient is being


transferred to the rehabilitation unit. The nurse
explains that the focus of his care will be "tertiary
prevention," or

Rational.842= Tertiary prevention care includes


restorative and rehabilitation activities to obtain
optimal level of functioning. Screenings and saftey
education are examples of primary prevention.
Illness care, such as first aid, is an example of
secondary prevention.

Question.847= A 45-year-old female patient


refuses to get out of bed and resists the nurse`s
attempts to assist her to sit up in a chair. If the
nurse attempts to remove the patient from her bed
without the patient`s approval, the nurse could be
charged with

Rational.847= Battery is defined as the offensive


touching of the body of an individual. It is not
necessary to have caused harm. Assault is to place
someone in fear of harm. Negligence is conduct
that falls below the standard of practice. A tort is a
civil wrongdoing.

Question.851= Which of the following disorders


would the nurse classify as a disease that is spread
through fomite transmission?

Rational.851= Formites are nonliving objects that


transmit infection. Examples include the spread of
ringworm of the scalp via contaminated combs,
hats, or the upper back portion of bus seats.
Tuberculosis is an organism spread through
airborne droplets. Lyme disease and Rock
Mountain spotted fever are spread through infected
ticks.

Question.857= Screening for elevated blood lipid


levels is an example of what level of prevention?

Rational.857= Early diagnosis and prompt


treatment are part of secondary prevention.
Primary prevention is aimed at health promotion
and protection against illness. Tertiary prevention
is aimed at rehabilitation and return of the patient
to maximum level of functioning.

Question.859= Conducting a diabetes screening


clinic for well adults is an example of

Rational.859= Secondary prevention includes


screening techniques and treatment of early stages
of disease.

Question.863= The method most likely to be used


to reduce the heart rate in a child experiencing
supraventricular tachycardia is to

Rational.863= Applying ice to face or iced saline


solution to the face causes vagal stimulation that
may reduce the heart rate. An older child can
perform the Valsalba maneuver to increase
intrathoracic and venous pressures, thus slowing
the heart rate. Other activities will not reduce the
heart rate.

Rational.865= Cystic fibrosis is an inherited


autosomal recessive disorder, which means that
both parents must carry the gene in order for a
Question.865= When a child is diagnosed with
child to have the disease. In past generations most
cystic fibrosis, the news is often met with disbelief
children with cystic fibrosis died very young, often
by the parents because
without ever having been diagnosed. Adults are
often unaware that they are carriers of this
disorder.

Question.869= A 4-year-old child has had chronic


otitis media. Parent education after myringotomy
and insertion of tympanostomy tubes would
include

Rational.869= The tubes allow air and fluid to


flow from the middle to the outer ear and also
allow water to flow in the opposite direction. To
avoid introducing bacteria directly into the middle
ear, the child should wear earplugs when
swimming because submerging or diving places
the child at increased risk for otitis media. Noise
has no effect on tube placement and tubes fall out
spontaneously.

Question.871= Which of the following symptoms


may be an indication of respiratory alkalosis in a
young child?

Rational.871= Respiratory alkalosis leads to


stimulation of the nervous system. The clinical
signs include nervousness tingling in fingers, toes,
and around the mouth and tetany.

Question.876= In counseling the parents of a child


with idiopathic thrombocytopenic purpura (ITP)
who has not responded to steroids and
immunoglobulins, the nurse will reinforce the
physician`s plan that the next treatment would be a

Rational.876= In ITP the platelets are destroyed


faster than they are produced by the bone marrow.
The spleen is the organ that destroys the platelets.
Splenectomy is reserved for those patients in
whom ITP has persisted for 1 year or longer.
Transfusions and transplantation are not indicated
in the treatment of ITP.

Rational.877= DDAVP is a synthetic drug that


temporarily increases the activity for factor VIII
Question.877= To avoid risk of transmission of the two- to threefold. The chance of HIV transmission
human immunodeficiency virus (HIV), children
is eliminated, because the drug is synthetic and
with mild hemophilia are often treated with
thus contains no human blood components. Factor
VIII is not used to treat mild hemophilia and
cryoprecipate is no longer recommended.

Question.883= A patient is being assessed for


suicidal risk. Which of these factors in the
patient`s history places the patient at risk for
suicide?

Rational.883= Suicide tends to run in families. It


may be that because one family member
committed suicide, suicide can be viewed by other
family members as an "appropriate" coping
mechanism. The other answers do not place a
person at risk for suicide.

Question.885= The nurse and a patient who


resides on a psychiatric unit are discussing the idea
of contracting. The patient has a history of violent Rational.885= Contracting is a system whereby the
behavior. The nurse will know that the patient
nurse, staff, and patient together decide how
needs further instruction about the concept of
aggressive behavior will be handled.
contracting if the patient makes which of these
statements?
Rational.894= Delusions are fixed beliefs
maintained despite experience and evidence to the
contrary. Hallucinations are a distorted
Question.894= On the psychiatric unit, the nurse is
misrepresentation of reality, channeled through the
talking to a patient who consistently claims that his
patient`s senses (2). Projection is putting feelings
food is being poisoned. The nurse should
that are unacceptable to the self onto another
recognize this symptom as
person or object (3). Dissociation describes the
abrupt disengagement with the present and loss of
contact with reality (4).

Rational.895= Schizophrenia is a chronic disease


Question.895= A patient who has been diagnosed
and various factors may lead to intermittent
as having schizophrenia is readmitted to the
hospitalization for acute exacerbation of
hospital. It is the patient`s third admission in 3
symptoms. These admissions should not be viewed
years. The patient`s mother says to the nurse, "I
as a treatment failure. Noncompliance would lead
don`t understand what my son is doing wrong.
to repetitive and frequent hospitalization (2). There
Every year he has to be hospitalized." It is
is no way to predict the frequency of
appropriate for the nurse`s response to be based on hospitalization (3). Reevaluating treatment options
which of these understandings?
can be done in the outpatient setting as well as in
the hospital (4).

Question.901= A patient is diagnosed as having a


schizoid personality disorder. The nurse can
expect this patient to exhibit a pattern of

Rational.901= Schizoid personality disorder


produces a pattern of detachment from social
relationships and a restricted range of emotional
expression.

Question.903= When the nurse begins to give the


patient a prescribed medication, the patient says,
"You`re poisoning me with those pills." Which of
these responses by the nurse is most appropriate?

Answer1.903= "I`ll get a fresh package of pills so


you can see me open the sealed
package."Rational.903= patients who are having
delusions, such as thinking that their medications
are being poisoned, need to be given an
appropriate degree of choice, e.g., seeing a closed
medication packet and watching the nurse open the
packet. The paranoid thought process isn`t
clarified by attempts to explain or establish logic.

Question.905= A young woman patient on the


psychiatric unit has a fluctuating mental status.
She can be calm and cooperative, and yet an hour
later be paranoid and combative. Her response to
environmental stimuli ranges from an appropriate
response to fleeting illusions. The most imperative
nursing care need for the patient is to

Rational.905= Providing safety for the patient and


staff is the primary concern on a hierarchy of
needs. Encouraging fluid intake, maintaining skin
integrity, and providing reality orientation are less
important needs.

Question.908= When the nurse suspects that an


individual in crisis may be having thoughts of
suicide, the appropriate intervention is to

Rational.908= Asking directly about a suicide plan


will help you determine the degree of risk for the
patient. Not asking, inquiring about moral beliefs,
and trying to persuade the patient that suicide is a
poor choice are less effective measures.

Question.917= A 50-year-old Asian American


woman was admitted to a psychiatric unit. She
states that she does not know why she is on a
psychiatric floor because she definitely is not
crazy. In your initial interview, you choose an
unstructured format in order to

Rational.917= An unstructured format will help


establish a trusting relationship, something that
must occur before she can begin to identify and
work on problems. Teaching about psychiatric
units will occur after you obtain more information
about the patient`s problems (2). Trying to
convince the patient she is in the right place will
invalidate her perceptions (3). It`s important to
focus on both verbal and and nonverbal behavior
(4).

Question.918= As you walk onto a psychiatric


unit, a 16-year-old adolescent yells at you, "You
are like all the rest. You are full of !@#$!" The
best response to her would be

Answer3.918= "It sounds like you are upset."

Question.924= A young woman is admitted to an


acute care psychiatric unit for having attempted
suicide. She is a recent graduate of a nursing
program and is employed at a local hospital. You
recognize that interaction with her may be more
complicated because she is a nurse. The most
obvious potential problem for you would be

Rational.924= Because the patient is also a nurse,


identification may make it difficult to maintain
well-defined boundaries. Because the patient is
also a nurse does not relate to remaining objective
(2). Confidentiality is an issue with all patients (3).
Your relationship with the patient should be
therapeutic, not professional (4).

Question.928= The drug of choice for alcohol


detoxification usually is

Rational.928= Benzodiazepines are commonly


used to decrease the CNS irritability that occurs
during alcohol withdrawal. Barbiturates are used
for benzodiazepine or sedative-hypnotic
withdrawal (2). Antihistamines are used to combat
sleeplessness (3). A neuroleptic would be given to
treat psychotic disorders (4).

Question.929= A patient on lithium believes salt is


bad for people, so she eliminates all sodium from
her diet. In counseling her, the nurse should

Rational.929= Deficiency of sodium results in


more lithium being reabsorbed, thus increasing
risk of toxicity.

Question.930= A male patient with the diagnosis


of depression is given a monoamine oxidase
(MAO) inhibitor. The nurse knows that MAO
enzymes destroy

Rational.930= Neurotransmitters are the targets of


MAO inhibitors. Normal transmission with a
MAO is partially responsible for keeping synaptic
levels of the neurotransmitter low.

Rational.931= Depersonalization is often


associated with an altered perception in which
Question.931= A 38-year-old patient who has been
persons feel as if they are having an out-of-body
diagnosed with schizophrenia comes to the nurse
experience. They report robot-like movements and
and states that his legs are moving like a robot.
loss of sensations in different body parts.
The nurse understands this to be a sign of
Delusional behavior is seen in examples of
delusions of grandeur ().

Question.932= An elderly gentleman is brought to


in the clinic and states, "The radio is sending
messages to the police that I am to be interrogated
and arrested." The nurse interprets this
communication to be consistent with

Rational.932= Ideas of reference are defined by


the patient`s belief that everything happening
around him is related to himself. Delusions of
grandeur represent an exaggerated belief of
self-importance (2). Hallucinations are not a
delusional experience (3). A disturbance in
volition represents an inability to initiate activities
(4).

Rational.939= patients who are scheduled to


undergo ECT should have a complete history and
physical examination to rule out any
contraindications for therapy. Diseases such as
Question.939= Electroconvulsive therapy has been
ordered for a patient, who has severe depression. brain tumors, seizures, and trauma to the head may
What is the most important assessment to
be contraindications for ECT. A family history of
unremitting depression does not provide
complete before the procedure?
information that would contraindicate ECT (2). A
social history that includes lengthy bouts of
sadness is not a contraindication of ECT (

Question.940= A patient is being maintained on


lithium therapy for bipolar disorder. Her serum
lithium level is 0.9 mEq/L. The nurse assesses this
level as

Rational.940= The therapeutic range for effective


ongoing management of the patient on lithium
carbonate can be as low as 0.6 mEq/L or as high as
1.5 mEq/L. A value of 0.9 mEq/L is within the
therapeutic range for lithium.

Rational.941= The development of extrapyramidal


symptoms is a side effect of this medication, and
tremors are part of the constellation of symptoms.
Medications are frequently prescribed to
Question.941= While caring for a 56-year-old
counteract and relieve these symptoms. The
patient, the nurse observes a newly developed
patient has begun to develop symptoms of side
slight tremor when he is at rest. The patient is
effects. The physician should be notified without
taking prescribed chlorpromazine (thorazine) as
waiting for other problems to occur (1).
part of his management plan for treating
Reassurance that his is a normal response is not
schizophrenia. Which of the following
the correct action (3). The patient has developed
interventions would be most appropriate?
side effects, and it`s important to notify the
physician.
The nurse
should
Rational.947=
When
usingnot
thisdiscontinue
category ofthe
medication
without
an
order
(4).
medications, the patient should be cautioned
against eating foods that contain tyramine. This
includes many foods that have been fermented in
Question.947= Which of the following foods
some way, such as yogurt, aged cheese, sour
should the patient taking an MAO inhibitor such as
cream, champagne, beer, pickled herring, shrimp,
isocarboxazid (Marplan) be taught to avoid?
overripe bananas, and yeast extracts. Spinach,
haddock, and tomatoes do not contain tyramine
and would not be contraindicated for the person
taking an MAO inhibitor.
Rational.1612= Administering oxygen at 5 liters
per minute is too much oxygen for a person who
has chronic lung disease. It will create respiratory
Question.1612= The intervention that would be
arrest potential because these people respond to a
contraindicated when administering oxygen to a
hypoxic drive. Oxygen dries the mucous
patient with chronic obstructive pulmonary disease
membranes so moisture to the nose is a comfort
(COPD) is
measure properly done by the nurse. Since oxygen
is a flammable gas, it is important for staff and
visitors to be aware that it is being used in the
room they are entering.

Question.1608= When caring for a patient


receiving a sitz bath, the intervention is to

Rational.1608= To maximize healing and


relaxation, the sitz bath should last from 10-20
minutes. Water temperature should be about 110
degrees F. Sitz baths are usually done 3 or 4 times
per day to provide cleansing and comfort.

Question.1607= The causative agent of syphilis is

Rational.1607= The causative agent of syphilis is


Treponema pallidum. Hemophilus ducreyi causes
chancroid neisseria causes gonorrhea and
chlamydia trachomatis causes chlamydia.

Question.1603= A patient injects his or her regular


insulin at 6 AM every morning, has breakfast, and
then jogs for three miles. This patient should be
taught to

Rational.1603= Exercise increases insulin


utilization. The patient should be prepared to treat
hypoglycemia as the action of onset of regular
insulin is from 30 minutes to 1 hour.

Question.1600= When administering neomycin


sulfate as part of the management plan for a
patient to prevent hepatic coma, the nurse
recognizes that this medication acts by

Rational.1600= Neomycin works locally on the


bowel to decrease or eliminate the normal bacterial
flora, which then decreases the amount of
ammonia produced. The impaired liver may not be
able to metabolize the ammonia and hepatic failure
may result.

Question.1598= Which of the following


oxygen-delivery systems would the nurse use to
provide the most accurate concentration of
oxygen?

venturi
Rational.1598= This system can deliver oxygen in
the 24% to 100% FIO with subscript((2)) range
with flow rates of 4 to 10 L/min. It is the most
accurate of the delivery systems mentioned.

Question.1596= A female patient is recovering


from an exacerbation of her chronic bronchitis.
Her sputum suddenly changes from a pale-green
color to a rusty-brown color. Assessment of her
lungs reveals bilateral crackles that are scattered in
both bases and do not clear upon coughing. This
change in sputum is most likely to represent

Pneumococcal pneumonia.

Question.1593= A sudden increase in pain in a


patient admitted with duodenal ulcers may indicate
that which condition is developing?

Rational.1593= Acute gastric perforation is


characterized by sudden, sharp, intolerable pain.
Bleeding or extending ulcers do not cause pain. A
pancreatic ulcer is not related to duodenal ulcers.

Question.1592= Cholinergic crisis in myasthenia


gravis is due to

Question.1591= The primary effects of sepsis


include

Question.1590= A patient is admitted to the


intensive care unit with signs and symptoms of
ascending paralysis and respiratory failure. The
critical care nurse would investigate for a past
history of

Question.1589= The nurse is administering


medications to a patient with acute myelocytic
leukemia. One of her medications is allopurinol,
and she questions why she is receiving this. An
appropriate nursing response would be

Question.1588= A 54-year-old man comes to the


clinic for evaluation of a painless lump that has
been present on the side of his neck for the past 6
weeks. Hodgkin`s disease, stage II, was
subsequently diagnosed. He is to begin radiation
therapy and is questioning why "an x-ray can kill
this cancer." The nurse`s best response should be

Rational.1592= Myasthenia gravis is medically


managed with the use of anticholinesterase
medications (or cholinesterase inhibitors). The
dose and medication schedule should be the
minimal amount needed to provide maximal
improvement in the vital muscles of swallowing
and breathing. Side effects of the drug, called
cholinergic reactions, result from excess
acetylcholine.
Rational.1591= Sepsis is characterized by
hypotension and altered tissue perfusion. Sepsis
stems from toxins produced by microorganisms.
Once the organism enters the body, a set of
complex humoral, cellular, and biochemical
mediators are released causing epithelial damage,
peripheral vasodilation, and increased capillary
permeability. Secondarily, the patient would
become acidotic with decreasing pH and PaO with
Rational.1590=
nurse would
suspect
subscript((2))
levels,The
followed
eventually
by loss
Guillain-Barrofsyndrome,
which is an acute,
consciousness.
rapidly progressive inflammation and
demyelination of nerve endings of the peripheral
nervous system that predominantly affects motor
funtion. Sensory and motor loss usually occur
rapidly in an ascending order. Current research
reports that 60-70% of patients with Guillain-Barr
syndrome report a mild febrile illness, usually
respiratory or gastrointestinal, 1-3 weeks before
onset of symptoms. Trauma to the head or spinal
cord may cause paralysis below the area of the
injury.
Rational.1589= Chemotherapy may cause
hyperuricemia due to rapid tumor destruction.
Allopurinol is administered before and during
therapy to break down urates. This is standard
treatment and is being done to prevent symptoms
of gout.

Rational.1588= Therapeutic radiation uses


ionizing forms of radiation that must produce cell
death to be effective. This type of radiation uses
high-energy beams to treat malignant cells. It is
not the same as x-rays. There are side effects
associated with radiation therapy and the patient
will be informed about those.

Question.1587= A compensated respiratory


acidosis would demonstrate which two blood gas
findings?

Rational.1587= The body compensates for


respiratory acidosis by increasing the HCO3 and
the PaO2.

Question.1580= Following a gunshot wound to the


abdomen, a 27-year-old male has a complete
colectomy with creation of an ileostomy. Nursing
measures that will be necessary for the patient,
considering the fact that the function of the large
intestine has been eliminated, will include

Rational.1580= Complications following an


ileostomy include fluid and electrolyte imbalance
therefore, accurate fluid intake and output records
must be maintained at all times. An intact stomach
and small intestine means that the patient does not
need tube feedings, enzymes, or emulsifying
agents.

Question.1577= As the nurse is orienting a new


Rational.1577= This patient has elevated
nurse to the unit, and she asks about treatment the creatinine levels (normal is 0.5-1.5 mg/dL). Serum
patient is receiving for an infection. The new nurse
creatinine levels do not elevate until half of the
tells the nurse she remembers being taught that
nephrons are not functioning. patients with
aminoglycosides (the patient is receiving
elevated creatinine are likely to have severe renal
gentamicin) can cause renal failure. The patient
impairment. BUN values vary widely according to
has a normal urinary output and the following lab the dietary intake of protein, but values elevated to
values: Na 142, K 4.6, Cl 103, HCO3 21,
more than 20 mg/dL suggest renal insufficiency.
creatinine 3.2, BUN 54. She asks if this patient is
Renal output is important but the creatinine level
at risk for developing renal failure. The nurse
denotes renal impairment (1). The normal range
would answer
for creatinine is 0.5-1.5 mg/dL (2).

Question.1574= To assess third cranial nerve


function, the nurse would test the patient`s

Rational.1574= The third cranial nerve governs


oculomotor responses. This includes pupil
response to light. Eye movement is controlled by
Cranial Nerve IV: Trochlear (2). Cranial Nerve I
controls olfactory function: smell (3). Cranial
Nerve VIII controls vestibulocochlear function
(acoustics or hearing) (4).

Question.1573= A 41-year-old female is admitted


to the unit with an exacerbation of chronic
lymphocytic leukemia. She states that she has had
small amounts of vaginal bleeding. Ecchymotic
areas are noted on her arms and legs. Laboratory
data reveal the following:

Rational.1573= A normal platelet count is


15,000-45,000/mm3. A low or below normal count
places the patient at risk for bleeding. A normal
WBC count is 3400-10,000/mm3. patients with a
low or below normal count are at risk for infection.
The other choices are partly correct, but the patient
needs both protective isolation and bleeding
precautions.

Rational.1572= Hypertonic solutions will decrease


edema because they will pull water out of the cells
by osmosis. The hypertonic solution has no
Question.1572= The rationale for using hypertonic
nutritional value (1). The goal of fluid replacement
solutions in burn resuscitation is
in burn resuscitation is to maintain a serum sodium
concentration of 140 Eq/L
Question.1571= A 72-year-old female is admitted
to the unit following a fall at home. Her daughter
Rational.1571= Left-sided weakness (left
explains that her mother attempted to stand after
hemiparesia) or paralysis (hemiplegia) indicates a
dinner and immediately fell. Currently she is
stroke involving the right cerebral hemisphere,
awake but unable to move her left side. She is able
because the motor nerves cross in the medulla
to talk and is alert and oriented. Admission vital
before entering the spinal cord and periphery. A
signs are blood pressure 176/100, pulse 62,
left-sided CVA would cause right hemiparesis or
respiratory rate 16, temperature 36.8 degrees C.
hemiparalysis (1). Hemianopsia is blindness in
The patient`s pupils are equal and reactive and her half the visual field, resulting from damage to the
eye movements are normal. The patient states that
optic nerve (2). Hypertonia (spastic paralysis)
she has been healthy and has never needed to see a
tends to cause fixed positions or contractures of
doctor. Based on the preceding information, the
extremities (4).
nurse would continue assessing for

Question.1570= Brudzinski`s sign is best


described by which of the following definitions?

Question.1569= The critical care nurse should


recognize that a major complication of diabetes
insipidus could include

Question.1567= If a patient develops a grand mal


(tonic-clonic) seizure, the initial nursing action is
to

Rational.1570= Brudzinski`s sign is positive when


the nurse gently flexes the patient`s head and neck
to the chest and flexion of the hips and knees
results. This is an indicator of meningitis.
Rational.1569= Diabetes insipidus is a condition
characterized by impaired renal conservation of
water caused by a deficit of ADH. This causes the
excretion of large volumes of diluted urine,
leading to dehydration. Serum sodium levels
increase (not decrease), due to decreased or absent
ACTH, if water replacement does not occur (2).
Serum osmolarity is increased, due to a decrease in
ADH and increased water loss (3). Due to
Rational.1567=
The first priority
protect
the
dehydration, intravascular
volume is
is to
low
therefore,
patient
from
injury
resulting
from
environmental
hypotension occurs. Tachycardia results as a
hazards, compensatory
such as siderails,
falls, and
response
(4).clothing.
Forcing an airway into the patient`s mouth after a
seizure has begun should never be attempted. This
is likely to injure the patient`s teeth and risk
aspiration on tooth fragments (1). Oxygen therapy
may be used, but is not the initial priority even
though cyanosis is common in a grand mal seizure.
It generally resolves without treatment (3). A
padded tongue blade should never be inserted after
a tonic-clonic seizure has begun, because it may
injure the patient`s teeth, and tooth fragments may
cause aspiration (4).

Rational.1566= Kernig`s sign is positive when a


patient is unable to extend his or her leg when the
Question.1566= Which of the following best
thigh
is flexed to Guillain-Barr?
the abdomen. This
is a positive
describes Kernig`s sign?
Rational.1565=
syndrome
is an
assessmentprocess
for meningitis.
acute inflammatory
characterized by
varying degrees of motor weakness and paralysis,
plus sensory and motor disturbances that occur in
an ascending, distal to proximal pattern.
Question.1565= A 36-year-old male is admitted to
Respiratory compromise is common. The cause is
the nursing unit with rapidly increasing symptoms
obscure, but evidence indicates a cell-mediated
of generalized weakness following an episode of
immunological reaction. Myasthenia gravis is a
"flu." He noticed that the weakness started in his
demonstration of progressive paresis that is
arms and legs and has progressed to his upper legs,
resolved, in part, by rest. The most common
abdomen, and chest. He has difficulty taking a
symptoms involve extraocular muscles of the eyes
deep breath. Vital signs are normal and he has
(2). Multiple sclerosis is a degenerative disease
some complaints of shortness of breath. Based on
manifested by increased fatigue, stiffness of the
the preceding symptoms, which condition is likely
extremities, and flexor spasms (3). ALS
to be developing?
(amyotrophic lateral sclerosis) is a progressive
degenerative disease characterized by fatigue
while talking, tongue atrophy, dysphagia,
dysarthria, fasciculations of the face, and weakness
Question.1564= The nurse, assessing a patient
of the arms and hands (4).
Bradycardia and hypertension.
with head injuries monitors for signs of increasing
ICP (intracranial pressure). These signs would be

Question.1562= Assessing for hypokalemia, the


nurse will observe for ECG changes of

Rational.1562= The cardiovascular effects of


hypokalemia include dysrythmias (PVCs),
inverted T waves, peaked P waves, and prolonged
QT interval. The other choices are incorrect. A
narrowed QT interval is indicative of hypokalemia
(4).

Question.1560= Trousseau`s sign is a test for the


electrolyte deficiency of

Rational.1560= Tetany resulting from a decrease


in serum calcium can be confirmed by a positive
Trousseau`s sign. Hyperphosphatemia may be
present with a positive Trousseau`s sign (1).
Hypercalcemia produces lethargy and weakness,
not signs of carpal spasm seen with Trousseau`s

Question.1559= A nurse in an extended care


facility is caring for a patient with a cuffed
endotracheal tube. The nurse understands that the
purpose of inflating the cuff is to

Rational.1559= Inflating the cuff creates a seal


between the tube and the patient`s trachea. This
forces air exchange to take place only through the
ET tube. Inflating the cuff on the ET does not
make suctioning easier (1). The percentage of
oxygen remains the same after the cuff is inflated

Question.1557= A patient is admitted to the


intensive care unit after sustaining a knife wound
to the back. Assessment findings include loss of
pain and temperature on the right side and loss of
motor function on the left. Vital signs are stable
and he is alert and oriented. No other injuries are
noted. Based on this information, which type of
neurological syndrome is likely to be developing?

Brown-Squard syndrome is caused by


hemisection of the spinal cord. Clinical
manifestations include paralysis below the level of
injury on the same side of the lesion and the
perception of pain and temperature is affected
below the level of injury on the opposite side of
the lesion. Central cord syndrome is not a known
disease process

Question.1556= The primary symptoms present in


cases of autonomic hyperreflexia are

Rational.1556= Autonomic hyperreflexia occurs in


patients with a spinal cord lesion at or above T6.
The response occurs as a result of a noxious
stimulus below the level of the lesion, causing
visceral lesion activity resulting from massive
sympathetic response. Symptoms include
hypertension, bradycardia, and profuse sweating
above the level of the injury. Autonomic
hyperreflexion includes bradycardia and
hypertension

Question.1554= For a patient with C-3-4 injury,


the nurse should first assess

Rational.1554= In spinal cord injuries that occur


above C-4, the muscles responsible for respiration
are paralyzed. Heart rate, motor ability, and
temperature may be important, but the priority is
respiration.

Question.1553= Guillain-Barr syndrome affects


the neurological component of the

Rational.1553= Guillain-Barr syndrome is an


acute, rapidly progressive inflammation and
demyelination of nerve endings of the peripheral
nervous system.

Rational.1551= In decerebrate posturing, the


patient has a rigid and possibly arched spine,
rigidly extended and pronated arms with the wrists
Question.1551= When assessing a patient on the
flexed and the palms facing backward, and
neurological unit, the nurse knows that decerebrate extended legs with plantar flexion. An abnormal
posturing is characterized by
flexion response is decortication (2). Decerbrate
posturing does not include hyperflexion of lower
extremities (3). Decerebrate posturing is a motor
response (4).

Question.1549= When auscultating a patient`s


lungs, which type of adventitious lung sounds is
the nurse most likely to hear with left ventricular
failure?

Rational.1549= Physical signs associated with left


ventricular failure include bilateral basilar rales.
Wheezing is associated with asthmatic or airway
restrictive disease (1). Tinkling sounds aren`t a
characteristic of left ventricular failure (2).
Increased respiratory breath sounds in the apex is
associated with lung collapse (4).

Question.1548= What is the dominant effect of


ADH (antidiuretic hormone) on the kidneys?

Rational.1548= ADH regulates the osmolarity of


extracellular fluids by increasing reabsorption of
water from the renal tubules.

Question.1547= A 59-year-old female with a


history of alcoholism is admitted to the unit after
being found unresponsive. During the next 24
hours, she develops decreased blood flow to her
distal extremities, manifested by discoloration of
her hands and feet. The physician believes that
DIC (disseminated intravascular clotting) may be
taking place. The test that will help confirm that
DIC is present is the

Rational.1547= A measurement of fibrinogen


degradation produces greater than 45 ug/mL which
helps to confirm DIC. This increased level (normal
= 10 ug/mL) indicates that intravascular clots have
formed and the body is attempting to dissolve
them to restore microcirculation.

Question.1546= A physician orders nitroprusside


and dobutamine for a patient with CHF
(congestive heart failure). The goal of this type of
medication regimen is to

Rational.1546= Nitroprusside is a vasodilator


which dilates arteries and veins. Arterial relaxation
reduces afterload, making it easier for the left
ventricle to eject blood. Dobutamine is a synthetic
catlecholamine that is primarily a beta receptor
stimulator. Dobutamine increases the force of
contraction by as much as 30-70%. This drug
combination reduces afterload and improves
contractility

Question.1544= The priority nursing goal for the


patient in status epilepticus is to

Rational.1544= When a patient is in status


epilepticus, follow the ABC (airway, breathing,
circulation) after calling for help. Do not leave the
patient, maintain a patent airway, and administer
oxygen via nasal cannula. Medications are given
after the airway and breathing are stabilized.

Question.1541= The nurse is caring for a


37-year-old female admitted with an intracerebral
bleed. The nurse on the preceding shift says that a
neurological examination was performed on the
patient and that she had an abnormal "doll`s eyes"
test. Which of the following descriptions best
describes an abnormal oculocephalic response to
the "doll`s eyes" test?

Answer4.1541= The eyes follow the direction of a


quick turn of the head.
= An injury that stimulates the neurons of the
cortical gaze center causes the patient`s gaze to
remain fixed in the direction of head movement.
This is called a positive "doll`s eyes" test. Normal
response is for eyes to move in the opposite
direction of head movement.

Question.1540= Cheyne-Stokes breathing is


characterized by which of the following
respiratory patterns?

Answer4.1540= Short periods of apnea followed


by respirations of increasing depth that then slow
again to apnea.

Question.1539= The primary function of the


cerebellum includes which of the following?

Rational.1539= The cerebellum plays an essential


role in modifying the force of contractions of the
muscles primarily responsible for movement and
the relaxation of muscles opposing movement.
This role is essential for smooth, coordinated
movements. The cerebellum also detects loss of
balance and restores it by modifying muscle
contraction. Thought and personality are higher
functions and occur in the frontal lobes. Sight is
controlled in the occipital lobe.

Question.1538= Following a head injury sustained


in an auto accident, the patient is admitted. The
nurse is assessing the patient`s level of
consciousness, based on her knowledge that
maintenance of an awake and alert status is
dependent on the proper functioning of which two
cerebral structures?

Rational.1538= Consciousness relies on the active


function of the cerebral cortex. This is maintained
by continuous stimulation of the cerebral cortex by
nerve impulses from a series of nuclei in the brain
stem that together are called the reticular
activating system.

Question.1537= A patient in your unit has suffered


frontal head injuries from a motor vehicle
accident. Which type of impairment may result
from injury to the frontal lobe?

Rational.1537= The frontal lobes make up


approximately one-third of the mass of the
cerebral hemispheres and are thought to be
concerned with the highest cognitive and
intellectual functioning, personality, and motor
control. Sensation is controlled by the parietal
lobes, vision in the occipital lobe, and hearing in
the temporal lobes.

Question.1536= A 20-year-old male is admitted to


the unit following a suicide attempt after breaking
up with his girlfriend. He ingested an unknown
drug or drugs and is currently combative but with
a reduced level of consciousness. A large-bore
nasogastric tube has been inserted in an attempt to
lavage his stomach. Which of the following
nursing actions should be initiated at this point?

Rational.1536= patients at greatest risk for


aspiration during gastric lavage are those who
have an altered level of consciousness, are
confused, are debilitated, or have impaired gag
reflexes. Initiation of mechanical ventilation or
sedation are physician actions. Maintenance of an
airway takes priority over other possible nursing
actions.

Question.1534= Asterixis is regarded as a sign of


the developing condition of

Rational.1534= Asterixis is the irregular flapping


movements of the fingers and wrists when the
hands are outstretched, with the palms down,
wrists bent up, and fingers spread. Jerking muscle
tremors are also seen in the feet and tongue. Also
called liver flap or hepatic tremor, this is the most
common and reliable sign that hepatic
encephalopathy is developing. Calcium
disturbances and cardiac problems are not
manifested in asterixis.

Question.1532= A patient with a C-6 fracture


would most likely be able to perform which of the
following movements?

Rational.1532= Spinal nerves originating in the


cervical disks 5 through 8 control the brachial
plexus. C-6 function involves good shoulder
control, wrist extension, and supinators. Injury
above C-5 would have additional impairment,
while injury below C-6 would determine how
much waist or leg movement was possible.

Question.1531= A 51-year-old female is admitted


with hypotension, bradycardia, and decreased level
Rational.1531= Myxedema coma is a complication
of consciousness. Her core temperature is
associated with hypothyroidism. Signs and
35.5degree C. No history is available regarding
symptoms include hypothermia, seizures, and
prior medical problems. She appears to be
exacerbations of hypothyroidism. Common CHF
overweight, with dry, scaly skin and puffy face
symptoms are rales, increased respirations, and
and lips. Shortly after admission, she has a grand
tachycardia. ARDS presents with wheezing and
mal seizure. She is intubated and placed on
tachycardia. Thyroid crisis has increased heart rate
mechanical ventilation. Based on the preceding
and respirations.
information, which condition is likely to be
developing?

Question.1530= A patient has just been admitted


to the unit following an accident in which he
sustained an injury to the temporal lobe. The
nurse`s assessment will focus on disturbances in
the patient`s

Rational.1530= The temporal lobes are located on


the lateral aspects of the cerebral hemispheres and
are primarily concerned with the perception of
verbal material: auditory-receptive and hearing.
Spatial orientation and taste arise from the parietal
lobes. The occipital lobe is associated with vision.

Question.1529= A 48-year-old female is admitted


to the unit. Her diagnosis is a possible syncopal
episode. She is currently awake, nervous, and
anxious. Vital signs are as follows: blood pressure
178/108 pulse 129 respiratory rate 28 temperature
39degree C (102degree F) During the initial
examination, the nurse notes that she has
exophthalmos and that her skin is warm and wet.
Given the preceding information, the nurse will
continue assessment for the possible condition of

Rational.1529= Thyroid storm is an extreme


exacerbation of severe hyperthyroisism. The
metabolic rate increases without regard for any
body system. Body temperature will rise from as
high as 102degree F (39degree C) to as high as
106degree F (41degree C). Severe tachycardia,
hypertension, and tachypnea also are present.
Myxedema would present with opposite
symptoms.

Question.1525= The nurse is analyzing data


collected from a patient in the first 24 hours post
acute myocardial infarction (AMI). Which of these
data suggests the onset of cardiogenic shock?

Rational.1525= Extremely close monitoring of a


post-MI patient`s vital signs is indicated, as
hypotension is a cardinal indicator of impending
cardiogenic shock.

Question.1522= A patient has a chest catheter


connected to a closed water seal drainage system.
Which nursing action should be included in the
plan of care?

Rational.1522= The documention of the amount,


color, and consistency of the drainage every shift
by the nurse should be included in the plan of care.
Any significant change in the amount, color, or
consistency is reported to the physician. Stripping
chest tubes is now contraindicated and should not
be included in the plan of care. Continuous
bubbling in the water seal chamber indicates an air
leak and must be evaluated to maintain proper
function of the system. The collection chamber
must be kept below chest level to function
properly.

Question.1521= A patient with a closed head


injury should be monitored for signs of increased
intracranial pressure. A clinical manifestation of
this condition would be

Rational.1521= Increasing systolic blood pressure


with widening pulse pressure, bradycardia, and
respiratory slowing are part of Cushing`s Triad, a
hallmark of increased intracranial pressure

Question.1520= The assessment finding that


indicates a flail chest is

Rational.1520= A flail chest usually occurs with


nonpenetrating trauma and produces a paradoxical
motion of the damaged area so that it moves in a
direction opposite to the motion of the uninvolved
chest wall.

Question.1519= The nurse evaluates a patient`s


arterial blood gases and determines that the patient
has a low plasma bicarbonate level. This condition
is known as

Rational.1519= Metabolic acidosis occurs when


there is either an increase in acids or a deficit in
the base (bicarbonate). Acid-base imbalances that
are respiratory in nature result from changes in the
CO2 levels.

Question.1518= Which of the following common


cardiac medications should the nurse anticipate
administering to a patient with a sudden onset of
extreme bradycardia?

Rational.1518= Atropine decreases AV conduction


time, thus achieving the desired outcomes of
increasing both heart rate and cardiac output in this
patient. Lidocaine is indicated in the management
of ventricular dysrhythmias.

Question.1511= A patient has just been diagnosed


with acute pancreatitis. Hypocalcemia is often
present in acute pancreatitis. Which of the
following lab findings is associated with
hypocalcemia?

Rational.1511= The calcium level is regulated by


parathyroid hormone, which maintains an inverse
relationship between calcium and phosphorous.

Question.1503= Which of the following ostomies


would most need to be irrigated?

Rational.1503= The sigmoid colostomy is most


likely to need irrigation, because the stool is
similar to the consistancy of normal stool. The
other ostomy types have less formed stool.

Question.1498= When assisting a patient from a


bed to a chair for the first time postoperatively, the
nurse would make sure to place the chair

Rational.1498= Placing the chair at the head of the


bed, facing toward the patient`s feet is a good
position for the patient to transfer to and still be
able to observe the activities within the room.,

Question.1496= When planning meals with a


patient who is hemodialyzed three times per week,
which of the following food selections would
indicate the patient understands the dietary
guidelines?

Rational.1496= Apple juice is low in phosphorus,


sodium, potassium, and protein. The patient in
renal failure must avoid these nutrients, because
the damaged kidney cannot clear them. Bananas
are high in potassium. Red meat contains protein
and phosphorus. Legumes contain phosphorus.

Question.1483= The nurse practice act is an


example of which of the following types of law?

Rational.1483= Individual state legislative bodies


enact specific laws for professional practice. Each
state has its own nurse practice act.

Question.1474= The proper position in which to


place a patient immediately postop until her gag
reflex returns is the

Rational.1474= If the gag reflex has not been


regained, place the patient on her side in order to
prevent aspiration until she is able to swallow. The
prone position would allow secretions to pool in
the airway, predisposing the patient to aspiration.

Question.1472= Which of the following signs


would indicate that a patient is having a severe
reaction to the contrast material administered in an
intravenous pyleogram?

Rational.1472= Severe respiratory signs of


reaction to the contrast material include
laryngospasm, cyanosis, laryngeal edema, and
apnea.

Question.1471= An elderly gentlemman patient


has a diagnosis of occlusive arterial disease. When
instructing the patient regarding proper positioning
in bed, the nurse would advise him to

Rational.1471= Allowing the patient to sleep in a


somewhat dependent position may help prevent
pain at rest. Raising the legs would decrease
arterial perfusion and cause more vascular
problems.

Question.1467= When assessing the lower


extremities of a patient with arterial insufficiency,
the nurse notices that they are reddish-blue in color
below the knees. This observation will be recorded
as

Rational.1467= Rubor is a reddish-blue color


caused by superficial capillaries that remain
dilated after injury. Cyanosis is a blue-gray color,
pallor is a white color, and striae are lines that are
red, then a silvery tone over time.

Question.1465= A female patient, age 57, is being


evaluated in the GU clinic. Her assessment reveals
paradoxical incontinence, which can be described
as

Rational.1465= Overflow or paradoxical


incontinence occurs with the leaking of small
amounts of urine from a distended bladder in the
absence of effective contractions. Urinary
hesitancy is not associated with paradoxical
incontinence. A decrease in the stream is
associated with benign prostate hypertrophy and
frequent urination is associated with an infection.

Question.1463= A patient diagnosed with


congestive heart failure (CHF) is noted to have
gained 4 1/2 pounds in the past 24 hours. When
considering the patient is retaining fluid, the nurse
would note this weight gain is consistent with

Rational.1463= The amount of fluid retained can


be determined by the weight gain. The accepted
ratio, one liter, is equivalent to one kilogram or 2.2
pounds. This patient had gained 4 1/2 pounds, or
roughly two liters of fluid.

Question.1454= To assist the left CVA patient out


of bed, the nurse would place the wheelchair

Answer1.1454= Facing the patient`s left side.

Question.1451= The proper temperature for


administration of a tepid sponge bath is

90 degrees F.

Question.1447= A 77-year-old woman suffered a


left-sided CVA, resulting in a right hemiparesis.
To preserve her skin integrity, the nursing care
plan would include

Rational.1447= The need to encourage


high-protein foods is essential for proper
maintenance of skin. A lack of protein leads to
promotion of skin breakdown and a lack of healing
ability. The position should be changed at least
every two hours. The pressure areas should be
massaged with each turn, at least every two hours.
The daily shower would not be recommended for a
person of this age because it would tend to dry
skin, leading to potential cracking and breaking.

Question.1444= The nurse explains to the patient


receiving pertoneal dialysis that when the dialysate
is removed after the initial peritoneal dialysis
exchange, it often appears

Rational.1444= The dialysate should be clear and


pale yellow, although after the first few initial
exchanges it may appear bloody or pink tinged.
Any cloudiness or green dialysate may indicate
infection.

Question.1443= To eliminate hyperglycemia in the


patient who has diabetes and is on peritoneal
dialysis, the nurse would

Rational.1443= Diabetics may become


hyperglycemic from the dextrose in the dialysate.
Insulin can be added to the dialysate, thereby
eliminating the need to give the patient
subcutaneous insulin when performing
dialysis.The dialysate solution must contain
dextrose (sugar) in order to create an osmotic
gradient. Blood sugar would be tested as
symptoms appeared.

Question.1439= When assessing an adult`s ECG,


which of the following findings would the nurse
recognize as being consistent with an MI?

Rational.1439= The area immediately surrounding


the infarcted areas has injured cells and is called
the zone of injury it produces ST segment
elevations. An increase in the PR interval indicates
a conduction delay between the SA and AV nodes.
A widened QRS complex indicates a conduction
delay in the ventricles and may be indicative of
ventricular dysrhythmias. U waves are associated
with a disturbance in the electrolyte potassium.

Question.1433= A patient with Type 1 Diabetes


Mellitus is given a stat dose of insulin because his
blood sugar is 525 mg/dL. After receiving the
insulin, the patient begins to demonstrate
dysrhythmias on the ECG monitor. Which of the
following disorders would the nurse consider to be
the MOST likely cause of this cardiac irregularity?

Rational.1433= The administration of regular


insulin tends to drive the potassium from the
extracellular fluid into the intracellular fluid,
causing a loss of serum potassium. Decreased
serum potassium can cause serious dysrhythmias.

Question.1432= The nurse is completing an


admission assessment on a 67-year-old male
patient who has a long-standing history of
emphysema. Upon percussion of the lungs, the
nurse expects to hear

Rational.1432= Hyperresonance is a very loud,


low-pitched, boomlike sound produced in air-filled
spaces, such as emphysematous lungs. Dullness is
a medium-pitched, thudlike sound heard over a
solid structure, such as an organ. Resonance is a
loud, low-pitched, hollow sound normally heard
over healthy lungs. Tympany is a loud,
high-pitched, drumlike sound normally heard over
a gastric air bubble.

Question.1429= The staff is developing a care plan


for a patient with hypercalcemia. An action the
nursing care plan will include to improve urinary
problems is to

Rational.1429= Increased fluids will decrease the


incidence of infection and stones both are less
likely to develop if the urine is dilute.

Question.1428= Which of the following diets is


most likely to be recommended for a patient
experiencing hypercalcemia?

Rational.1428= An acid-ash diet will decrease the


risk of urinary tract infection and renal stone
development because it helps acidify urine.

Question.1424= Which of the following signs


would the nurse assess for in a patient with a
suspected diagnosis of Lyme disease?

Rational.1424= Early signs and symptoms of this


disorder may include distinctive flat or raised red
lesions, lesions that are hot to the touch, fever,
malaise, fatigue, headache, chills, nausea, and
lymph node enlargements.

Question.1422= Confabulating and lack of voice


control in a patient with a CVA can be attributed
to

Rational.1422= A right-hemisphere-damaged
CVA patient lacks voice control, talks incessantly,
confabulates, and reads aloud fluently without
comprehension.

Question.1415= The nurse in the PACU notes a


change in the patient`s respiratory rate from 18 to
36 respirations per minute. The nurse`s first action
would be to

Rational.1415= If the patient has any difficulty in


breathing or exhibits a change in the pattern of
breathing, notify the anesthesiologist immediately.

Question.1414= A patient has just been informed


that she will be started on a CAPD protocol. She
wants to know more about CAPD and asks the
nurse how many exchanges she would have to
perform per day. The nurse explains that

Rational.1414= CAPD usually involves 3 to 4


exchanges per day of 2 liters each in a 24-hour
period, with dwell times of 4 to 6 hours each. Most
patients who need dialysis can use CAPD.

Question.1411= The wife of a patient on dialysis


asks the nurse how chronic renal failure causes
anemia. The nurse explains that

Rational.1411= Anemia in CRF occurs as a result


of decreasing erythropoietin and because the
uremic environment decreases the life span of the
RBC.

Question.1406= A patient has undergone a


transspheniodal hypophysectomy. Procedures
contraindicated in the immediate postoperative
period include

Rational.1406= Frequent mouth care should be


given, but brushing teeth may damage the gingival
suture line. Elevating the head of the bed promotes
an open airway. Advancing the diet and
monitoring for infection are routine nursing
responsibilities.

Question.1403= The nurse is teaching the wife of a


Rational.1403= Propulsive gaits are usually
patient with Parkinson`s disease. She explains that
exhibited in later disease progression. The
patients with Parkinson`s have difficulty slowing
patient`s steps become faster and faster and shorter
down once they walk forward, and in fact, they
and shorter.
actually speed up. This problem is described as a

Question.1397= A male patient has been admitted


for treatment of end-stage renal disease. The nurse
understands that the primary cause of renal failure
is

Rational.1397= Diabetes mellitus is the most


frequent cause of renal failure, followed by
hypertension.

Question.1396= The nurse plans to prevent


complications in the diuretic stage of a patient`s
renal failure. Information that the nurse will use in
developing this plan includes recognizing that

Rational.1396= Although renal healing is


occurring during this stage, the tubule system is
frequently unable to concentrate urine
satisfactorily. Thus, patients may excrete large
volumes of urine (but not 25 to 30 liters in a day)
and lose substantial amounts of electrolytes.

Question.1395= Which of the following disorders


would be of greatest concern for a patient in the
oliguric stage of acute renal failure?

Rational.1395= Acute uremia results in fluid


retention and electrolyte abnormalities,
particularly hyperkalemia.

Question.1392= Which of the following would the


nurse expect to find on the urinalysis report of a
patient who has a diagnosis of
hyperparathyroidism?

Rational.1392= As a result of hypercalciuria,


hyperphosphaturia develops and altered renal
tubule function causes unusually alkaline urine.
Ketones and glucose are associated with diabetes
mellitus. RBCs are associated with infection or
trauma.

Question.1390= A female patient, age 49, has been


admitted to the hospital with an exacerbation of
rheumatoid arthritis. The nurse understands that
this disease primarily affects adults between 20
and 55 years old and the incidence is

Rational.1390= Rheumatoid arthritis affects


women three times as often as men. It is an
autoimmune disease that is not genetically linked
and is not as common in children as in adults.

Question.1388= When percussing a patient`s


abdomen, the nurse knows that fluid will be
revealed as a

Rational.1388= Fluid in the abdomen will pool in


the flanks and will percuss with a note that is flat
or dull. Tympany or hyperresonant sounds occur
over air. A bruit is a swirling sound.

Question.1387= A patient is diagnosed with a


left-sided heart failure. One major goal in the
management of congestive heart failure is

Rational.1387= Afterload is a major concern in


CHF. Ways to reduce it include the administration
of vasodilating agents. These agents decrease
peripheral vascular resistance (afterload) and
increase cardiac output by reducing resistance.
Contractility is thereby increased. Relief of anxiety
is not an immediate goal of treatment.

Question.1386= To assess the peripheral pulses of


a patient`s lower extremities, which of the
following areas would the nurse palpate?

Answer1.1386= The popliteal, dorsalis pedis, and


posterior tibial.

Question.1385= To confirm the diagnosis of


left-sided CHF, the patient will undergo a cardiac
catheterization. Which of the following
preprocedure orders would the nurse question?

Rational.1385= Sodium should be restricted. A


reduction of sodium will decrease the retention of
salt and water and therefore reduce vascular
volume.

Question.1378= Which of the following


interventions may cause injury in a patient with
rheumatoid arthritis experiencing a flare-up
(exacerbation) of the disease?

Rational.1378= During periods of exacerbation, or


acute inflammation, active range of motion
(especially to the point of pain) can cause
significant joint damage. Consequently, range of
motion should be done by the nurse but not to pain
or beyond. Splinting of joints and medication are
useful in treating exacerabations of RA.

Rational.1374= Aphasia, both expressive and


Question.1374= The aphasia seen in CVA patients
receptive, is seen in patients who have sustained
is usually caused by damage to the
damage to the left cerebral hemisphere. Damage to
the cerebellum usually results in motor deficits.
Question.1371= A 34-year-old female is in the
critical care unit with acute respiratory distress
secondary to sepsis following a motor vehicle
accident. She currently has a chest tube in place,
set at 20 cm H2O suction. There is no bubbling in
the water seal although there is bubbling in the
Rational.1371= Fluctuations in the water seal
show that there is patency between the pleural
suction control chamber. While the nurse was at
lunch, one of the unit technicians says he clamped cavity and drainage bottle. Fluctuations will stop if
the chest tube to determine if the suction level was lung expansion has occurred or there is kinking of
the tube or clots present. Listening to the lungs
still at 20 cm H2O. It is still clamped when the
would indicate if expansion has occurred, in which
nurse goes into the room. The patient does not
case no x-ray will be necessary.
complain of any symptom change. About the same
time, the patient`s physician comes into the room
and notices the clamped tube. He becomes very
upset and says to get an immediate chest x-ray to
see if a tension pneumothorax has occurred. The
Rational.1366= For traction to be successful,
nursing action is to
Question.1366= When evaluating the effectiveness countertraction must be present. Weights resting
of traction, the finding that would counteract this on the bed would not allow for traction. Force and
direction must be maintained with the ropes in the
is
center track of the pulley.

Question.1365= When assessing the breath of a


Rational.1365= Fruity or acetone odor is indicative
patient with advanced liver disease, the nurse notes of ketosis, whereas an ammonia smell is associated
an ammonia-like odor. This most often represents
with uremia.

Question.1361= A young patient is diagnosed with


mitral stenosis. When assessing the patient, which
symptoms are consistent with this condition?

Rational.1361= Mitral stenosis refers to the


obstruction of the mitral orifice due to adhesions
between the two mitral valve leaflets. The first
symptom observed is exertional dyspnea.

Rational.1358= Neurovascular assessment of a


casted extremity involves color, sensation, and
Question.1358= Which of the following
movement. Tingling would indicate a problem.
assessments would indicate possible neurovascular
Normal assessment would expect that skin is warm
compromise in a casted extremity?
and pink with equally palpable pulses. Mild pain
on passive motion is expected.
Question.1357= A female, age 45, is admitted to a
cardiac medical unit for preoperative care prior to
having a possible valve commissurotomy.
Pertinent past history includes her having had a
case of rheumatic fever when she was 12. She is
married and has two children aged 17 and 13. The
patient seems very anxious and asks the nurse,
"How could a sore throat I had 30 years ago
damage my heart now?" The best reply by the
nurse is

Rational.1357= Rheumatic fever is a systemic,


inflammatory, nonsuppurative disorder that
usually occurs as a sequela to a throat infection
caused by group A beta-hemoloytic streptococci. It
is characterized by a diffuse proliferative and
exudated inflammatory reaction in connective
tissues, particularly of the heart, joints, and skin.

Question.1353= A patient diagnosed with a


hypothalamic disorder is being admitted. In her
assessment, the nurse will question the patient
about disturbances in

Rational.1353= The hormones of the hypothalmus


control many important functions. One of the
major functions is temperature control

Question.1352= The nurse is caring for a patient


undergoing peritoneal dialysis and notes that the
dialysate has not drained. To help facilitate
drainage, she will

Rational.1352= Draining problems may occur in


peritoneal dialysis due to obstruction of the
catheter by the omentum. If this occurs, turning the
patient from side to side will change the position
of the catheter.

Question.1347= The nurse continues teaching the


patient with AIDS that the agent used to treat
pneumocystis carinii pneumonia is

Septra (Bactrim).

Question.1346= The AIDS clinic nurse is teaching Rational.1346= T4 lymphocytes, known as "helper
a patient about his lab profile. She explains that the cells," are vital in activating the immune response.
cell known as the "helper cell," vital in activating
T8 lymphocytes are suppressor cells and
the immune response, is the
neutrophils are leukocytes.

Question.1344= The nurse is checking the


patient`s pleural drainage system. She recognizes
indication of an active pleural leak when she sees

Rational.1344= Constant bubbling in the water


seal indicates leakage of air in the drainage system
fluctuation is normal with respiration. Bubbling is
expected in the suction control chamber.

Question.1338= In the patient with smoke


inhalation, the most likely time period during
which pulmonary complications may develop is
the

Rational.1338= Any patient with smoke inhalation


must be observed for at least 24 hours for
respiratory complications. It may take a full 24
hours for complications to develop.

Question.1334= A 19-year-old African-American


woman presents with a nontender, unilateral breast
lump in the right breast, which she noticed on a
breast self-exam. Her last menstrual period was
normal and ended 2 days ago. She denies pain,
nipple discharge, or recent trauma. The most likely
diagnosis is fibrocystic breast changes versus

Rational.1334= A mass in an adolescent is almost


pathognomonic for fibroadenoma, especially in
African-Americans. If breast symptoms are
unilateral or there is a palpable mass, refer the
patient for further evaluation. Women with
fibrocystic changes usually present with bilateral
nodularity and increased tenderness or pain prior
to menses. The nodularity may be generalized or
localized in the upper outer region.

Question.1325= A pregnant woman who is not


immune to rubella should be vaccinated

Rational.1325= To reduce the risk of acquiring


rubella at the beginning of a future pregnancy, the
woman should receive the vaccine after she gives
birth. Caution her to avoid becoming pregnant for
at least 3 months after receiving the vaccine.

Question.1324= The nursing plan for an infant


diagnosed with necrotizing enterocolitis (INEC)
should include

Rational.1324= Measuring the abdominal girth


provides information on whether or not the gastric
distention is resolving.

Question.1323= At what age do term infants


usually regain their birth weight after the initial
weight loss?

10 days.

Question.1320= When a newborn cries, the right


side of his face remains motionless. When he
sleeps, his right eyelid remains open. The nurse
should interpret this as a

Question.1319= In order to prevent the need for a


backup method of contraception, the nurse
instructs the patient starting oral contraceptives to
take the first pill

Rational.1320= A facial nerve paralysis results


from pressure on the facial nerve during labor or
from the use of forceps for delivery. The paralysis
is usually unilateral and most apparent when the
newborn cries. If the nerve has been injured, the
paralysis disappears within a few days or weeks. It
may take as long as several months. If the nerve
has been torn, it requires neuroplasty. Facial nerve
paralysis is an alteration in expected findings of
the newborn assessment. It is neither a congenital
anomaly
a genetic
defect. method
Rational.1319=
Nonor
backup
contraceptive
is needed if the OCs are begun on the first day of
menses since ovulation is inhibited for that cycle.
The pill works primarily by suppressing ovulation
(via suppressing release of gonadotropin-releasing
hormone [GnRH], which inhibits the pituitary
release of FSH and LH), causing thick cervical
mucus (which makes it difficult for sperm to
penetrate), and making endometrium or lining of
the uterus unfavorable for implantation.

Rational.1318= Assessment of pregnancy history


is accomplished using the pneumonic GTPAL G
(gravida) T (term pregnancy) P (preterm birth) A
Question.1318= In addition to the current
(abortion) L (number of living children). Since
pregnancy Sandra has been pregnant three times.
Sandra has been pregnant 4 times, she is gravida 4
She lost the first pregnancy at 10 weeks; the
. She delivered her last pregnancy at term, making
second pregnancy produced twin girls who were
1 term pregnancy. She had 1 birth (delivery) of
born prematurely; and the last pregnancy was
twins, making preterm (P) births equal to 1. Her
carried to term and her son was born at 39 weeks.
first pregnancy was a miscarriage giving her 1
All of the children are alive and well. Which of the
(spontaneous) abortion. All of her children are
following numeric scores accurately depicts
currently living (twin girls and a son). In addition,
Sandra`s reproductive history?
some
pneumonics
add "M" gingivitis
(multiple gestations)
Rational.1316=
Pregnancy
may result
making
the
pneumonic
GTPALM,
in
which
from proliferation of localized blood vesselscase
and
Sandra
wouldAbe
4-1-1-1-3-1.
softening of
the gums.
focal
hypertrophy of the
gums, epulis, may present with erythema,
Question.1316= A patient who is 8 weeks
swelling, and bleeding when traumatized as in
pregnant tells the nurse that her gums have been
teeth brushing. The teeth are not affected and the
bleeding with every tooth brushing. The nursing
hypertrophy resolves after pregnancy as it is most
intervention is to
likely related to elevated estrogen levels. However,
Rational.1315=
signs,
or in
objective
signs,
the frequencyProbable
of bleeding
early
pregnancy
include
those
which
can
be
visualized
on
requires further assessment before considering this
examination
(Chadwick`s
sign, uterine
physiologic
cause.
enlargements, ballottement). A positive pregnancy
test suggests pregnancy by testing for the presence
of HCG. However, the similarity between HCG
Question.1315= Which of the following are
and other hormones (such as LH) may result in
cross-reactions resulting in a positive pregnancy
probable signs of pregnancy that a patient might
experience at some point during her gestation?
test. Therefore, it is considered as a probable sign.
Presumptive signs are subjective signs (including
"morning sickness," amenorrhea, breast
tenderness, and quickening). Audible fetal heart
tones, palpated fetal movements, and visualization
of the fetus on ultrasound are all positive signs of
pregnancy.
Rational.1314= Presumptive signs of pregnancy
Question.1314= A nurse determines that a patient
such as amenorrhea, breast tenderness, nausea,
has several presumptive indicators of pregnancy.
fatigue, and urinary frequency, are subjective and
They are called "presumptive" because they
are not diagnostic, only suggestive of pregnancy.

Question.1313= By the end of 12 weeks the


pregnant uterus should be palpable

Rational.1313= The ascent of the uterus from the


pelvis into the abdomen occurs at approximately
12 weeks. The uterus cannot be palpated when it is
below the level of the symphysis pubis.

Question.1311= A patient who is 12 weeks


pregnant reports that her mouth frequently feels
like she has too much saliva. This condition is
known as

Rational.1311= Ptyalism is the production of


excessive saliva during pregnancy,
characteristically noted during the first trimester.
Oral hyperemia (redness) and gingivitis (gum
inflammation) may occur also. Hyperemesis is
persistent vomiting during pregnancy, which
results in dehydration.

Question.1307= A vaginal exam at eight weeks of


pregnancy would reveal which of the following
signs of pregnancy?

Rational.1307= In the early weeks of pregnancy,


Goodell`s sign (softening of the cervix) and
Chadwick`s sign (a bluish discoloration of the
cervix) both occur. Ballottement occurs in
midpregnancy. The mucus glands form a mucus
plug, but there is also an increase in white vaginal
secretions. However, in addition the mucus
secretions are acidic.

Question.1303= Shortly after entering the room of


a patient with PIH, the nurse observes facial
twitching followed by a tonic contraction of the
patient`s entire body. The nursing intervention is
to

Rational.1303= Eclampsia is marked by the


occurrence of tonic-clonic seizures or vascular
collapse. In the event of seizures, the following
signs are observed: a prodromal facial twitching
lasting only a few seconds, a tonic contraction of
the entire body lasting about 20 seconds, and a
convulsion lasting about a minute. Protecting the
patient from injury is the priority intervention.

Question.1302= A pregnant woman is admitted to


the ER after having a convulsion. The diagnosis of
eclampsia is made based on the presence of

Rational.1302= Eclampsia is marked by


hypertension characterized by the occurrence of
tonic-clonic seizures or vascular collapse.

Question.1301= A patient with mild pre-eclampsia


should be encouraged to

Rational.1301= Left lateral position decreases


pressure on the vena cava and is believed to
improve venous return and placental and renal
perfusion.

Question.1300= A 2-week-old newborn is brought


to the hospital clinic by her mother with a
complaint of continuous regurgitation. A barium
swallow reveals a diagnosis of gastroesophageal
reflux. The nurse will instruct the mother to

Rational.1300= Prone positioning has been shown


to decrease reflux and facilitate gastric emptying.
Some textbooks recommend a prone position with
the head of the bed elevated. Either position is
currently accepted therapy.

Question.1299= The nurse is caring for a


2-week-old infant with a ventricular septal defect.
In planning interventions for the nursing diagnosis
"alteration in nutrition: less than body
requirements related to fatigue," the nurse will
suggest feeding

Rational.1299= A soft nipple helps the infant suck


more easily, decreasing the energy expenditure.

Question.1297= A clinical manifestation of


increased intracranial pressure in infants may be
observed as

Rational.1297= The early signs and symptoms of


increased ICP are often subtle and assume many
patterns, such as irritability and changes in
feeding.

Question.1296= Infants triple their birth weight at


age ___________________ months.

Rational.1296= The answer if 12 months. During


early infancy, the energy requirements for this
period of rapid growth are high. By 1 year of age,
infants weigh approximately 3 times their birth
weight.

Rational.1295= Although the acquisition of gross


Question.1295= Assessing an average 2-month-old
motor skills varies in its timing, infants at 2
months of age can lift their heads up to a 45 degree
child, the nurse knows that he should be able to
angle when prone.

Question.1294= Recommended treatment of


children with sickle cell anemia includes which of
the following?

Rational.1294= Poor splenic function predisposes


the child with sickle cell disease to overwhelming
infection from S. pneumoniae and H. influenzae.
Therefore, recommendations for management
include oral doses of prophylactic penicillin to
protect from infection.

Question.1293= A 2-year-old African-American


child with sickle cell anemia has his
immunizations up to date. Now that he is two,
which of the following vaccines should be
initiated?

Rational.1293= The pneumococcal vaccine should


be administered to all children with sickle cell
anemia at 2 years of age. These children run a high
risk of developing pneumococcal septicemia or
meningitis.

Question.1292= Which of the following is the


most common serious complication of sickle cell
anemia in children?

Rational.1292= The most common serious


complication of children with sickle cell anemia is
overwhelming septicemia and meningitis due to
Streptococcus pneumoniae and Haemophilus
influenzae type B.

Question.1284= A child who is in a chronic state


of hypoxemia would have a physiologic response
of

Rational.1284= In chronic hypoxemia, the body


compensates by producing more red blood cells
(polycythemia). By producing more red blood
cells, the oxygen-carrying capacity of the blood is,
in theory, increased. This is only true if adequate
amounts of iron are available for the formation of
hemoglobin.

Question.1283= A 2 month old has hydrocephalis.


He has been admitted for insertion of a
ventriculoperitoneal shunt. After surgery the nurse
observes that the baby`s anterior fontanel is
elevated. The nursing intervention is to

Rational.1283= An early sign of increased


intracranial pressure in an infant is a bulging
fontanel when at rest. This is a serious sign after a
shunt operation because it indicates that there is an
obstruction to the flow of cerebrospinal fluid, and
corrective surgery may be required.

Question.1282= Which of the following is not a


common clinical presentation of AIDS in children?

KS

Question.1280= The first sign or symptom of an


acyanotic heart defect is often

murmur

Question.1278= A 32-year-old multipara is


Rational.1278= Labor with a fetus in an occiput
pregnant with her third child. She has been in labor posterior position is long and difficult. Backache is
for 20 hours, and the fetal position is LOP. The
common as the occiput bone presses against the
patient is likely to experience
mother`s sacrum.

Question.1272= A primipara, aged 24 and 39


Rational.1272= Braxton-Hicks contractions, or
weeks` gestation, phones the clinic and reports that
false labor, are mild, intermittent, painless
she is having contractions every 15 minutes. The
contractions which occur throughout the second
nurse will help her distinguish the difference
half of pregnancy. Women often obtain relief from
between early and false labor by explaining that it
them by walking or lying down.
is usually false labor when the patient

Question.1271= A 25-year-old primipara is in


labor, having strong contractions occurring every 2
minutes and lasting 60-90 seconds. She feels
frustrated, starts to tremble, and is afraid she is
losing control. The nurse explains to her that she is
moving into the

Rational.1271= The answer is the transition phase


these are characteristic findings of the final or
transition phase of the first stage of labor.

Question.1270= A pregnant patient, gravida 2,


para 1, is admitted for generalized edema,
proteinuria of +2, and an elevated blood pressure.
She is started on magnesium sulfate and is
receiving 2 grams/hour IV. Preparing to mix the
next liter of medication, the nurse assesses her,
finding absent reflexes and a respiratory rate less
than 10 breaths per minute. The first nursing
action is to

Rational.1270= Absent reflexes and a respiratory


rate of 10 per minute or less indicate magnesium
toxicity and the antidote (calcium gluconate)
should be in the patient`s room ready for
administration. The physician should also be
notified at once.

Question.1269= When caring for a patient with


cardiac disease who has just entered the second
stage of labor, the nurse understands that the
management of the patient will likely include

Rational.1269= The goal of management with this


patient is to shorten the second stage of labor.
Appropriate management could include 1) spinal
anesthesia, 2) episiotomy, and 3) forceps delivery.

Question.1268= A priority goal for a patient


experiencing an incomplete abortion is

maintain vs

Question.1266= A pregnant woman is admitted to


the ER. She states that she is 16 weeks pregnant
and has been bleeding vaginally. The nurse would
differentiate between a threatened abortion and an
inevitable abortion when the assessment findings
include the

Rational.1266= A threatened abortion is suggested


when a woman experiences vaginal spotting or
bleeding early in pregnancy and no cervical
dilation or effacement is present. An inevitable
abortion, which cannot be prevented, is indicated
when the cervix has begun to dilate, uterine
contractions are uncomfortable, and vaginal
bleeding increases.

Question.1254= The nurse/midwife performs a


procedure in the last trimester to determine fetal
position, lie, and presentation. The nurse would
describe this procedure as

leopold Fetal lie, presentation, position, and


engagement can be determined by abdominal
palpation of the mother.

Question.1253= Samantha has been taking an oral


contraceptive for 3 years. Since a recent head
injury, she is also taking phenytoin (Dilantin) 100
mg three times a day for seizure control. What
information would the nurse include in the patient
teaching?

Rational.1253= Phenytoin can decrease the


effectiveness of oral contraceptives. The oral
contraceptive should be supplemented with other
means of birth control.

Question.1249= The nurse in the prenatal clinic is


teaching a goup of pregnant women about
self-care. Instructions will include that when lying
down, it is best to lie on the left side in order to

Rational.1249= The growing uterus may press on


the inferior vena cava when the pregnant woman is
supine. This reduces blood flow to the inferior
vena cava when the pregnant woman is supine,
and also reduces blood flow to the right atrium.
This may lower the blood pressure, causing
dizziness.

Question.1244= An elderly patient with pedal


edema is admitted to a medical floor with
symptoms of vascular disease. The nurse caring
for this patient is aware that edema may result
from

Question.1241= A patient with cervical cancer


enters the hospital to have a cesium implant.
Before the insertion of the implant, the nurse tells
the patient what to expect while it is in place.
Which of the following statements is accurate?

Question.1239= The nurse caring for a patient


with polycythemia vera notes all of the following
clinical signs and symptoms. Which of these
should be reported to the physician immediately?

Rational.1244= Edema can result from increased


capillary pressure or from the passage of protein
molecules and fluid from blood into interstitial
fluid because of increased capillary permeability.
An aneurysm is a localized dilatation of an artery
caused by a weakened arterial wall ). Third
spacing occurs when fluid is filtered from the
plasma and then accumulates in other areas of the
Rational.1241= It is necessary
body (4). for staff, family and
friends, to have decreased exposure to the source
of radiation. Nursing care is to be delivered in a
timely fashion, with special attention given to
physical distance. The patient receiving any type
of radiation should not be in any pain as a result of
the treatment (1). Bed linens need to be changed as
needed, not because of the radioactive implant (3).
patients with radioactive implants should be able
to at least assist with activities of daily living.
patients are encouraged to provide self-care
measures to decrease the possibility of
complication due to immobility (4).
Rational.1239= This is a classic clinical
manifestation of thrombophlebitis, a serious
complication of this blood disorder.

Carminative enema.
Rational.1217= This enema is made up of a
solution which is 30 mL of magnesium, 60 mL of
glycerin, and 90 mL of water. A physiologic
Question.1217= The physician ordered an enema
normal saline enema is safest for infants and
for a patient to relieve gastric distention. The nurse
children because of their predisposition to fluid
would expect the type of enema ordered would be imbalance. A soap suds enema is most frequently
a (an)
used for cleansing the bowel prior to surgery. An
oil retention enema is an oil-based solution. It
permits administration of a small volume, which is
absorbed by the stool. The absorption of the oil
softens the stool for easier evacuation.

Question.1211= When teaching a patient to use a


walker, the nurse would instruct the patient to
move the walker

Rational.1211= The proper distance is 6 inches so


that the individual maximizes the safe and
effective use of the walker.

coumadin s/e

orange urine

50 GRAMS OF DEXTROSE

4 CALS PER GRAM

lactating mothers

Restricted fat intake. Lactating mothers need 4 to 5


glasses of milk a day. They should never be
advised to restrict any nutrient or attempt to diet
during lactation.

The nurse can instruct a diabetic patient that blood


glucose levels can be moderated by including
more

Dietary fiber decreases the absorption of


carbohydrates and may affect the release of
gastrointestinal hormones, which influence insulin
and glucagon secretion. Fiber also increases and
prolongs satiety and helps control blood lipids.
Omega 3 fatty acids appear to decrease serum
triglycerides, but also tend to increase fasting
plasma glucose levels.

A male client has just returned to the unit after he


had an exploratory laparotomy. His orders include
applying a binder to minimize muscular tension
over his incision site. The binder that will be used
on this client is a

The straight abdominal binder centers support over


abdominal structures. It provides continuous
wound support and comfort. A stretch net binder is
used for support of dressings or surgical sites over
the client`s arms or legs. The T binder is applied to
facilitate placement of perineal dressings and
provide support to perineal muscles and organs. A
double T binder is used to support perineal
dressings for male patients.

When obtaining a urine sample from a catheter,


the period of time for clamping the catheter is

The urinary catheter should be clamped off for


short periods of time only, in order to obtain a
small sample of urine. Longer periods of time may
cause the urine to back up into the bladder and
increase the potential for UTIs. Thirty minutes is
sufficient to obtain a specimen in a client with a
normal urine output.

The normal potassium level is 3.5-5.0 mEq/L. The


patient`s potassium level is low, and he needs to
replenish what has been lost as a result of taking
the Lasix. In addition to taking potassium
supplements, the patient should be given a list of
the appropriate foods that have an average of 7
mEq potassium per serving. (Fruit, meat, fish,
instant coffee, and milk are high in potassium.

electrolytes

normal electrolyte values for an adult are as


follows: sodium of 135-145 mEq/L, chloride of
100-106 mEq/L, potassium of 3.5-5.0 mEq/L, and
bicarbonate of 22-29 mEq/L.

sed rate

This is a normal sed rate for a female over age 60.


Under age 50, normal is 20 mm/hr. If it were
increased, it would indicate presence of infection
or inflammation, and surgery might have to be
postponed.

PT abnormal

Because the patient is not on anticoagulant


therapy, the results are abnormal (normal PT is
11-15 seconds). It is important to notify the head
nurse or physician before the biopsy; bleeding
could be life threatening. The patient will probably
be given vitamin K therapy and when the PT
results return to the normal range, the procedure
can be done. Liver disease likely caused the
prolonged PT.

lipid agents

to the sun or photosensitization is a risk for


patients taking HMG-COA reductase inhibitors
(Mevacor). (1) Niacin is usually given with bile
acid sequestrants because they work
synergistically. (2) Bleeding from the gums or
rectum is a sign of vitamin K deficiency from bile
acid sequestrants (Questran).

creatine

The normal serum creatinine level for a male is


0.6-0.9 mg/dL. A patient with a mild degree of
renal insufficiency would have a slightly elevated
level, which in this case would be 1.7. Levels of
3.3 (2) and 4.0 (1) may be associated with acute or
chronic renal failure.

labs for pancreatitis

These elevated serum levels (amylase and lipase)


are the hallmark of acute pancreatitis. Increased
white blood cell count and serum bilirubin level is
also seen with acute pancreatitis. Elevated alkaline
phosphatase (4) is found in chronic pancreatitis.

pyloric stenosis distinctive signs

peristalic waves

quickening

17-19 weeks first felt

thrombocytopenia

platlet count 50k or below. Sx petechie,purpura


and hematuria

Dornase alfa

reduces viscosity of sputum with CF

blurred vision

advanced indicatior of
PIH

suction

mouth then nose of infant

menstrual cycyle regulated

FSH and LH which are produced in the


hypothalmus

epidural

vasodilation and blood pooling in extremites. Can


lead to hypotension so monitir BP. Immediate TX
elevate legs.

Bucks extension

following a hip Fx is to immobalize to relieve


muscle spasm at the fx site

pacemaker malfunction

weakness and fatigue R/T hypoxia of the tissues

LVN can't

take admissions

Denver shunt

used for pts with ascites and cirrhosis. diverts fluid


from the abd to jugular or vena cava

shunt

should feel warm, hear bruit and feel thrill

chronic hypoxia

R/T copd may stimmulate excessive RBC


formation will increase blood viscosity and risk for
thrombus

gentamycin

aminoglycide nephrotox!

B12

comes from animal products increase or get b12


shot R/T pernicous anemia

mechanical vent

may cause stress ulcers so maintain Ph above 5

lactalose

decreases NH3 levels with pt s who have hepatic


coma

positioning for pancritits

sit up or lean foward to decrease pain

laproscopic shoulder pain

after co2 is injected into the abd main cause


shoulder pain after surgery

colchine (for gout) sx of toxicity

diarrhea

aminoglycides

more active when the urine is alkaline so soda


bicarb may be given

seconal

barbiturate used for sleep. Monitor for sedation

spironolactone

decrease urine na then causes diuresis

hypertonic fluids

higher in osmotic pressure used for icp because it


reduces idema

thiazide diuretics

monitor for k loss monitor muscle weakness leg


cramps hypotension

start solids

5-6 months

6 months

sits with minimal support

sterile technique

when skin is broken or STERILE body cavity is


entered

surgical asepsis

cath to bladder

after surgery

immediatly assess pulmonary function first thing


post op

critical pathway

managment tools developed for particular types of


patients . they have interventions outcomes and
progress. they are multidisaplinary.

tens

blocks painful stimuli from traveling over small


muscle fibers.

osteoarthritis assememt findings

joint pain, crepitus, heberdens nodes

at a 4 y/o wc visit the kid is really small. what is


the nurses first action in evaluating his growth

compare with siblings

increased vag drainage

nl during pg

soap enema

nurse should hold about 12 inches above rectum.


admin slowly. insert the tube 3-4 inches in rectum,
direct the tip towards the umblicus

hospice

inhances the quality of life for terminal ill pt. helps


pt and fam to live life to the fullest each day

lasix

because lasix is k wasting the pt needs to eat k


foods such as apricots, dates, and citrus foods

a pt is chocking on his lunch and coughing


forcefully what should you do

if he is coughing he should be able to dislodge the


object if obstruction occurs then do abd thrusts
hemi

intermittent cluadication

chronic perhrial vascular disease that reduces 02 to


the feet so do meticulus foot care. Bath feet in
warm water and dry throughly cut toe nails straight
across, wear well fitting shoes, avoid meds unlesed
cleared by M.D, stop smoking not cut back!

rubella iz

12-15 months

if 51/50 and client is feeling better answer is:

continue to monitor for suicide

tetany bactrium

clostrdium

late sign of hypoxia

increased heart rate R/T increased energy demands

fluid overload causes

cerebral edema with a pt with meningitis check for


icp too

care plan for client in manic state is

listen attentivly to pt request, express wilingness


and seriously consider the request, and respond
later.

nurse can anticipate a client in the 2nd trimester to


be

increased intorspection and general sence of well


being

dislodge infants airway

5 back blows followed by 5 chest thrusts

chest tube working correctly

flux with fluid level while breathing. Bubbling


indicates an air leak. constant level indicates an
obstruction

on g tube first be concerned with

aspiration

while collecting data on unresponsive pt. first


check

responsivness

mao

may cause hypotension so nursing dx is r/f injury

group b strep mom

low temp in neonate = signof infection. Place mom


and baby in seprate rooms

mom rh neg baby rh pos

give rhogam in 72 hrs

hemangiooma

vascular tumor in baby

2nd trimester behavior

narcissistic and facinated by children

change position first action.


variable decels

then they might do amniofusion or c sec if does


not help

hypogly in the neonate

lathargic

postpartum perineal discomfort

contract your butt before sitting

cna can

bottle feed a 24 hr neonate. no first bath, no diaper


changes after circ, no vitals in transitional period

pre term labor nursing intervention

provide adequate hydration to halt contractions

cocaine an pg

cause increased uterine contractlity, and preterm


labor. notify neonatologist about pts arrival

contraction breathing

shallow chest breathing

34 weeks pg and in the er complaning of vag bleed


nurses first action is

check fhr and maternal bp

vbac incsion

lower uterine transverse is the only one allowed


with vbac

pg chocking

provide chest thrusts

heat loss from evaporation with the neonate

drying throughly after bath can prevent this

false labor contractions

usually occur in the abd irregular and typically


relived by walking

mom with gdm baby is hyperglycemic then is


hypoglycemic in the first 24 hrs what is a nursing
intervention

provide frequent early feeding with formula

high folic acid

egg yolks, nuts, seeds and liver

phototheraphy adverse rxn

watery stools

lochia 2nd day post op

red and scant

embryo third week

heart beats

neonate nl resp

30-60

heprin

does not cross placenta

gave birth yesterday the fundus should be

1 fb below umbilicus

molar pg

use bcp for 1 year

placenta previa

softrelaxed non tender uterus

A client with glaucoma has been prescribed


Timoptic (timolol) eyedrops. Timoptic should be
used with caution in the client with a history of:

Emphysema
eta blockers such as timolol (Timoptic) can cause
bronchospasms in the client with chronic
obstructive lung disease

A client being treated with sodium warfarin has an


INR of 9.0. Which intervention would be most
important to include in the nursing care plan?

Assess for signs of abnormal bleeding


The normal international normalizing ratio (INR)
is 2 - 3. A 9 might indicate spontaneous bleeding.

An elderly client asks the nurse how often he will


need to receive immunizations against pneumonia.
The nurse should tell the client that she will need
an immunization against pneumonia:

Every 5 years
Immunization against pneumonia is recommended
every 5 years for persons over age 65, as well as
for those with a chronic illness.

A 45-year-old client returned from a colon


resection 2 hours ago. Which vital signs indicate
possible hemorrhagic shock?

BP 96/60, heart rate 120

A client is 2 days post-operative colon resection.


After a coughing episode, the client's wound
eviscerates. Which nursing action is most
appropriate?

Cover the wound with a sterile saline-soaked


dressing
If the client eviscerates, the abdominal content
should be covered with a sterile saline-soaked
dressing.

gathering data on an ocd pt what is most inportant


to ask

do you have trouble controlling upseting thoughts?


intrusive thoughts repeat over and over

pt is dnr and has cardiac arrest lvn should

assess for signs of death

inserting an ng tube wear

gloves gown mask and goggles

s/s of fat embolism

confusion, agitation, delirum, coma, increased


resp, cp, sob, pallor, htn , paetechiae on chest.

liprium

antianxiety used to tx etoh w/d

walker

pick up move forward 10inches then take a few


steps to it!

pt after a cataract removal c/o severe pain what lvn


to do next

tell RN

dumping syndrome

decrease carbs

acute renal faliure

decreased urinary output

6-12

industry aspires to be best learns social skills, how


to finish tasks, sensative about school.

return and turn off feeding.


tube feeding at 100ml hr risdual checked its a 90
what to do?

stop feeding if risdual is 50% of the volume


infused in 1 hr

pku testing

p a source of protein is injested. pref in the


hospital in the first 24 hrs repeat in 3 weeks

preshoolers

need to see and play w/medical equiptment

nl cvp

5-10 cm h20

tx for hyperkalemia

decrease k, iv nacl, kayexaltae ,ekg.

tx hypokalemia

give KCL no more then 20 qhr observe i/o, ekg

hyponatremia
135-145nl

weakness, restless,delirium, confusion

ca 4.3-5.3
hypocalcemoa

cramping, tetany

ca 4.3-5.4
hypercalcemia

a/n/v lathargy

tx of metabolic acidosis

na bicarb, na lactate. watch labs. monitor for


hyperkalemia and dehydration.i/o.

metabolic alkalosis tx

high k high cl foods. i/o. Kcl. ammonium cl to


increase hydrogen ions. labs, monitor for
hypokalemia.

resp acidosis tx

all resp stuff to promote drainage, breathing etc.


give na hc03,RL, and k

resp alkolosis tx

s/s hyper reflex, muscle twitching, convulsions.


use rebreather. monitor k and hco3

ab compatable w/

all blood groups

bladder irrigation

30-50ml

dvt

bed rest 3-4days no pillow under leg . monitor for


PE. antiemboletic stocking to the other leg

fat emboli tx

etoh drip, cortisone therapy, deholin to emulsify,


lipid lowering drug

mag toxicity first indication

extreme thirst

3.5-5.5

na

135-145

p mva fat emboli

24-48 hrs p

following removal of ng tube for a pt with


pancreatitis diet should be

high in carb

diet after cholectomy

low residule and no dairy r/t the moucus it can


cause

central line usually

r subclavian

cl

100-106

bicrb

22-29

high gastric acid indicates

duodeal ulcer

sed rate under age 50

20mm/hr

nl PT

11-15 seconds

HMG/COA

teach photsensativity

elsa

verifies hiv dx

male creatinine

0.6-0.9 elevated with renal insuff

nl bun

10-20

neutrophils

first line defense

most common nosocomial infection

c diff

removing isolation gown

untie front waist strings, remove gloves, untie neck

emergency community wide disaster network who


is notified first

local health officer commander

rad exposure

1 hr to radiation exposure = 5o RADS

smallpox contraindicated

cardiac, leukemia, lymphoma, PG, burns, hiv,


shingles, eczema

anthrax

standard percautions

pneumonic plague

standard and droplet percautions

dm type 1 bg over 250 then

check for ketones teach the pt this

high na in milk

FYI

phenobarb

excreted by the kidneys so good for hepatic coma

pt starts to shiver during a tempid bath

stop as the body is attempting to produce heat

dicumarol

an anti coag so teach pt about this type of drug

colchacine tox

diarrhea first sign

priority assesment for bacterial endocarditis

emboli

eartquake in the hospital

first thing!!! look for instructions. dont call about


pts that need help. weird

kid stong by jellyfish first thing

bath in vinigar and apply shaving cream

pt is has severe reaction to meds initial action

place pt supine in shock position

with heating pad first sign of thermal injury

redness to area

proper depth for infant cpr

1/2 to 1 "

earlisest sx of shock

narrowing pluse pressure

radiation beads have follon out of pt you cant find


them

let the rn know

nl ms dose

1mg per//ml
or 5mg/ml on PCA

after masectomy position pt

semi fowler with affected arm elevated

radiodermititis

avoid all creams or lotions on the area. wash with


leukwarm water and mild soap

early warning ca sign

change in bowl habits

wbc nl

5-10k

neomycin preop bowl surg

to sterilize the bowl

cesium needle in cervix

bed rest until it is removed

malignant melonoma

worst prognosis

alkylating chemo drugs work by damaging dna in


the cell nuclus

fyi

after abortion pt develops dic what is the most


critical intervention

administer ordered meds (hep)

nst is reactive means that

good outcome increased fetal hr w/movment

dm mom ask about breast feeding

its encouraged insulin dose not cross to the milk

jaundice appears

2-3 days of life

primipara

normally go through effacment before dilation

syphillis

may be passed to the fetus after 4 months of pg

lvn care p gen anesthsia

monitor bp q 3-5 min until stable

tetany

uterus contarcted for more then 1 min= can rupture


report to md

mag sulfate

assess resp at least 12 per min

dm mom check baby for

hypogly, resp. distress and hypocalcemia

baby chocking or resp distress

pick it up by the feet and head low

post term babys look

small like they have lost wt long nails little subq


dry skin

bcp's

inhibit fsh production

small for gestational age infants develop

hypothermia

pg diet

increase iron increase fa and increase cals 300

mastitis

apply heat to the breast

american red cross

clara barton

elizabeth mahoney

first black nurse

managed care

80's to halt rapidly growing health care costs

complemetry care

accupuncture, theraputic tuch and imagry

joint commision

makes sure hospitals meet rigid standards

inferred consent

no consent given by pt but situation is life


threating so its inferred the pt wants it

medical asepsis

process to minimize or eliminate organisms that


can cause disease

bureaucratic leadership

policy minded

extingushier c

electcal fire

paradoxical responce

opposite effect that you wanted

enteric coated

dose not disolve until reaches the intestines

woodslamp

high pressure mercury lamp long wave ultra violet


to detect suprafical fungi and bacterial infx

moist packs applied to the skin

for debridment

vitiligo

lack melanocytes results in white patchy skin

carbuncle

several intr connecting boils in a cluster...yuck

burn types

chemical,electrical, rad or thermal

entire back burn

18%

major burn

includes second degree over 25% of body and


inhilation burn injuries

digestion of carbs

begins in the mouth

major fnx of fat

provide energy

albumin and globulin

blood proteins that help keep fluid were it belongs

electrolytes form

acid bases and salts

trace minerals

needed in the body in small amounts like zinc

fat soluble vits

likely to cause tox cuz they accumulate in the body

major intracellular electrolyte

basal cell

non metasta in whites around the head and neck


looks like fleshy nodule

saquamous

may mets red scaly patch second most common


face lips mouth and rim of ears

malignant melonoma

danger dark pigmented mole occurs near a mole or


other dark spot

endocrin consists of

adrenals ovaries testes islets parathyroid pit and


thyroid gland

pit gland master gland cuz

it stimulates other endocrin glands to secrete

fbg

65-115 nl

cushings syndrome

excessive adrenalcortico hormone from the adrenal


cortex or brain tumor

hypotension, circulatory colapse coma death!


Dont stop steroids all at once
or an infx can cause this
adrenal crisis
emergency tx: iv fluids to increase bv, Iv
hydrocortisone , recombant position with legs up

tyroid storm tx

hypothermia blanket antithyroid and antiarrythmic


meds

myexdema coma

long term untx hypothyroidism

4 main oral anti dm catagories

alphaglucaside inhibators, biguanides, sulfono's,


thiazolidenedes

glyburide

sulfon

regular insulin humulin r

short acting

humalog insulin lispro

intramediate acting

lipodystrophy

change insulin sytes often!

hypogl sx

sudden onset of pale moist cool clammy skin w/


sweating shaking and hungur

hypergly sx

SL
Ow onset of flushed dry skin, hot drowsy fruity
breath, labored breathing

somongi effect

hypogly followed by rebound hyper gly

tx of hyper gly

iv fluids antiemetics and low dose insulin

insulin or oral dm meds dont drink cuz

etoh can cause extra hypogly effect

glucotrol and etoh

disulfaram effect violent, flusshing, throbbing ha,


sob and unstable bp

20 grams simple carb =

4 oz OJ

high humidity ant temps 95-100

watch for heat related injuries

muscle cramps, ha, dizzines, weakness and


excessive sweating=

heat exhustion
tx cool the pt w/o chilling, loosen clothes cold wet
compress to the skin water replacment

heat stroke

core temp 105-110 sweatring stops and brain cells


FRY!

heat stroke nursing tx

remove clothes wrap client in cold wet sheets,


place ice axillary and groin

core 94 or less
sx shake clumbsy slow movment arrythmias
hypothermia
tx gradual rewarming cardiac monitor warm
bevrages when awake

frost bite

blisters some nerve damage increased


hypersensativity to cold

anaphlatic SIRES

stabilize,identify,reverse,eleminat,support

s/s of anaphlaxis

hives cold clammy skin dizzy weak low bp nv


itchy watery eyes

nervous sytem

communicate and control

frontal lobe

written and motor speech

expressive aphasia

understand written and verbal but can not write or


speak it

can not understand writen and verbal

receptive aphasia

nervous sytem aging

short term memory loss and thermoreg

206 bones in body

FYI

rickets

vit d
bone formation

foramen

were stuff goes through

cardiovascular aging results in

hematopoiesis

low co2 stimulates breathing

for nl adults

aging of the resp system

increase r/f infx

aging urinary system

nocturia

lens

focuses on light rays

frail elderly

85 and over

maoi

nardil

avoid pickles,aged, smoked, fermeted foods, dry


sausage and wine
maoi on thymine foods

watch for htn crisis sx include occipital ha stiff


neck palp sweating dilated pupils photophobia and
n

ect adverse effects

ha and myalgia

benzo's

xanax and valium

epse

akathsia

nms cardinal sign

hyperthemia and rigidity

anametadine and cogentin

treat adverse effects of antipycotics

autisim sx

rituals and lack of social interactions

adhd meds

strattera, ritalin and aderall

amniotic fluid

cushing

para

viable newborn or gestation p 20 weeks

adolsent pg complications

pre eclamp, eclamp, spon ab

molar pg early indication

uterus enlarging early

iorn rich

tofu, fortifed cearl, bread and lean meat

accreta placenta

failing to seperate in 20-30 minutes retained


placenta

mag sulfate(epson saltsO

prevent and control sz

postpartum hemmorage

notify md and do external uterine massage

rh neg mom rhpos baby

baby will have eretroblastosis fetalis if left untxed

moro reflex

startle through out arms draw up legs

post partum

3-10 days post delivery = mild depression usually


resolves in 2 weeks

immediatly report jaundice in the newborn esp, in


the first 24 hrs cuz

indignative of pathologic jaundice results from


mother building up ab against her own fetus

first meconium

greenish black

nl newborn axillary temp

97.6-98.6

newborn bp

50/30-80/50

newborn apical for 1 min

120-160 and slightly irregular

newborn resp

30-60

2 months

dtap,hib, ipv,pcv

scarlet fever

pinpoint reddots and strawberry tongue

chronic renal failure diet

avoid high k foods,


Eat high ca and high fe

cushing diet

increase protein
increase k
descrease na and cals

addisons diet

increase na
decrease k

1 L of water=

1kg

1 cup

240 ml's

1 pint

480 ml

1 quart

960ml's

turgur

forearm and sternum if wrinkled greater then 20


sec poor

stage4 full thickness fascia, involves muscle


tendon and bone

skin breakdown

stage3 full thickness damage and necrosis down to


the facia
stage 2 epidermis interrupted abrasion or blister
stage 1 intact skin nonblanching errathemia

trochlear

downward and inward eye movment

trigeminal

corneal reflex, chewing, face and scalp sensation

vagus

gag reflex, swallowing talking,sensations of the


throat and larynx

hypoglossal

tongue moevemnt

battle sign

ecchymosis at mastoid process in basilar skull fx


ususally occurs 48hrs p

nystagmus

constant involontary eye movment

hippus

rythmatic and rapid dilation and constriction of the


pupil

decorticate

in c sz

generalized absence sz

brief of conciouness or posture

myoclonic generalized sz

repetative muscle contarctions jerking

tonic clonic

stiffness followed by loc and rythmatic contrations


of extremites

simple partial sz

focal no loc aura usually

complex partial sz

focal sz w/alteration in loc

afterload

resestance against which the vebtericles must


pump when ejecting blood

nl icp

10-15

l side heart failures sx

pulmonary edema, sob, orthopnea,

r side heart failure

dependent edema, hepatomegaly, abd pain,


bloating

ra

fatigue anorexia low grade fever joint pain

when is chest tube ready for removal

when suction is removed no flucuation is noted in


the water seal chamber

elderly client c PNU first symptom

altered mental status

chemo before bone marrow transplant cuz

to make space for new bone marrow and that all


the cancer cells are removed

nl neutrophil count

2200 to 7000mm3

neupogen

colony stimulating factor given p chemo to stim


wbc's

nl platlett count

150,000-400,000

s/s of hyponatremia

ha lathargy confusion sz

hyper mg

loss of deep tendon reflexes

demerol metabolites

cause sz's

albumin iv

colliod osmotic pressure

cholinergics

increase ach in the brain good for Alzheimers

chronotropic

change in heart rate

inotropic

force of heart rate

dromotropic

change w/conduction of electrical inpluses

p ng med administration

leave pt sitting for 30 min

IV ntg drip

in glass bottle cuz ntg adhers to plastic

iv acess

insert needle at 30 degrees

phelbitis

remove cath apply warmth moist pack

intr atrial lines

for chemo to give close to tumor

when drawing blood from CVAD

disgard 10 ml's

disconnected CVAD pt c CP first do

turn pt on left side in trendelinburg

Groshong cathether

has no clamp just a 3 way valve at the end

needle to access inplanted port

20 gauge noncoring

flushing Groshong

5-10 mls of NS weekly

RH positive can recieve rh neg bloos

but RH neg cant recieve RH pos


If it happens and its admisterd wrong till th eblood
back and theMD

type o blood can only recieve

type o blood

neutropenic pt usually be transfused with

granulocytes

unit of whole blood must be used in

4 hrs

Graft vs. host disease

attack on host tissue from lymphocytes. Usually


occurs w/comprimised pt

type AB

means no antibodies in the blood

amiodarone and dig

cause increased dig levels

aldactone

k sparing diuretic should not have increase k

electrolyte inbalance w/dig

hypokalemia can cause dig tox

glucose6 phosphate dehydrogenase

if on asa watch for hemolysis of rbcs. Esp c jews

flagly and etoh

= n/v

antacids and dig

can decrease absorption of dig!

desmepression acetate

for hemophilia and von willebrands releases factor


8

timoptic

reduces aqueous humor production

tapazole

tx for hyperthyroid

cerebyx

tx of sz's

s/e of NTG

reflex tachycardia

tetracycline

no if less then 8 cuz it stains teeth

phenergen adverse rxn

tortacollis neck head tilted to one side

trycyclic

adverse rxn= increased temp

body h20

60-70%

main source of energy in a diet

carbs

breast feeding

increase cals by 500

antabuse and drinking etoh

cp, ha, nausea sweating

opioiod w/d sx

tachycardia, tacypnea, htn, mydiasos

thrill

narrow or buldging blood vessel pluse feels like a


vibration

passive immunity

administration of ab produced by other people or


animals. the pt is exposed to the antigen and
produces his own ab

2 y/o

uses 1-2 word sentances

late fetal heart decels

turn on l side give 02

p epidural priority action

monitor bp

evaluation for restrant need

q4h

haldol s/e

hypotension tacycardia

regression

pt's use it to elimimate anxiety

ivp

be NPO

physiological jaundice

on the second or third day decreases by 6th or 7th

renal failure be concerned w/

change in BP

infectious hep

enteric percautions

high k foods

oj, dried dates, milk

position for hypovolemic shock

head on pillow and feet slightly elevated

cobalt implant percations

visitors 1 hr q day exposure, everyone wears a


protective shield, rad tx sign on door

fetus in posterior position

back discomfort

expressive aphasia

theraputically encourage to communicate even


though not using correcrt words

preeclampsia early signs

htn, proteinurea, and edema

following a hip replacment during first postop


hours observe for

hemorrage

infant by 6 months

doubles there wt

sengstaaken blakemore tube for espohgeal varices


most important safety intervention

scissors at bedside
resp system can be occluded if balloon slips moves
up esophagus and pressures trach. cut tube if in
distress!

newborns usually sleep

20 out of 24 hrs

GDM most hard to control BS

early post partum R/T the [placenta contains the


hormone insulinase which blocks insulin during
PG

bethanechol cl

stimulates PNS increases tone and motility of


smooth muscles.
Give to a pt if they cant pee p a TURP..

preemie

use soft preemee nip and half strength formula


then work to full

nl pluse for a 5 y/o

100

pg be sure to take

fe and folacin supplemental

3 month old can

focus eyes on stuff

lactalose

decreas blood ammonia

hypoglycemia in a infant

shrill or high pitched cry

korsakoff syndrome

neuro degeneration R/T vit deficiency

pt in er c DT's orders for IV w/vit b6, valium, or


check VS. Which one to do first?

give IV w/ vit b6 first! wow strange!

baby w/spinal bifida most important assesment

measure head circ qd

hirschsprugs diseas sx

fails to pass meconium in 24-48 hrs classic sx

apical on a infant

left mid clavicular, 3rd and 4th intra costal

interventions for a kid with RA

warm comressions and splints at noc

mumps percautions

droplet, direct and indirect contact via the


resp.system

position for extrophy of the bladder

side lying to aid in emptying

bronchiolitis priority assesment

flaring nostrils, exp. stridor and wheezing

negativism

struggle by a kid between what they can and can


not do

one dram

4 ml's

interssusception

sudden onset of sx

left side heart failure

fatigue, dyspnea, wheezing

menieres disease

extreme vertigo, bedrest.

osteoporosis teaching

diet high in protein, ca, vit d, wt bearing excercise


no etoh

low k

skeletal muscle weakness

bucks extension p hip fx

relives muscle spasms

pt wants to leave AMA what to do FIRTS

notify MD, then they will try to make her stay,


then you can give AMA form!

breech of duty

care did not meet accepted standards

child had cardiac cath monitor

peripheral pluses for symetry

child admitted with sickle cell crisis what should


nurse do first

vs to get a baseline including temp. they are prone


to infx

low absolute granulocyte

restrict visitors w/active infx's

cytoxan for chemo monitor for

hematuria which can indicate hemorragic cystitis


which is an adverse effect

subtle signs of resp distress in a child/infant

restless, increase resp effort, increase resp,increase


pulse, can not calm kid

a child with asthma attack is brought in he has no


wheezing!

probabley hypoxic or would be wheezing!


emergancy

after shunt placment

bed flat to prevent subdural hematoma

meningitis

droplet spread my resp.

after cleft repair

sit upright for feeding

bactrim

crystalizes the kidneys drink lots of water

child is a home and has scolding burn

immediatly flush site with tempid water to stop


progression

glomerulonephritis

monotor bp q 4 h
htn major complication

nl urine output for infant

5-10 ml/hr

nephroblastoma

surg in 24-48 hrs after dx then chemo and rad to


follow up

infant w/ an overdose of thyroid med

tachycardia, fever, irratiblity,sweating

neonate w/pyloric stenosis

postion on right side

NG tube and complains of nausea

check placment then irragate the tube

gluten free diet

no malted milk, wheat bread or spaghetti

cleft palate surg delayed till

18 months

cleft lip

birth to 3 months

exacerbation of juvenile RA

stress and climate

a child w/ duchennes ms

low cal high protein high fiber


R/F constipation

absence sz

brief loss of responsivness with minimal or no


alteration in muscle tone

Ritilin adverse rxn

slow growth and ht, sleepy, decreased appitite

do not admin fe with

milk it delays absorption

breast feeding

natural acquired passive immunity

kid w/ cyanotic heart defect what sx

cyanosis, crabby, clubbing, crouching

ultrasound for PG

used at 18-40 weeks

what is priority when feeding a neonate wit a cleft

frequent small feedings

fetal blood ph shows 7.12 what nursing


intervention is called for

prep for c section. baby in fetal acidosis

decrement

letting down phase of uterine contractions

to halt uterine contractions give

brethine/trebutaline

monitor i/o with pitocin because

it causes water intoxication leading to sz,coma and


death

circulating HCg disapears

8-24 hrs p delivery

letting go phase mom excepts the kid as a

separate individual

neonate temp range

96-97.7

phototherapy to reduce

unconjugated bili

methergine ergot alkaloid given to stimulate

uterine contractions p birth to decrease bleeding

mag sulfate produces smooth muscles depression


asses for

post partum hemmorage

p amniotomy priority nursing intervention

asses fetal heart tones

transitional phase of labor

cerv dilation 8-10 contractions 1-2 min apart

hiatal hernia

avoid carbonated drinks R/t to esophageal


irratation and increased pain

crohns diet needs

high cal,protein,carbs and low fat

cytoctec for gastric ulcer

monitor for diahrrea

levothroxine

can cause tacycardia indicator of thyroid tox

sinemet is working if pt

decreases tremors

eccrine gland

associated w/body temp regulation

on corticosteriods check

blood glucose

partial thickness burn

fluid filled vesicles

eswl

kindney stones are shattered

a pt w/kidney stones

drink 3 l h20 each day

PTH deficiency abnl serum levels

ca and phospurus

GTT dx w/Dm if greater then

200

radioactive iodine test

increase w/hyperthyroid decrease w/hypo t

levothyroxine

inhances coumadin dose!

addisonian crisis

hyponatremia needs NS

abd rigidity

classic sign of peritonitis

HA after lumbar puncture

increase fluids to help restures CSF volume

greatest r/f femoral fx is

hypovolemia from hemorrhage

temporal lobe

controls speech and hearing

plasmapheresis

ab removed from blood

to prevent vasospasms

give crystalloids

epitaxis

head slightly forward sitting up

breathing w/ emphysema triggered by

low 02 levels

TB drugs effectvness can be indicated when

sputum results show neg for TB

after a cath there is swelling at site

apply firm pressure and have some one call MD

ca channel blocker

decrease bp and hr

if BP below 90 systolic

delay NTG

Growth spurt, fever, and irritability changing to


lethargy.
The nurse is monitoring a child with
hydrocephalus who has a ventriculoperitoneal
(VP) shunt. What clinical manifestations will the
nurse be most concerned about?

he major complications of VP shunts are infection


and malfunction. Children can "outgrow" shunts or
distal ends can dislodge after growth spurts. Fever
can be a sign of an infected shunt, and irritability
deteriorating to lethargy could be due to increased
intracranial pressure (ICP) from a blocked shunt.
Appetite usually decreases with increasing ICP;

A 2-year-old toddler was diagnosed with


iron-deficiency anemia. Which of the following
statements best describes the anemias of
childhood?

The clinical manifestations of anemia are directly


related to the decrease in oxygen-carrying capacity
of the blood.
Clinical manifestations of fatigability, anorexia,
weakness, and tachycardia are a result of vitamin
B12 and folic acid deficiency. This results in
reduced production of red blood cells, and a
2-year-old child will manifest symptoms of this
disorder.

When assessing clinical indicators of adequate


cardiac output in children, which of the following
signs are most important?

Pedal pulses, skin temperature, and capillary refill.

A mother with a 4-month-old infant comes to the


clinic for a well-baby examination. The nurse
advises the mother to change the formula she is
feeding the baby to one that contains iron. The
nurse explains that the reason for this is

The infant`s iron source from the mother is


depleted.
Between 3 and 5 months, the infant has used the
iron provided by the mother and requires further
supplementation if bottle feeding

Which of the following symptoms is the priority


assessment because it is suggestive of a
complication of a central nervous system
infection?

Separation of cranial sutures


Meningitis is a common CNS infection of infancy
and early childhood. Increased intracranial
pressure, which can accompany meningitis,
accounts for separation of the cranial sutures,
bulging not depresse.

What anatomical condition must be present in


order for an infant with complete transposition of
the great vessels to survive at birth?

Large septal defect.


Because complete transposition results in two
closed blood systems, the child can survive only if
a large septal defect is present

Assessing a child with a possible cardiac


condition, the nurse knows that a child with a large
patent ductus arteriosus would exhibit which of the
following symptoms?

Is acyanotic but has difficulty breathing after


physical activity.PDA is acyanotic.

Following a saline-induced therapeutic abortion, a


patient has developed disseminated intravascular
coagulation (DIC). The most critical nursing
intervention for this patient is to

Administer ordered medications.In DIC, the


patient begins to hemorrhage after the initial
hypercoagulability uses up the clotting factors in
the blood. Administering heparin, therefore, is a
critical nursing intervention. Heparin prevents clot
formation and increases available fibrinogen,
coagulation factors, and platelets

A patient`s laboratory results indicate a creatinine


level of 7 mg/dL. This finding would lead the
nurse to place the highest priority on monitoring
the patient`s

This elevated creatinine suggests impaired renal


function. Monitoring intake and output will
provide data related to renal function

A common test used to determine fetal status in


the presence of preeclampsia is the nonstress test
(NST). If this test is "reactive," the nurse knows
that it means

Reactive = good outcome. Increased FHR with


movement indicates normal reaction and adequate
CNS integration

Which of the following statements is usually true


about cervical changes in primiparas

Primiparas normally go through effacement before


dilation of the cervix. Multiparas tend to dilate and
efface simultaneously.

A patient is 3 days postpartum. Her vital signs are


stable; her fundus is 3 fingerbreadths below the
umbilicus, and her lochia rubra is moderate. Her
breasts are hard and warm to the touch. The nurse
would evaluate that the patient

Is normal for 3 days postpartumFrom the


assessment findings of the lochia and fundus, the
new mother is progressing normally during the
postpartum period. The breast signs indicate
normal engorgement, which occurs about 3 days
after birth. With stable vital signs, infection is not
likely to be a problem. Applying warm packs and
wearing a nursing bra will reduce discomfort.

Assessing a patient with eclampsia, the nurse


knows that a cardinal symptom is

High blood pressure is one of the cardinal


symptoms of toxemia or eclampsia, along with
excessive weight gain, edema, and albumin in the
urine.

An eclamptic patient has been receiving


magnesium sulfate IV 2 g/hour. What symptom
would indicate that the current dose be continued

A respiratory rate of 16 per minute.The respiratory


rate must be maintained at a rate of at least 12 per
minute as a precaution against excessive
depression of impulses at the myoneural junction

A patient is gravida 3 para 2 and is in a labor


room. After a vaginal exam, it is determined that
the presenting head is at station +3. The
appropriate nursing action is to

Prepare for delivery of the baby.f the head is +3, it


is just about crowning, and because the patient is a
multipara, it would be reasonable to assume
delivery is imminent.

Counseling a patient who is starting to use oral


contraception, the nurse explains that birth control
pills work by the mechanism of

Birth control pills are small doses of estrogen and


progesterone that maintain sufficient levels in the
body to inhibit the pituitary from producing the
follicle-stimulating hormone.

During a physical exam of an infant with


congenital hip dysplasia, the nurse would observe
for which of the following characteristics?

Abduction is limited in the affected leg. The nurse


would also find asymmetrical gluteal folds and an
absent femoral pulse when the affected leg is
abducted.

If RhoGAM is given to a mother after giving birth


to a healthy baby, the condition that must be
present for the globulin to be effective is that the

RhoGAM will not work if there is any titer in the


blood; thus, it is important to administer it within
72 hours after delivery or abortion if the mother
shows no evidence of antibody production. The
mother would be Rh negative and the baby Rh
positive for RhoGAM to be needed.

The nurse is doing data collection on a postpartum


patient. Suspecting infection, the nurse would
assess for

The major symptoms of infection would be rapid


pulse, foul-smelling lochia or discharge, and
discomfort and tenderness of the abdomen

A patient, 36 weeks pregnant, is having a


contraction stress test (oxytocin challenge test).
After 35 minutes, her uterus begins to contract,
and the nurse observes three 40-second-long
contractions in a 10-minute period. She has two
contractions within 5 minutes, and her uterus
remains contracted after the second contraction.
The first nursing action is to

The first action is to turn the Pitocin off. If the


fetal heart rate has dropped in response to the
prolonged contraction, turning the mother on her
side (3) and administering oxygen (2) may be
necessary.

A nurse working in a prenatal clinic recognizes


that the physician should immediately see any
patient who presents with

Blurred vision is an advanced indicator of


pregnancy-induced hypertension (PIH) and the
physician should see the patient immediately

The nurse is assessing a 75-year-old patient who is


taking digitalis. Assessing for digitalis toxicity, the
nurse would identify

Anorexia, nausea, vomiting.

NL wbc

5-10 k

A patient has had a partial colectomy because of a


diagnosis of cancer. Surgery began at 7:30 AM.
She returned to the unit at 1:30 PM. During a 6:00
PM assessment, the nurse observed all of the
following. A priority concern that would require
the earliest intervention is a

Inability to void after surgery is a common


problem resulting from anesthesia or pain
medication and requires an early intervention. It is
important to be aware of the patient`s output for
several reasons: to ensure adequate intake, to
detect renal problems, and to assess for blood
pressure problems. Solution to this problem is
catheterization, based on a physician`s order. The
dressing (1) should be closely observed but is not
presently a problem. The area on the calf (2) may
be developing throbophlebitis and should be
reported to the physician immediatel

Administering care to a patient in hypovolemic


shock, the sign that the nurse would expect to
observe is

n shock, there is decreased blood volume through


the kidneys. This is evidenced by a decrease in the
amount of urine excreted. The body has numerous
compensatory mechanisms that assist in keeping
the blood pressure normal for a short time.

A systolic blood pressure of 60 mm Hg or less


would indicate shock in which of the following
patient age groups?

A systolic blood pressure of 60 mm Hg or less


found in children 5 to 12 years old would indicate
shock.

Removing the tube quickly while keeping it


pinched lessens the risk of gastric secretions
The nurse has an order to remove a patient`s
falling into the trachea during removal. Instilling
nasogastric tube. The correct nursing action related 20-30 mL of air, rather than normal saline, into the
to this procedure would be to
tube will also help prevent aspiration of gastric
secretions. Unsterile gloves are worn for this
procedure.

A patient is about to be discharged on the drug


bishydroxycoumarin (Dicumarol).

Dicumarol is an anticoagulant drug and one of the


dangers involved is bleeding. Using a safety razor
can lead to bleeding through cuts. The drug should
be given at the same time daily but not related to
meals. Due to danger of bleeding, missed doses
should not be made up. The LVN is prepared to do
this patient teaching.

The nurse is inserting a Foley catheter into a male


patient. How far should the catheter be inserted
before inflating the balloon?

7-9 inches.

The priority assessment for a patient with acute


infective (bacterial) endocarditis is

While all of the symptoms may be present, the


major complication with this condition is that of
emboli. If emboli arise in the right heart chambers,
they will terminate in the lungs; left chamber
emboli may travel anywhere in the arterial tree.
The nurse should constantly monitor for this
complication.

Basilar crackles are present in a patient`s lungs on


auscultation. The nurse knows that these are
discrete, noncontinuous sounds that are

Basilar crackles are usually heard during


inspiration and are caused by sudden opening of
alveoli.

In developing a nursing care plan for a patient with


Buerger`s disease, it is important to include

Buerger-Allen exercises improve peripheral


arterial circulation which is specific for treatment
of Buerger`s disease

The nurse is collecting data on a patient with joint


pain. The nurse knows that a patient who is in the
early stages of rheumatoid arthritis is most likely
to complain of pain, swelling, and limitation of
motion in the

Rheumatoid arthritis typically begins with


inflammatory changes in the small joints of the
hands, wrists, and feet.

The nurse would expect to find an improvement in


which of the blood values as a result of dialysis
treatment?

High creatinine levels will be decreased. Anemia


is a result of decreased production of
erythropoietin by the kidney and is not affected by
hemodialysis

A 50-year-old patient has a tracheostomy and


requires tracheal suctioning. The first intervention
in completing this procedure would be to

Before deflating the tracheal cuff, the nurse will


apply oral or nasal suction to the airway to prevent
secretions from falling into the lungs

The nurse is teaching a type 1 diabetic patient


about her diet, which is based on the exchange
system. The nurse will know the patient has
learned correctly when she says that she can have
as much as she wants of

Lettuce contains primarily water and fiber, and is


considered a "free food" in the American Dietetic
Association exchange lists.

A patient with a bile duct obstruction is jaundiced.


Itching is made worse by vasodilation. Tepid water
The priority intervention to control the itching
prevents excessive vasodilation
associated with jaundice is to

When using nasotracheal suction to clear a


patient`s airway of excessive secretions, a
principle of the suctioning procedure is to

To prevent trauma to the mucous membranes


lining the airway, suction should be applied only
while withdrawing the cathete

A Glasgow Coma Score of 3 - 5 - 4 means that the


A patient with an admitting diagnosis of head
patient is able to open his eyes when spoken to and
can localize pain, attempting to remove noxious
injury has a Glasgow Coma Score of 3 - 5 - 4. The
nurse`s understanding of this test is that the patient
stimuli when motor function is tested. He is not
able to follow commands

Part of a plan of care for a patient with increased


intracranial pressure is to maintain an adequate
airway and to promote gas exchange. To
accomplish these goals, an effective nursing action
is to

Hypercapnia leads to vasodilation, thus increasing


cerebral blood flow and increasing intracranial
pressure.

Based on nursing knowledge, the nurse is aware


that an epidural hematoma is characterized by

A short period of unconsciousness followed by a


lucid period, followed by rapid deterioration.
Epidural hematomas classically present with a
brief period of unconsciousness, followed by a
lucid interval of varying duration, and finally
followed by rapid deterioration of the level of
consciousness, accompanied by complaints of a
severe headache.

A patient is admitted following an automobile


accident in which he sustained a contusion. The
nurse knows that the significance of a contusion is
that

Laceration of the brain may occur.


Laceration, a more severe consequence of closed
head injury, occurs as the brain tissue moves
across the uneven base of the skull in a contusion.
Contusion causes cerebral dysfunction, which
results in bruising of the brain. A concussion
causes transient loss of consciousness and
retrograde amnesia, and is generally reversible.

A patient in the early stages of progressive renal


failure is admitted to the hospital. The initial
assessment will probably reveal

Polyuria, low urine specific gravity, polydipsia.

The nurse will assess for the most significant


complication in patients undergoing chronic
peritoneal dialysis, which is

Peritonitis is a grave complication with peritoneal


dialysis. Hemodialysis may be necessary until
infection clears. Excess fluid and protein effluent
into the peritoneum also complicate care. Use of
aseptic technique is essential

The most important teaching the nurse should do


for a patient to have well managed intermittent
hemodialysis is

It is essential that the end-stage renal patient


adhere to all aspects of the medical regimen. Only
excess solutes and fluid are removed with dialysis.
Blood pressure management needs to be
consistent, not just between treatments, aspects of
care concerning concomitant anemia, and
phosphate/calcium/vitamin D imbalance, as well
as protein restriction and fluid restriction, must be
carried out at all times. The dialysis patient
continues to be uremic and has multisystem
problems that continue despite dialysis.

A 12-year-old patient has just been returned to the


unit following a tonsillectomy. A priority nursing
intervention during the postoperative period is to

Apple juice or water is given as soon as the patient


is awake and not hemorrhaging. Avoidance of
citrus juices will prevent irritation of the operative
site. The patient should be placed on his abdomen
or side to facilitate drainage and prevent
aspiration. Ice bags are applied to the neck to
prevent edema and bleeding.

A patient with chronic renal failure is on


continuous ambulatory peritoneal dialysis
(CAPD). Which nursing diagnosis would have the
highest priority?

Which of the following statements is true of


skeletal traction?

Imbalanced nutrition: less than body requirements.


There is a high risk of infection in patients
receiving CAPD because microorganisms can
enter the body by migrating around, or through,
the peritoneal dialysis catheter. They may also
enter through contaminated dialysate solutions.
The other diagnoses are not life threatening for a
patient on CAPD.
Fractures can be reduced because more weight can
be used than with skin traction.Because more
weight can be applied with skeletal traction, it can
be used to reduce fractures and maintain
alignment. It is not used commonly in the elderly
because of prolonged immobilization. It is not
preferred for children because some displacement
of fracture fragments is desirable to prevent
growth disturbance. Frequently, patients have
more mobility than they do with skin traction,
because balanced suspension is often incorporated
with skeletal traction.

Russell`s traction is easily recognized because it


incorporates a

Russell`s traction is a type of skin traction that


incorporates a sling under the knee that is
connected by a rope to an overhead bar pulley. It is
frequently used to treat femoral shaft fractures in
the adolescent.

When evaluating all forms of traction, the nurse


will check that the direction of pull is controlled
by the

Rope/pulley system.

A patient has had a cystectomy and


ure-teroileostomy (ileal conduit). The nurse is
assigned this patient in the postoperative period.
Which of the following observations indicates an
unexpected outcome and requires priority care?

Feces in the drainage appliance.


The ileal conduit procedure incorporates
implantation of the ureters into a portion of the
ileum that has been resected from its anatomical
position and now functions as a reservoir or
conduit for urine. The proximal and distal ileal
borders can be resumed. Feces should not be
draining from the conduit.

A patient requires that a bronchoscopy procedure


be done. Due to his physical condition, he will be
awake during the procedure. As part of the pretest
teaching, the nurse will instruct him that before the
scope insertion, his neck will be positioned so that
it is

Hyperextension brings the pharynx into alignment


with the trachea and allows the scope to be
inserted without trauma.

A patient with chronic lymphocytic leukemia is


started on chemotherapy. Monitoring the
administration of these drugs, the nurse would
suggest dietary guidance to

Consume fluids and foods high in bulk and fiber


several hours before the treatment.
R/T R/F constipation

Which one of the following conditions could lead


to an inaccurate pulse oximetry reading if the
sensor is attached to the patient`s ear?

Hypothermia or fever may lead to an inaccurate


reading. Artificial nails may distort a reading if a
finger probe is used. Vasoconstriction can cause an
inaccurate reading of oxygen saturation. Arterial
saturations have a close correlation with the
reading from the pulse oximeter as long as the
arterial saturation is above 70 percen

There is a physician`s order to irrigate a patient`s


bladder. The priority nursing measure to ensure
patency is to

Irrigate with 20 mL of normal saline to establish


patency.

When assessing an ECG, the nurse knows that the


P-R interval represents the time it takes for the

Impulse to travel to the ventricles

When a patient has suffered severe burns all over


his body, the most effective method of monitoring
the cardiovascular system is

Pulmonary artery pressure is the most effective


method of monitoring the cardiovascular system
for this patient. Patients with a large percentage of
burned body surface often do not have an area
where a cuff can be applie

A female patient has orders for an oral


cholecystogram. Prior to the test, the nursing
intervention would be to

Explain that diarrhea may result from the dye


tablets.

Knowing that a patient has the diagnosis of heart


failure (HF), what symptoms would the nurse
assess during data collection?

Cyanosis, crackles, gallop rhythm

The laboratory result that should be monitored


regularly in a patient who is receiving gentamicin
(Garamycin) is

Platelets.

A patient with thrombophlebitis should be


positioned so that his legs are

Elevating the legs about 30 degrees promotes


venous return and reduces leg edema. Elevation
beyond 45 degrees reduces arterial flow and
causes sharp flexion at the hip, thereby reducing
venous return. Leaving the legs flat on the bed or
dependent promotes edema formation and venous
stasis. Patients with arterial, rather than venous,
insufficiency benefit from a dependent position.

A hypothyroid patient has orders for all of the


following medications. The nurse would evaluate
the patient most closely following administration
of which medication?

Hypothyroidism reduces the metabolic rate and


prolongs the sedative effects of medications.
Narcotics are especially dangerous and should be
given in smaller doses. The patient must be closely
monitored for signs of oversedation and
respiratory depression.

A patient with COPD has developed secondary


polycythemia. Which nursing diagnosis would be
included in the care plan because of the
polycythemia?

Chronic hypoxia associated with COPD may


stimulate excessive RBC production
(polycythemia). This results in increased blood
viscosity and the risk of thrombosis. The other
nursing diagnoses are not applicable in this
situation.

A colostomy appliance should be cut to fit the


The nurse is teaching a patient with a new
stoma so that there is no pressure placed on the
colostomy how to apply an appliance to a
stoma by the appliance and there is a minimum
colostomy. How much skin should remain exposed
amount of skin exposed to fecal drainage. Leaving
between the stoma and the ring of the appliance?
1/8 inch of skin exposed conforms to these criteria.

The liver becomes engorged with blood in


right-sided heart failure. Liver function studies,
Which of the following blood chemistry results
such as the LDH, an enzyme production test for
would the nurse expect to find elevated in a patient
the liver, will be abnormally elevated in 40 percent
with right-sided heart failure?
of the patients. Serum bilirubin is also frequently
increased

The treatment prescribed for the burned area of


skin before skin grafting can take place will
include

In addition to the germicidal soap scrubs, systemic


antibiotics are administered to prevent infection of
the wound. Silver nitrate is not a common
treatment today

When a head injury patient has fluid draining from


the left ear, the nurse will immediately position the
patient with the head of his bed

The patient has arrived in the recovery room


following a lobectomy. As the nurse assigned to
care for the patient during the immediate
postoperative period, the first intervention will be
to

It is important to decrease intracranial pressure


(head of bed elevated) and to allow for drainage
(head turned to left)

Closed chest drainage is used for lobectomies to


reestablish negative pressure in the chest. Because
the breathing mechanism operates on the principle
of negative pressure, this is an essential action.
The other interventions would follow this one.
Connect the Pleur-evac to suction.

A nursing care plan for a patient with a suprapubic


cystostomy would include

Allowing the patient to void naturally will be done


prior to removal of the catheter to ensure adequate
emptying of the bladder

A patient admitted for possible bleeding in the


cerebrum has vital signs taken every hour to
monitor the neurological status. Which of the
following neurological checks will give the nurse
the best information about the extent of bleeding?

Pupillary checks reflect function of the third


cranial nerve, which stretches as it becomes
displaced by blood, tumor, etc.

Patient teaching following cataract surgery should


include

They must use only one eye at a time to prevent


double vision.
The function of the lens is that of accommodation,
the focusing of near objects on the retina by the
lens; therefore, only the remaining lens will
function in this capacity, depending on whether a
cataract is present.

Preoperative teaching for a patient scheduled for a


laryngectomy should include the fact that

The patient will be able to speak again, but it will


not be the same as before surgery.
Most of the laryngectomy patients will use
esophageal speech or a mechanical device for
communication. They can usually begin to take
oral fluids sometime after 48 hours. They are
generally fed by an intravenous or nasogastric tube
prior to oral feedings. Because the larynx is
removed, it will be impossible to breathe through
the nose.

he main complication following a nephrostomy


that the nurse must assess for is

While all the other conditions may be


complications, bleeding from the site is the main
concern. The procedure is done to achieve relief
from infection caused by urinary stasis, which may
have resulted in kidney congestion.

Hemorrhage is a major complication following


oral surgery or radical neck dissection. If this
condition occurs, the most immediate nursing
intervention would be to

Putting pressure over the vessels in the neck may


be lifesaving because a severe blood loss can occur
rapidly, leading to shock and death. The surgeon
would be notified as soon as possible.

To achieve the desired outcome of fracture


healing, which nursing goal should receive the
highest priority?

Maintaining the prescribed immobilization and


body alignment will keep the fracture fragments in
close anatomical proximity, thereby promoting
functional fracture healing. This goal should
receive the highest priority. The other goals,
although applicable in the

The nurse, collecting data for a nursing history


from a newly admitted patient, learns that he has a
Denver shunt. This suggests that he has a history
of

The Denver shunt is a type of peritoneovascular


shunt used in the treatment of patients who have
cirrhosis with ascites. The shunt diverts ascitic
fluid from the abdomen into the jugular vein or the
vena cava

The nurse has been teaching a patient to use


crutches. Which statement made by the patient
indicates a need for more teaching?

The hand grips should be placed so that the elbows


are flexed at 20-30 degrees when standing with the
crutches. This placement should not be changed as
long as the patient continues to need crutches. The
other statements indicate effective learning.

The nurse is preparing a patient for a myelogram


using metrizamide (Amipaque), a water-soluble
contrast material. The nurse will know the patient
understands the postmyelogram care regimen
when she says

The head must be kept elevated because this drug


could provoke a seizure if it reaches the brain in a
bolus form. After myelography that uses an
oil-based contrast medium (Pantopaque), patients
are kept flat. Forcing fluids helps prevent
postmyelogram headache by replacing lost spinal
fluid. Itching suggests an allergic reaction, while a
stiff neck suggests meningeal irritation; neither is
an expected response to a myelogram.

The precaution protocol necessary to implement


for the biohazard of Pneumonic Plague is

Precautions include Standard plus Droplet (eye


protection and surgical mask)until 48 72 hours
after antibiotic treatment.

All staff must wear disposable particulate


respirators (HEPA filter) when

Staff must wear disposable respirators when there


is inadequate room ventilation. If the room has a
directional negative-pressure ventilation system,
the staff would not be required to wear a HEPA
filter mask, even if the patient had TB. These
masks are required for droplet transmissionbased
conditions.

The nurse is assigned to care for two patients. One


patient has just returned from surgery for an
abdominal resection. The second patient is
hospitalized with an acute case of tuberculosis.
What special precautions should the nurse take
when providing care for these two patients?

There are no special precautions; however, the


nurse must strictly adhere to barrier nursing
principles and the two patients must be treated
separately. Providing care to the abdominal
surgery patient before the TB patient would be
appropriate. Proper handwashing is essential (1),
but isolation garb is needed only for the TB patient

When removing an isolation gown, steps the nurse


should take would be to

Untie front waist strings, remove gloves, and untie


neck ties.

Two major factors that influence whether an


infection occurs in an individual are

Inherent health and immunologic status.

Which of the following is a type of


transmission-based precaution?

droplet

The nurse should explain to a patient who takes


Lasix and has a potassium of 3.2 mEq/L that he
should``

The normal potassium level is 3.5-5.0 mEq/L. The


patient`s potassium level is low, and he needs to
replenish what has been lost as a result of taking
the Lasix. In addition to taking potassium
supplements, the patient should be given a list of
the appropriate foods that have an average of 7
mEq potassium per serving. Eat three servings
daily of fruits and meat or fish.`

The premenstrual hemoglobin of a 24-year-old


patient with no history of trauma, recent surgery,
or hemorrhage is 9.8 g/dL. The nurse interprets
that this value is due to

The normal Hgb for a female is > 12-16 g/dL.


With the data given, the nurse would suspect
anemia Iron-deficiency anemia.

A patient is scheduled for a carotid endarterectomy


in 3 days. Which of the following preadmission
lab test results must be immediately reported to the
physician?

Sodium of 151 mEq/L.The normal electrolyte


values for an adult are as follows: sodium of
135-145 mEq/L, chloride of 100-106 mEq/L,
potassium of 3.5-5.0 mEq/L, and bicarbonate of
22-29 mEq/L. The serum sodium is the only
abnormal value.

At the physician`s office, a patient has a random


plasma glucose test. The results were 250 mg/dL.
The patient asked the office nurse why the doctor
told him to come back the next day to repeat the
test. The best Answer is

"This test requires that it be done at least twice for


accurate results."The best Answeris to be truthful,
but not to frighten the patient by telling him that he
may have diabetes (2) (this is the domain of the
physician). Levels of > 200 mg/dL on more than
one occasion would, however, be diagnostic of
diabetes, so the doctor would order at least two
tests.

A patient comes to the clinic complaining of a


variety of symptoms including pain. The patient
has a gastric analysis done and results show that
gastric acid is high. This test result would indicate
to the nurse that the patient may receive the
diagnosis of

High gastric acid levels may indicate a duodenal


ulcer.

A 60-year-old patient is admitted to the surgery


unit for removal of fibroid tumors. When the nurse
checks the lab results for routine blood chemistry,
she notes that the sedimentation rate is 29 mm/dL.
The appropriate intervention is to

This is a normal sed rate for a female over age 60.


Under age 50, normal is 20 mm/hr. If it were
increased, it would indicate presence of infection
or inflammation, and surgery might have to be
postponed

Because the patient is not on anticoagulant


A patient is admitted to the hospital for evaluation.
therapy, the results are abnormal (normal PT is
11-15 seconds). It is important to notify the head
His physician writes in the chart "rule out liver
nurse or physician before the biopsy; bleeding
cancer" and schedules a liver biopsy. Before the
procedure, the nurse reviews the PT results just
could be life threatening. The patient will probably
returned from the lab: 24 seconds. The nurse also
be given vitamin K therapy and when the PT
notes that this patient is not on an anticoagulant.
results return to the normal range, the procedure
can be done. Liver disease likely caused the
The nursing intervention is to
prolonged PT.

As part of an annual physical exam, a 60-year-old


man has had lab work done. Which of the
following serum creatinine levels would indicate
that the patient has a mild degree of renal
insufficiency?

The normal serum creatinine level for a male is


0.6-0.9 mg/dL. A patient with a mild degree of
renal insufficiency would have a slightly elevated
level, which in this case would be 1.7. Levels of
3.3 (2) and 4.0 (1) may be associated with acute or
chronic renal failure.

A patient with damaged or impaired lungs cannot


remove all of the CO2 from the body. When the
excess CO2 combines with H2O, it will form

Excess CO2 in the blood, when combined with


H2O, forms H2CO3, carbonic acid. Depending on
the amount of acid in the blood, the lungs will
increase or decrease ventilation to remove excess
CO2 (4). The kidneys can excrete or retain H+ (3)
and HCO3 (2); thus, the equation representing
homeostasis is: CO2 + H2O = H2CO3 = H+ +
HCO3.(Lungs) (Kidney)

An 80-year-old patient has been admitted to the


hospital with influenza and dehydration. Which of
the following blood urea nitrogen (BUN) levels
would indicate to the nurse that the patient has
received adequate fluid volume replacement?

The normal BUN is 10-20 mg/dL

Which group of cells is the first line of defense


against bacterial infection working primarily
through phagocytosis?

Neutrophils are the first line of defense against


infection. They live in the circulation for about 6
hours after bacteria are ingested. The cells die and
become the main component of pus. Monocytes
(1) are the second group to defend the body.
Platelets (2) are blood components that go to the
site of injury and stem blood loss. Basophils (4)
release heparin and histamine in areas that are
invaded by antigens.

A 53-year-old patient with Crohn`s disease is


placed on total parenteral nutrition (TNA). The
fluid in the present TNA bottle should be infused
by 8 AM. At 7 AM, the nurse observes that it is
empty and another TNA bottle has not yet arrived
on the unit. The nursing action is to attach the
solution nearest a TNA solution which is a bottle
of

In order that the patient not experience a sudden


drop in blood sugar, the solution nearest most TPN
solution concentrations is D10W.

The nurse`s discharge teaching for a patient with


acute pancreatitis will include advising him to take
a dietary supplement of

Because the patient will be on a low-fat diet to


decrease pancreatic activity, he will need
supplements of the fat-soluble vitamins

The nurse will know that the patient understands


presurgical instructions for hemorrhoid surgery if
his diet is

A high-fiber diet produces a soft stool without


mechanically irritating the hemorrhoidal area.
Foods include bran and complex carbohydrates.

The nurse`s diet instructions for a patient with a


colostomy will be

Diets are individualized and patients are generally


able to eat the same foods they enjoyed
preoperatively. Fresh fruits may cause diarrhea in
some, but not all, individuals.

Which of the following statements would be


correct when counseling a patient about the
postoperative diet he would receive following a
simple surgical procedure?

A daily intake of 2800 calories is required for


usual/general tissue repair, whereas 6000 calories
may be required for extensive tissue repair. Fluid
intake is 2000 to 3000 mL/day for uncomplicated
surgery. Diet progresses from nothing by mouth
the day of surgery to a general diet within a few
days.

The nurse will know that the diabetic patient


understands his diet when he says that he should
obtain the greatest percentage of calories from

The diabetic`s diet should be between 50 and 65


percent carbohydrate calories with only 5 percent
of these being simple carbohydrates (sucrose). Fat
recommendation is less than 30 percent of calories,
and protein should be 0.8 mg/kg/day.
complex carbs

A patient with acute pancreatitis required


nasogastric intubation due to persistent vomiting
and paralytic ileus. Following NG tube removal,
the feeding schedule would start with a diet that is

Foods that are high in carbohydrate are given,


because those with high protein or fat content
stimulate the pancreas. Alcohol is forbidden. There
is no need for the patient to be NPO.

A patient with cirrhosis and ascites is placed on a


sodium-restricted diet to help control the ascites.
In order for this plan to be effective, it is important
that the patient also

It is important that fluids be restricted as well,


because unrestricted fluid intake leads to a
progressive decrease in serum sodium from
dilution. Electrolyte imbalance with potential
neurologic complications could result.

A patient with a history of pancreatitis should


avoid which of the following foods?

Patients with this condition must not consume


foods high in fat content because there are
inadequate pancreatic enzymes to digest the fat.
High fat content also causes pain 2 to 4 hours after
ingestion. The suggested diet is high in
carbohydrates.

The nurse questions the dietary department about


the lunch delivered for a patient with the diagnosis
of cirrhosis when she finds on his tray

Ham is high in sodium and can increase fluid


retention, leading to edema. Cirrhosis patients are
prone to edema as the osmotic pressures change
due to a decrease in plasma albumin.

The nurse will know that her teaching has been


effective when the patient responds that a
low-fiber diet allows the inclusion of

Cooked vegetables and fruits as well as refined


breads are included in a low-fiber diet. Bran, fresh
fruits, and whole grains and seeds are included in a
high-fiber diet.

Patients with hepatitis may have a regular diet


ordered, unless they become increasingly
symptomatic. The diet will then be modified to
decrease the amount of

With liver cell damage, the liver cannot break


down and eliminate protein. Protein needs to be
decreased until symptoms dissipate.

A pregnant patient comes to the clinic, and the


nurse is responsible for nutritional counseling.
When the patient says that she has eliminated all
salt from her diet, the nurse should respon

Research has indicated that pregnant women


require a moderate amount of salt, because it is
essential in maintaining increased body fluids
needed for adequate placental and renal flow as
well as tissue requirements. Highly salted foods
should still be avoided

Following surgery, a patient has an IV of D5W to


run 50 mL/hr. When the nurse checks his
condition for the evening shift, she realizes the IV
is 1 hour behind. The first action would be to

Increase the flow so that the loss is made up over


the remaining hours in the day.
Hmmmmm?

One of the duties of the rehabilitative nurse is to


teach the activities of daily living (ADLs) to a
patient about to be discharged. One of the most
important nursing interventions to accomplish
these goals is to

One of the most important principles of teaching is


to demonstrate the activity, encourage the patient
to perform, and then give positive reinforcement.
It is important that the patient learn to do these
activities him- or herself

The nurse responsible for administering a thiazide


medication to a patient evaluates his recent lab
reports, which are K+ 3.0 and NA+ 140. The nurse
would

The appropriate intervention is to withhold the


thiazide medication (until the nurse receives
further orders) and report the K+ level to the
physician. Normal K+ is 3.5-5.5 mEq/L. His NA+
level is normal (range 135-145 mEq/L)

A patient scheduled for surgery is given a spinal


anesthetic. Immediately following the injection,
the nurse will position the patient

Usually, the patient is positioned on the back


following the injection. If a high level of
anesthesia is desired, the head and shoulders can
be lowered to slight Trendelenburg`s. After 20
minutes the anesthetic is set, and the patient can be
positioned in any manner.

The nurse is assigned to closely observe a patient


for signs of magnesium toxicity following an IV of
4 g magnesium sulfate in 250 mL D5W. The first
indication of this condition is

The first sign that the nurse will observe is


probably extreme thirst. There will also be a loss
of the patellar REFLEX

When a person is experiencing severe stress, the


nurse would assess for behaviors such as

Crying and being upset is typical behavior


experienced when a person is under stress.
Restlessness and anxiety (1) might be present, but
they are not typical responses

According to Selye`s stress theory, when the


individual is in the alarm phase of the general
adaptive syndrome, the body first responds by

Going into shock and countershock.

At which of the following ages would the nurse


first expect a child to sit with no support?

nfants begin to sit with support or leaning forward


on both hands at 6 months. They sit with minimal
or no support between 7 and 8 months. If this
milestone does not occur, the infant should be
assessed for retardation.

The patient is age 4 and while in the hospital, he


becomes very bored. The best activity to
implement for this patient is

Fantasy is very active in this stage of development.


Puppets would allow for expression of feelings.
Also, this activity is more active than TV (1) or
books (3) and involves the nurse with the child,
which is a positive way of establishing a
relationship.

The ultimate outcome, when the grieving process


is successfully completed, will be when the
bereaved

When the grieving process is completed, the


bereaved will no longer feel emotionally
dependent on the person who died. They will
always feel emotion (1) when thinking of the loved
one, but they will be able to realistically recall
both the good and bad times. There will always be
the need to talk about the loved one (4), even when
the grief has been resolved.

In the presence of the RN, a physician asks the


LPN to remove the sutures from the incision
before the patient is discharged. The initial
response to the physician should be

"Please write the order and the sutures will be


removed."

The LVN observes the nursing assistant (NA)


regulating the IV of an oncology patient receiving
morphine sulfate for pain. The LVN is responsible
for the patient and has assigned the patient to the
NA. The appropriate intervention is to

Ask the NA to meet with the RN and him/her to


discuss the responsibility parameters that are
appropriate for the NA.

The LVN assigns a patient with uremic frost from


renal failure to the UAP. The patient is
complaining of dry, itchy skin. To alleviate this
problem during bathing, the nurse will instruct the
UAP to use

A weak vinegar solution with no soap Vinegar


solutions may alleviate itching by dissolving
crystal deposits in cutaneous layers and leaving an
acid layer on the skin. .

A state's Nurse Practice Act would not include

Difference between RN and LVN functions.


Each state has its own Nurse Practice Act for RNs
and LVNs. Separately, they are a series of statutes
enacted by a state to regulate the practice of
nursing in that state. It includes all of these plus
education.

The nurse is asked to do a TV commercial for


hand lotion. In this commercial, she will wear her
nurse`s uniform and advocate the use of this lotion
by nurses in their work setting. In doing this, the
nurse is violating

The code of ethics is a set of formal guidelines for


governing professional action. This situation is not
illegalit is unethical.

The physician wrote a medication order for a


patient. The LVN thought the dosage was
incorrect. She questioned the physician who said it
was all right. Still questioning, she asked the RN,
who said it was all right. The LVN gave the
medicine, and the patient died from an overdose.
Who is liable?

Both the physician and the nurse who gave the


medication.

The primary purpose and criteria of licensure is to

The primary purpose of licensing nurses, both RN


and LVN, is to safeguard the public by
determining that the nurse is a safe and competent
practitioner.

A nursing assessment on a male patient indicates


that he is showing evidence of increased
intracranial pressure. The first nursing action is to

As the PaCO2 increases in the cerebral tissues,


blood rushes to the area and this further increases
the intracranial pressure. Decreasing the PaCO2,
accomplished by breathing deeper and more
slowly, will decrease the intracranial blood flow,
thus decreasing intracranial pressure.

The nurse is caring for a patient undergoing


chemotherapy for cancer. One of the goals of care
is to discuss possible side effects of this therapy,
one of which is

Alopecia, or hair loss, will probably occur caused


by damage to the rapidly dwindling cells of hair
follicles. Hair loss begins 2 to 3 weeks after
therapy begins and continues through the course of
therapy. The other side effects listed do not occur,
as nausea, anorexia and diarrhea are common.

A neighbor asks the nurse to look at her 3-year-old


child`s rash. Her face, neck and chest are covered
with a maculopapular rash. She appears feverish
and her nose and eyes are "running." There are
small red spots with bluish-white centers on the
mucosa of her mouth. The nurse recognizes that
these manifestations are most likely caused by

Rubeola is a highly contagious virus. It is more


severe than rubella or roseola because of the
complications. The virus is transmitted by a cough
or sneeze.

The ball in socket position is maintained by


A patient is several days postoperative following a
minimizing hip flexion (60 degrees or less). This is
right hip replacement with a prosthesis. Assisting
done with wheelchair and commode extenders,
the patient to sit in a chair, the nurse will use
high chairs, and proper bed positions.

A patient has sustained a femoral shaft fracture


and is being treated with skeletal traction using
balanced suspension with a Thomas splint and
Pearson attachment. The goal of maintaining
optimum positioning will be accomplished by

It is important that the established angle between


the affected thigh and the bed be maintained. The
patient can usually have the head of the bed flat or
elevated and the lower leg can be exercised, then
rest in the Pearson attachment. If the patient
migrates toward the head or foot of the bed, then
the angle between the thigh and bed would be
altered, so it is important that adequate
countertraction be maintained.

Which one of the following conditions


contraindicates giving morphine?

Morphine causes spasm of the gallbladder and the


common bile duct, thus worsening the pain if
stones are present. The spasm and resulting
inflammation can also block the pancreatic outlet
(ampulla of Vater). Meperidine (Demerol) would
be a safe choice for pain relief

A patient has been in a motor vehicle accident and


Brown-Squard syndrome is caused by
has received cervical and spinal stabilization. He is
hemisection of the spinal cord. Clinical
alert and oriented with no evidence of head injury. manifestations include paralysis below the level of
He develops lower extremity paralysis on the same injury on the same side as the lesion, and the loss
side as the wound and loses pain and temperature
of the perception of pain and temperature below
sensation on the side opposite the injury. Based on
the level of injury on the opposite side of the
the preceding information, this type of spinal
lesion. A total transection would lead to paralysis
injury response would be referred to as
with no sensation

Apnea of prematurity is defined as a pause

Apnea of prematurity is defined as: 1. cessation of


breathing for 20 seconds or longer. 2. cessation of
breathing for less than 20 seconds,but associated
with cyanosis, bradycardia or limpness

The nurse is interviewing a patient who has been


addicted to opiates. The patient tells the nurse that
he cannot understand why his drug screening test
was positive for opiates, as he is not using the
drugs now. Which question would help the nurse
understand why the patient`s urine test was
positive for opiates?

Ingestion of ibuprofen may cause a false positive


on a urine screen for opiates

The father of a 24-hour-old baby boy asks how to


hold his son, Christopher, who was circumcised 15
minutes ago. The nurse should respond

"There are a variety of ways to hold a baby. Let


me show you a few that I find comforting to
newborns."

Which of the following data place a patient at risk


for developing pregnancy-induced hypertension
(PIH)?

Primigravid women or women pregnant for the


first time after an abortion are more susceptible to
PIH than are multiparous women.

A 54-year-old female patient is 3 days


Do nothing because this is a normal count.This is a
postoperative following abdominal surgery. The
normal cell count (the range is 4500 to 11,000/cu
lab results indicate that her white blood cell count
mm) so the nurse would do nothing except note
is 8,000/cu mm. After analyzing this lab report, the
that it is normal.
nursing action is to

When educating the patient about prevention of


coronary heart disease and about risk factors,
which of the following would the nurse describe as
a predisposing pathologic factor?

CHD is significantly higher in persons with


pathologic conditions such as hypertension,
hyperlipidemia, and glucose intolerance (such as
diabetes mellitus). Gender is an unmodifiable risk
factor. Smoking and stress are considered
modifiable risk factors.

During visiting hours, a patient the nurse is caring


for becomes very agitated and angry with his
visitor. The most effective nursing approach to this
patient is to

Approach your patient in a warm, supportive


manner and assist him to explore his feelings

The nurse is teaching the family of a patient with


adrenocortical insufficiency. It is important to
assess coping skills and family functioning of
patients with this condition because

Clinical findings in Addison`s disease reflect


involvement in most body systems including
psychologic and behavioral changes.

When planning meals with a patient who is


hemodialyzed three times per week, which of the
following food selections would indicate the
patient understands the dietary guidelines

Apple juice is low in phosphorus, sodium,


potassium, and protein. The patient in renal failure
must avoid these nutrients, because the damaged
kidney cannot clear them. Bananas are high in
potassium. Red meat contains protein and
phosphorus. Legumes contain phosphorus.

A quadriplegic patient tells the nurse that he


believes he is experiencing an episode of
autonomic hyperreflexia (dysreflexia). The first
nursing intervention is to

Elevate his head as high as possible.Blood


pressure can become dangerously elevated during
an episode of dysreflexia and can cause cerebral
and retinal hemorrhages. Elevating the head will
help prevent these complications and should be the
nurse`s first action. Identifying the precipitant is
useful in terminating the episode by removing the
noxious stimulus which provoked the exaggerated
autonomic response. A full bladder may
precipitate dysreflexia and emptying the bladder
would be appropriate if it was the precipitant. The
blood pressure and pulse should be monitored
throughout the episode of dysreflexia.

Dry heat (AquaK pad) is ordered for a patient with


Because the affected veins are inflamed, the use of
the diagnosis of right leg thrombophlebitis. The
orders read: apply to patient`s right calf
heat is indicated to help relieve the inflammation
and venospasm
continuously for 20 hours. The nurse would expect
that this treatment would

The most important consideration when working


with or teaching older adult patients is to evaluate
their

The developmental stage is very important when


developing plans and when considering the health
needs of the individual patient. Occupation, social
relationships, and economic status should also be
considered in working with the elderly, but these
factors are not as important.

A patient is admitted to the labor room with


contractions that are about 4 to 5 minutes apart and
30 seconds long. The couple have attended
Lamaze childbirth classes. During transition, the
nurse should support a breathing pattern that is

Shallow chest breathing, slightly faster than a


normal rate, is the pattern used most in transition.
Slow breathing, while effective for some women
during transition, is usually most effective in
earlier labor

When catheterization is performed to obtain a


urine specimen for culture from a young child, the
procedure would include

Urinary catheters come in a variety of sizes


however infants have urethras that are too small
even for the smallest urinary catheters. For these
infants, 5 Fr and 8 Fr feeding tubes are used to
obtain a specimen. All catheters must be sterile
and well lubricated.

Infants with sickle cell disease rarely show any


symtpoms before 4 months of age because

Fetal hemoglobin resists sickling.

The primary purpose for adult patients having


executed a valid living will is that it

Protects health care providers who abide by its


provisions against being charged with criminal
negligence.

A 29 year old is in the labor room and has been


there for 6 hours. It is the change of shift and,
when the nurse is receiving report, a nurse`s aide
rushes in and says that the patient is delivering. As
the nurse enters the room, the baby`s head is
crowning. The first nursing action is to

Support the baby`s head with a clean or sterile


towel.

When instructing a group of mothers-to-be, which


of the following would the nurse note as the virus
that most commonly causes acute diarrhea?

Rotavirus is the most common virus causing


diarrhea in young children and accounts for 50%
of the admissions for dehydration and diarrhea.

A nurse is in a medication-oriented group therapy


session with patients who are all on medication
therapy. One of the patients says, "I`m sick and
tired of taking these drugs." Many group members
express similar feelings. In addition to discussing
the patients` feelings and the importance of and
reasons for medication therapy, the group should
have a goal that patients will

Work closely with staff when they discontinue


medications.

In a generalized seizure the airway may be


compromised during the period of the

During the tonic phase of a seizure, the individual


loses consciousness, and continuous muscular
contractions occur. If the tonic phase is prolonged,
the person may be unable to breathe because the
respiratory muscles are contracted. This can result
in hypoxia. During the clonic phase, there is a
rhythmic jerking of extremities. The postictal
phase is the postseizure phase when the person is
relaxed.

Potrebbero piacerti anche